You are on page 1of 109

Course Notes for

MA 561& 661
ABSTRACT ALGEBRA
Alberto Corso
Contents
Chapter 1. Basics 5
1. Basic set theory 5
2. Basic properties of Z 5
3. Z/nZ the integers modulo n 5
Chapter 2. Introduction to groups 7
1. Basic axioms and examples 7
2. Dihedral Groups 7
3. Permutation groups 7
4. Matrix groups 7
5. The Quaternion group of order 8 7
6. Homomorphisms and isomorphisms 7
Chapter 3. Subgroups 9
1. Denition and Examples 9
2. Centralizers, Normalizers and Kernels 9
3. Cyclic Groups and Cyclic Subgroups 9
4. Subgroups Generated by Subsets of a Group 9
5. The Lattice of Subgroups of a Group 9
Chapter 4. Quotient Groups and Homomorphisms 11
1. Cosets and Normal Subgroups 11
2. More on Lagrange Theorem and Cosets 11
3. The Isomorphisms Theorems 12
4. Simple and Solvable Groups 12
5. The Holder Program (Holder 18591937) 14
Chapter 5. Group actions 17
1. Group actions and permutation representation 17
2. Transpositions, Alternating groups, Conjugacy in S
n
and the Simplicity of A
n
20
3. Automorphisms 22
4. Sylows Theorem 24
Chapter 6. Introduction To Rings 31
1. Basic Denitions and Examples 31
2. More Examples of Rings 33
3. Ideals and Ring Homomorphisms 35
4. The Chinese Remainder Theorem 38
5. Prime, Primary and Maximal Ideals 40
6. Quotient Rings (or Rings of Fractions) 43
7. Ane Schemes 45
Chapter 7. Factorization in Rings 51
1. UFDs, PIDs and Euclidean Domains 51
2. Polynomial rings 54
3. Roots of Polynomials 56
4. Polynomial Rings over UFDs 58
5. Hilberts Basis Theorem 60
3
4 CONTENTS
Chapter 8. Fields and Field Extensions 63
1. Basic properties of eld extensions 63
2. Ruler and compass constructions 65
3. Algebraic closure 66
4. Splitting elds and normal extensions 68
5. Separable extensions 69
6. Purely inseparable extensions 73
7. Finite elds 75
Chapter 9. Galois Theory 77
1. Galois extensions 77
2. The Galois group of a polynomial 81
3. Symmetric functions 84
4. Cyclotomic elds 86
5. Characters 87
6. Norm and trace 88
7. Cyclic extensions 89
8. Solvable and radical extensions 90
Chapter 10. Solution to Homework Problems 97
Chapter 11. Exam Material 105
Bibliography 109
CHAPTER 1
Basics
1. Basic set theory
A set is just a collection of objects. Let A and B be sets then AB = x[ x A and x B
is the intersection of A and B. By the union of A and B is meant A B = x[ x A or x B.
Let X be a set containing A. By the complement of A in X is the set A
C
X
= x X[ x , A.
Note X = A A
C
X
and A A
C
X
= , the so called empty set.
Example 1.1.
Z = x[ x is an integer = 0, 1, 2, . . .
= x[ x is a rational number =
_
a
b
[ a, b Z, b ,= 0
_
1 = x[ x is an real number
C = x[ x = a +ib, a, b 1, i
2
= 1 = set of complex numbers.
Z
+
,
+
and 1
+
will denote the positive (non-zero) elements in Z, and 1, respectively.
Let A and B be sets. By a function of A into B is meant a rule of correspondence f such that
for each a A there is a unique b in B to which a corresponds to under f; we write f(a) = b.
We also write f : A B to denote the function of A into B and observe that f is also called a
mapping or map from A into B. A is called the domain of f and f(A) = f(a) [ a A B is
called the range or image of f. B is called the codomain of f. Let C B, C is a subset of B.
Then f
1
(C) = a A[ f(a) C is called inverse image of C.
Let f : A B be a mapping. Then
1. f is called surjective or onto if B is the range of f.
2. f is called injective or 1-1 if whenever f(a
1
) = f(a
2
), with a
1
, a
2
elements of A, then
a
1
= a
2
.
3. f is called bijective provided that it is injective and surjective.
2. Basic properties of Z
3. Z/nZ the integers modulo n
5
6 1. BASICS
Homework set # 1.
The due date is on September 5 (Wednesday), 2001.
1. Determine whether the following functions f are well dened:
f : Z dened by f(a/b) = a.
f : dened by f(a/b) = a
2
/b
2
.
2. Let f : A B be a surjective map of sets. Prove that the relation
a b if and only if f(a) = f(b)
is an equivalence relation whose equivalence classes are the bers of f.
3. For each of the following pairs of integers a and b
a = 20, b = 13
a = 792, b = 275
a = 11391, b = 1567
determine their greatest common divisor (a, b), their least common multiple [a, b], and
write (a, b) in the form ax +by for some integers x and y.
4. Show that if the integer k divides the integers a and b then k divides as +bt for each pair
of integers s and t.
5. Let a, b be nonzero integers and let p be a prime such that p divides ab. Show that p
divides either a or b. However, prove that if n is composite (i.e., not prime) then there
are integers a and b such that n divides ab but n does not divide either a or b.
6. Determine the value of (n) for each integer n 30 where denotes Eulers -function.
7. If p is a prime prove that there do not exist nonzero integers a and b such that a
2
= pb
2
(i.e.,

p is not a rational number).
8. Prove that if d divides n then (d) divides (n) where denotes Eulers -function.
9. Prove that if a = a
n
10
n
+ a
n1
10
n1
+ + a
1
10 + a
0
is any positive integer then
a a
n
+a
n1
+ +a
1
+a
0
(mod 9).
10. Compute the remainder when 37
100
is divides by 29.
11. Prove that the squares of the elements in Z/4Z are just 0 and 1.
12. Using the previous exercise, prove for any integers a and b that a
2
+ b
2
never leaves a
remainder of 3 when divided by 4.
13. Prove that if a, b (Z/nZ)

then a b (Z/nZ)

.
14. Let n Z, n > 1, and let a Z with a 1 n. Prove if a and n are not relatively prime,
there exists an integer b with 1 b < n such that ab 0 (mod n).
15. For the pair of integers a = 13 and n = 20, show that a is relatively prime to n and
determine the multiplicative inverse of 13 in Z/20Z.
CHAPTER 2
Introduction to groups
1. Basic axioms and examples
2. Dihedral Groups
3. Permutation groups
4. Matrix groups
5. The Quaternion group of order 8
6. Homomorphisms and isomorphisms
7
8 2. INTRODUCTION TO GROUPS
Homework set # 2.
The due date is on September 17 (Monday),2001.
1. Let G be a nite group and let x be an element of G of order n. Prove that if n is odd,
then
x = (x
2
)
k
for some k.
2. If x is an element of nite order n in G, prove that the elements 1, x, x
2
, . . . , x
n1
are all distinct. Deduce that [x[ [G[.
If x is an element of innite order in G, prove that the elements x
n
, n Z, are all
distinct.
3. Let G be any group. Prove that the map from G to itself dened by g g
1
is an
homomorphism if and only if G is abelian.
4. Let G be a nite group of even order. Dene t(G) = g G[ g ,= g
1
. Show that t(G)
has an even number of elements. Conclude that G contains an element of order 2.
5. Let G be a group such that (ab)
i
= a
i
b
i
for three consecutive integers i and all a, b G.
Prove that G is abelian.
6. Let D
2n
= r, s [ r
n
= s
2
= 1, rs = sr
1
.
Show that if x is not a power of r then rx = xr
1
.
Show that any such x has order 2.
If n = 2k is even and n 4, show that r
k
is an element of order 2 and that it
commutes with all the elements of D
2n
.
7. Find a set of generators and relations for Q
8
.
8. Write out the group tables for D
8
and Q
8
.
9. Prove that D
8
and Q
8
are not isomorphic.
10. Show that Q
8
is isomorphic to the subgroup of GL(2, C) generated by
_
0 1
1 0
_
and
_
0 i
i 0
_
.
11. Show that D
8
is isomorphic to the subgroup of GL(2, 1) generated by
_
0 1
1 0
_
and
_
0 1
1 0
_
.
12. Show that the following two groups of order four: Z
4
and Z
2
Z
2
are not isomorphic.
13. Prove that the order of GL(2, F
2
) is 6. Note that F
2
= Z
2
.
14. Prove that if is the m-cycle (a
1
a
2
. . . a
m
) then for all i 1, 2, . . . , m then
i
(a
k
) =
a
k+i
where k + i is replaced by its least residue mod m when k + i > m. Deduce that
[[ = m.
15. Let be the m-cycle (1 2 . . . m). Show that
i
is also an m-cycle if and only if i is
relatively prime to m.
CHAPTER 3
Subgroups
1. Denition and Examples
2. Centralizers, Normalizers and Kernels
3. Cyclic Groups and Cyclic Subgroups
4. Subgroups Generated by Subsets of a Group
5. The Lattice of Subgroups of a Group
9
10 3. SUBGROUPS
Homework set # 3.
The due date is on October 3 (Wednesday), 2001.
1. Prove that a group G cannot have a subgroup H with [H[ = n 1, where n = [G[ > 2.
2. Let H
1
H
2
be an ascending chain of subgroups of a group G. Prove that

_
i=0
H
i
is a subgroup of G.
3. Let G be an abelian group. Prove that H = g G[ [g[ < is a subgroup of G,
called the torsion subgroup of G.
Give an explicit example where this set is not a subgroup when G is non-abelian.
4. Let H be a subgroup of the group G.
Show that H N
G
(H).
Show that H C
G
(H) if and only if H is abelian.
5. Let H be a subgroup of order 2 in G. Show that N
G
(H) = C
G
(H). Deduce that if
N
G
(H) = G then H Z(G).
6. Prove that the subgroup generated by any two distinct elements of order 2 in S
3
is all of
S
3
.
7. Draw the lattice subgroup of: Z/16Z and Z/24Z.
8. Let G = 1, a, b, c be a group of order 4. Show that either G

= Z/4Z or G

=
Z/2Z Z/2Z.
9. Let G = x be a cyclic subgroup of order n. Our goal is to show that the group
(Aut(G), ) of automorphisms of G is an abelian group of order (n), where is Eulers
function.
For each integer a dene the map

a
: G G, x
a
(x) = x
a
.
Prove that
a
is an automorphism of G if and only if (a, n) = 1.
Prove that
a
=
b
if and only if a b (mod n).
Prove that every automorphism of G is equal to
a
for some a.
Prove that
a

b
=
ab
.
Deduce that the map
: (Z/nZ)

Aut(G), a (a) =
a
is an isomorphism.
10. Let G be a nite group of order n. Use Lagranges Theorem to show that the map
: G G, g (g) = g
k
is surjective for any integer k relatively prime to n. That is, for such integer k any element
g G has a k
th
root in G.
CHAPTER 4
Quotient Groups and Homomorphisms
1. Cosets and Normal Subgroups
2. More on Lagrange Theorem and Cosets
Proposition 4.1. Let G be a group such that G/Z(G) is cyclic. Show that G is abelian. The
same conclusion does not hold if G/Z(G) is only abelian.
Proof.
Proposition 4.2. Let G be a group of order pq for some primes p and q (not necessarily
distinct). Then
1
either G is abelian or Z(G) = 1.
Proof.
Proposition 4.3. Let G be a group of order pm where p is a prime and p > m. Then G has
2
at most one subgroup, say H, of order p. Such an H is then normal in G.
Proof. Suppose there exist H ,= K distinct subgroups of G of order p. Then [H K[ , = p
and [H K[ divides p by Lagranges Theorem. Hence [H K[ = 1. Therefore
[G[ [HK[ =
[H[ [K[
[H K[
= p
2
,
which is impossible.
This brings up the issue: When do subgroups of a certain order exist? A rst partial converse
to Lagranges Theorem is given by Cauchys Theorem. Thus far we can only prove it for abelian
groups. Later we will prove it in general. Our ultimate goal will be to prove Sylows Theorem.
Theorem 4.4 (Cauchy). Let G be a nite abelian group and p a prime dividing [G[. Then G
contains an element of order p.
Proof. We use induction on [G[. Assume that the result is true for all nite abelian groups of
order < [G[. Since [G[ > 1, there is an element x G, x ,= 1. If [G[ = p, then [x[ = p by Lagranges
Theorem. Assume [G[ > p. Suppose p divides [x[. Let [x[ = n and let n = pt, t an integer. Then
x
t
has order p. Hence assume (p, n) = 1. Let N = x and since G is abelian , N G. By
Lagranges Theorem [G/N[ = [G[/[N[ so that p divides [G/N[. By induction G/N has an element
yN of order p where y G. Put y = yN. Then y , N and y
p
= N or y
p
N. Note y
p
, = y else
y N which is not the case. Hence [y
p
[ < [y[. Notice that by the Lattice Theorem 20 (see 3.3)
p must divide [y, x[. Since G is abelian yx is a subgroup and [xy[ = [x[[y[/[x y[.
But (p, [x[) = 1 so that p must divide [y[/[x y[ and hence p divides [y[ by Lagrange. As
above y has an element of order p.
Actually, for abelian groups the full converse to Lagranges Theorem holds.
Corollary 4.5. Let G be a nite abelian group and n any divisor of [G[. Then G contains a
subgroup of order n.
Proof.
1
As a consequence of the class equation, we will actually see that any group of order p
2
is abelian.
2
Cauchys Theorem will give us the existence of such a subgroup.
11
12 4. QUOTIENT GROUPS AND HOMOMORPHISMS
3. The Isomorphisms Theorems
4. Simple and Solvable Groups
Two classes of groups which arise in the theory of solving polynomial equations with coecients
from a eld are simple and solvable groups.
Composition Series and the Jordan-Holder Theorem.
A group G is simple if the only normal subgroups of G are 1 and G itself.
Proposition 4.6. A nite abelian group G is simple if and only if [G[ = p, a prime.
Proof. Assume that [G[ = p, a prime. Then clearly G is cyclic of order p and hence simple by
Lagrange. Conversely, assume that G is simple. Since G is abelian, each subgroup of G is normal
so that [G[ is a power of some prime p by Cauchys Theorem. Assume that [G[ = p
n
with n 2.
By Cauchys result G has an element x with [x[ = p. But 1 ,= x G so G = x.
Remark 4.7. The rst non-abelian simple group has order 60 and is A
5
, the alternating group
on 5 elements. (This will be explained later see Theorem 5.28).
In a group G a nite chain of subgroups
1 = N
k
N
k1
N
k2
. . . N
1
N
0
= G
is called a normal series provided that N
i
N
i1
(N
i
need not be normal in G) for all 1 i k.
The factor groups N
i1
/N
i
are called the factors of the normal series, and k is called its length. If
each N
i
is a maximal normal proper subgroup of N
i1
, the normal series is called a composition
series and the factors are called composition factors. In other words, a composition series is a
normal series that has no repetitions and that cannot be rened. Thus, a normal series is a
composition series if and only if its factors are simple groups and ,= 1.
Example 4.8.
(1)
(2)
(3)
(4)
A normal subgroup M of G is called a minimal normal subgroup of G provided that M ,= 1
and 1 and M are the only normal subgroups of G that are contained in M.
Let M be a normal subgroup of G. Then G acts on M by conjugation. That is, if n N and
g G, we dene g n = n
g
= gng
1
N. Then M is a minimal normal subgroup of G if and
only if
(1) M ,= 1
(2) The only G-invariant subgroups (under the G-action on M by conjugation) are 1 and M.
In a group G a nite sequence of normal subgroups of G
1 = H
0
H
1
H
2
. . . H
l1
H
l
= G
is called a G-composition series or chief series or principal series provided that for 0 i l 1
the factor H
i
/H
i1
is a minimal normal subgroup of G/H
i1
. The H
i
/H
i1
are called the chief
factors of the given chief series.
Remark 4.9. Let M be a minimal normal subgroup of G. Then M is G-simple under the
G-action of conjugation.
Example 4.10.
(1)
(2)
(3)
(4)
(5)
4. SIMPLE AND SOLVABLE GROUPS 13
We show that any nite group has a composition series; moreover, any two compositions series
have the same length and the same quotient factors (up to order and isomorphism).
Theorem 4.11 (JordanHolder). Let G be a nite group.
(a) G has a composition series.
(b) If G = H
0
H
1
H
k
= 1 and G = N
0
N
1
N
s
= 1 are two
compositions series for G, then k = s and there is a permutation of 1, 2, . . . , k(= s)
such that H
i1
/H
i

= N
(i)1
/N
(i)
.
Proof.
Remark 4.12.
(1)
(2)
(3)
Abelian Series, Solvable Groups and Commutator Subgroups.
A group G is said to be solvable if it has a normal series whose factor groups are abelian. Such a
series is called an abelian (or solvable) series. In particular, we see that G is solvable if and only
if for one (hence every) composition series the quotients are all cyclic groups of prime order.
Example 4.13.
(1)
(2) Any abelian group is solvable. It will be showed later that every group of order < 60 is
solvable. By contrast, a non abelian simple group, such as A
n
for n 5, is not solvable.
Note that S
n
, for n 5, is also not solvable.
Remark 4.14. If G is solvable then G
(1)
_ G (unless G = 1).
Theorem 4.15 (Feit-Thomson). Let G be a nite group of odd order. Then G is solvable
3
.
Hence, it follows that if G is a simple group of odd order, then G

= Z
p
for some prime p.
Another important result is the following.
Theorem 4.16 (P. Hall). A nite group G is solvable if and only if for every divisor n of [G[
such that (n, [G[/n) = 1, G has a subgroup of order n.
Notice that the Halls Theorem is a partial converse of Lagranges Theorem. A beautiful
theorem of Burnside which both motivated Philip Hall
4
and was the rst inductive stage of the
proof of the above theorem states:
Theorem 4.17 (Burnside). Let G be a nite group of order p
n
q
m
where p, q are distinct primes
and m, n Z
+
. Then G is solvable.
Let G be a group and let x, y G. The element [x, y] = x
1
y
1
xy is called the commutator of
x and y. Observe that xy = yx[x, y], hence one easily obtains that [x, y] = 1 if and only if xy = yx.
Moreover, we also have that [x, y]
1
= (x
1
y
1
xy)
1
= y
1
x
1
yx = [y, x]. Dene G
t
to be the
subgroup generated by commutators of elements of G. By the previous observations, G
t
consists
of nite products of commutators of elements of G
G
t
= [x, y] [ x, y G =
_
t

i=1
[x
i
, y
i
] [ x
i
, y
i
G, t Z
+
0
_
G
t
is called the commutator subgroup of G. Also, for any x, y, g G we have that
g[x, y]g
1
= gx
1
y
1
xyg
1
= (gx
1
g
1
)(gy
1
g
1
)(gxg
1
)(gyg
1
) = [gxg
1
, gyg
1
]
so that it easily follows that G
t
G. The meaning of G
t
is that it measures the failure of G from
being abelian: in fact G is abelian if and only if G
t
= 1.
3
The proof of the Feit-Thomson Theorem occupies a whole issue of the Pacic J. Math. This theorem has played
a very important part in the development of group theory, because it shows that every non abelian nite simple
group contains an element of order 2. It was a starting point in the program that eventually led to the classication
of all nite simple groups.
4
Note that Sylows Theorem is also a major part of motivation of the Hall result.
14 4. QUOTIENT GROUPS AND HOMOMORPHISMS
Theorem 4.18. Let G be a group and let G
t
be the commutator subgroup of G. Then
(a) The quotient group G/G
t
is abelian.
(b) If N G and G/N is abelian, then G
t
N.
(c) If N G and G
t
N, then N G and G/N is abelian.
In particular, G/G
t
is the largest abelian factor group of G.
Proof. Let g, h G. Then (gG
t
)(hG
t
) = ghG
t
= gh[h, g]G
t
= hgG
t
= (hG
t
)(gG
t
) so that
G/G
t
is abelian and (a) is established. Suppose now that N G and assume that G/N is abelian.
For all g, h G we have that (gN)(hN) = (hN)(gN) or equivalently ghN = hgN. This means
that for all g, h G we have [g, h] = h
1
g
1
gh N. Hence G
t
N, which yields (b). Finally, let
g G and n N. Observe that n[n, g
1
] N as G
t
N. But n[n, g
1
] = gng
1
and since g and
n were chosen to be arbitrary we conclude that N G, as desired. Pick now g, h G and observe
that g
1
h
1
gh G
t
N. This means that ghN = hgN or, equivalently, (gN)(hN) = (hN)(gN).
Thus G/N is abelian and this completes the proof of (c).
Theorem 4.19. Let G be a group and let M, N be subgroups of G. Assume also that N G.
(a) If G is solvable then any subgroup of G is solvable.
(b) G is solvable if and only if both N and G/N are.
(c) If both M and N are solvable then MN is also solvable.
Proof.
Let G be a group and G
(1)
= G
t
be the commutator subgroup of G. Then G
(2)
= (G
(1)
)
t
and
in general G
(i)
= (G
(i1)
)
t
. G
(i)
is called the i-th commutator subgroup of G. Observe that if is
an endomorphism of G, then ([x, y]) = [(x), (y)] for all x, y in G. In particular, this gives a
shorter proof that G
t
G. Let g G be xed and f
g
the inner-automorphism of G induced by g.
Then f
g
restricted to G
t
is an automorphism. Hence G
(2)
is invariant under f
g
so that G
(2)
G.
Assume that G
(i)
G. Then the same reasoning as above shows that G
(i+1)
is normal in G.
Theorem 4.20. Let G be a group. Then G is solvable if and only if there is an integer s such
that G
(s+1)
= 1.
Proof. Assume

Example 4.21.
(1)
(2)
(3)
Problem 4.22.
Theorem 4.23. Let G be a nite solvable group and M a minimal normal subgroup of G.
Then M is an elementary abelian p-group for some prime p.
Proof.
Corollary 4.24. Let G be a nite group. Then G is solvable if and only if it has a chief
series whose factors are elementary abelian groups.
5. The Holder Program (Holder 18591937)
(1) Classify all nite simple groups.
(2) Find all ways of putting simple groups together to form other groups.
These two problems have been the underlying motivation for much research in nite groups over
the past one hundred years. Eorts by over 100 mathematicians covering between 5,000-10,000
journal pages in between 300-500 research papers established the following:
5. THE H

OLDER PROGRAM (H

OLDER 18591937) 15
Theorem 4.25. There is a list consisting of 18 innite families of simple groups and 26 simple
groups not belonging to these families (the so called 26 sporadic simple groups) such that every nite
simple group is isomorphic to one of the groups in the list.
One such family is Z
p
[ p a prime. A second innite family in the list of nite simple groups
is:
SL
n
(F)/Z(SL
n
(F)) [ n Z
+
, n 2 and F a nite eld.
These groups are simple except for SL
2
(F
2
) and SL
2
(F
3
), where F
2

= Z
2
and F
3

= Z
3
. (Here we
are considering Z
2
and Z
3
as elds).
The FeitThompson shows that a nite non-abelian simple group has even order. Let G be a
nite (non-abelian) simple group. Then the Sylow 2-structure of G is an important consideration
in the structure of G.
Part (2) of the Holder Program is sometimes called the extension problem. A more precise
description of putting two groups together is: given two groups A and B, describe how to obtain
all groups G containing a normal subgroup N such that N

= B and G/N

= A. This is very
dicult even when [A[ and [B[ are small. See the second paragraph on p. 106 of text.
The study of nite groups is much broader than the study of nite (non-abelian) simple groups.
For example, the study of solvable groups and other classes of interest.
16 4. QUOTIENT GROUPS AND HOMOMORPHISMS
Homework set # 4.
The due date is on October 19 (Friday), 2001.
1. Prove that if H and K are nite subgroups of G whose orders are relatively prime then
H K = 1.
2. Let H and K be normal subgroups of a group G with H K = 1. Show that
hk = kh for every h H and k K.
HK is a subgroup of G with HK

= H K.
Give an example of a group G with two such subgroups H and K.
3. Let H and K be normal subgroups of G such that G = HK. Prove that G/(H K)

=
(G/H) (G/K). In particular, if H K = 1 one has that G

= (G/H) (G/K).
4. Use Lagranges Theorem in the multiplicative group (Z/pZ)

to prove Fermats Little


Theorem:
If p is a prime then a
p
a mod p for all a Z.
5. Let H G and let g G. Prove that if the right coset Hg equals some left coset of H
in G then it equals the left coset gH and g must be in N
G
(H).
6. Let G be a group. For any g G dene the map
f
g
: G G, x f
g
(x) = gxg
1
,
called inner automorphism (or conjugation).
Verify that f
g
is an automorphism.
Show that the map g f
g
is a homomorphism of G into Aut(G) with kernel Z(G).
Conclude that Inn(G) = f
g
[ g G is a subgroup of Aut(G) with Inn(G)

=
G/Z(G).
Verify that Inn(G) is a normal subgroup of Aut(G).
The quotient group Aut(G)/Inn(G) is called the group of outer automorphisms.
7. Prove that if G is an abelian group of order pq, where p and q are distinct primes, then
G is cyclic.
8. Let N G and M G be subgroups of a group G. If both M and N are solvable then
so is MN.
9. Prove the Jordan-Holder Theorem.
(Hint: use induction and the second Isomorphism Theorem.)
10. Let H be a subgroup of a group G of nite index. Show that there exists a normal
subgroup N of G of nite index with N H.
CHAPTER 5
Group actions
1. Group actions and permutation representation
Let G be a group and S a set. A (left) action of G on S is a map
: GS S (g, s) g s
such that 1 s = s for all s S and g
1
(g
2
s) = (g
1
g
2
) s for all g
1
, g
2
G and all s S.
Remark 5.1. We will always consider left group actions. But one can also consider right group
actions. Given a left action g s of a group G on a set S, one can dene (check!) a right action in
the following way
s g
def
= g
1
s.
Proposition 5.2. Dening an action of G on S is equivalent to dening a homomorphism of
groups : G Sym(S).
Proof. Given an operation GS S, dene : G Sym(S) by setting (g)(s) = g s
for all g G and all s S. We need to show that is well dened, that is (g) Sym(S). But
(g
1
) (g) = (g
1
g) = (1) = id
S
, and likewise (g) (g
1
) = id
S
. Thus (g) is a bijection
of S into S. Also, we have that (g
1
g
2
) = (g
1
) (g
2
) which shows that is a homomorphism.
Conversely, given a homomorphism : G Sym(S) we can dene an action of G on S by setting
g s = (g)(s). Observe that 1 s = (1)(s) = id
S
(s) = s, since (1) = id
S
. Moreover, since
is a homomorphism one has (g
1
g
2
) s = (g
1
g
2
)(s) = ((g
1
) (g
2
))(s) = (g
1
)((g
2
)(s)) =
g
1
(g
2
s).
Such an homomorphism is called the permutation representation of G determined by the
action of G on S. G is called a permutation group on a set S if it has a permutation representation
on S.
Proposition 5.3. Let G be a group acting on a set S.
(a) For x, y S set x y to be dened by y = g x for some g G. This is an equivalence
relation, whose equivalence classes, also called orbits, are x = g x[ g G.
Thus S =
_
i
x
i
.
(b) The set G
s
= g G[ g s = s is a subgroup of G. It is called the subgroup xing s, or
the isotropy group of s, or the stabilizer of s.
(c) Ker() =

sS
G
s
, where is the permutation representation aorded by the group action.
Proof. Observe that x x, as x = 1 x for all x S. Also, x y means that y = g x for
some g G. However g
1
y = g
1
(g x) = (g
1
g) x = 1 x = x, or equivalently y x. Finally,
x y and y z mean that y = g x and z = g
t
y for some g, g
t
G. Hence z = g
t
(g x) = (g
t
g) x,
which means that x z. Thus the relation is reexive, symmetric and transitive and (a) is easily
established. G
s
is clearly non empty as 1 G
s
. Moreover, if g
1
, g
2
G
s
we have that g
1
g
1
2
G
s
.
Indeed, g
2
s = s implies that s = 1 s = (g
1
2
g
2
) s = g
1
2
(g s) = g
1
2
s or g
1
2
G
s
. Moreover,
(g
1
g
1
2
) s = g
1
(g
1
2
s) = s. Thus (b) is established because of the Subgroup Criterion. Finally,
Ker() = g G[ (g) = id
S
= g G[ g s = s for all s S =

sS
G
s
.
The action is called faithful if is an embedding, i.e. Ker() = 1. The action is called
transitive if there is only one orbit.
17
18 5. GROUP ACTIONS
Proposition 5.4. Let G be a group acting on a set
(a) If x = y then G
x
and G
y
are conjugate subgroups, i.e. G
y
= gG
x
g
1
if y = g x.
(b) If the action is transitive, i.e. S = x for some x S, then Ker() =

gG
gG
x
g
1
, where
is the permutation representation aorded by the action of G on S.
(c) [x[ = [G: G
x
].
(d) (Orbit Decomposition Formula) Write S =
_
i
x
i
, then [S[ =

i
[G: G
x
i
].
Proof. If x = y we have that y = g x for some g G. We noted earlier that we also have x =
g
1
y. Now, pick h G
x
; thus hx = x and we have that y = g(hx) = g(h(g
1
y) = (ghg
1
)y.
Hence gG
x
g
1
G
y
. Conversely, if k G
y
we have that k y = y. This yields that k (g x) = g x
or, equivalently, (g
1
kg) x = x. Thus g
1
kg G
x
and also k = g(g
1
kg)g
1
gG
x
g
1
as desired.
This establishes (a). The proof of (b) follows by combining the previous proposition and part (a)
that we just proved. To prove part (c), dene a function
f : x = g x[ g G gG
x
[ g G,
by f(g x) = gG
x
. Now, g
1
x = g
2
x is equivalent to (g
1
2
g
1
) x = x. Thus g
1
2
g
1
G
x
, and this
is equivalent to g
1
G
x
= g
2
G
x
. This shows that f is well dened and injective. Since f is clearly
surjective, f is a bijection. The proof of (d) is now clear.
Example 5.5 (Trivial action). Dene an action of G on S by setting g s = s for all g G
and all s S. Observe that the corresponding homomorphism aorded by this action is given by
(g) = id
S
.
Example 5.6 (Evaluation action). Let S be a set and let G = Sym(S). Then Sym(S) acts
on S in the following way s = (s) for all Sym(S) and all s S. Observe that the
corresponding homomorphism aorted by this action is = id
Sym(S)
. This action is faithful and
transitive. In particular, observe that G = S
n
acts on S = 1, 2, . . . , n. Also, observe that the
stabilizer G
i

= S
n1
for all i 1, 2, . . . , n. Hence n = [S[ = [S
n
: G
i
] = [S
n
[/[S
n1
[, which gives
that [S
n
[ = n!.
Groups acting on themselves by conjugation and the class equation.
Theorem 5.7. Let G be a nite group.
(a) The number of elements in the conjugacy class of x G is [G: C
G
(x)], which divides [G[.
(b) If x
1
, . . . , x
n
are the distinct conjugacy classes of G then [G[ =
n

i=1
[G: C
G
(x
i
)].
(c) Let H be a subgroup of G. Then the number of subgroups of G conjugate to H is
[G: N
G
(H)], which divides [G[.
Proof.
Also, observe that if H is a normal subgroup of G and x is a conjugacy class of G, then either
x N or x N = . Thus a normal subgroup N of G is the union of the conjugacy classes
contained in N.
Theorem 5.8 (Class Equation). Let G be a nite group, then
[G[ = [Z(G)[ +
m

i=1
[G: C
G
(x
i
)],
where x
1
, . . . , x
m
are the distinct conjugacy classes with [G: C
G
(x
i
)] > 1.
Proof.
Example 5.9.
Let G = D
8
= r, s [ r
4
= s
2
= 1, srs = r
1
. The conjugacy classes of G are 1, r
2
,
r, r
3
, s, sr
2
, sr, sr
3
. Recall that Z(G) = 1, r
2
.
1. GROUP ACTIONS AND PERMUTATION REPRESENTATION 19
Let G = Q
8
= x, y [ x
4
= y
4
= 1, x
2
= y
2
, yxy
1
= x
1
. The conjugacy classes of G
are 1, x
2
, x, yxy
1
, xy, (xy)
1
, y, y
1
. Recall that Z(G) = 1, x
2
.
Corollary 5.10 (Cauchys Theorem). Let G be a nite group and let p be a prime dividing
the order of G. Then there exists an element g G with [g[ = p.
Proof. We induct on the order of G. If [G[ = p we are done. We deal with the inductive step.
If there exists a proper subgroup whose order is divisible by p, then we are done by induction.
Thus we may assume that for all subgroups H of G then p does not divide [H[. But then for all
proper subgroups H of G we have that p divides [G: H]. The class equation says that
[G[ = [Z(G)[ +
m

i=1
[G: C
G
(x
i
)]
with [G: C
G
(x
i
)] > 1. Hence p divides [G: C
G
(x
i
)] and so we conclude that p divides [Z(G)[.
Hence G = Z(G), that is G is abelian. In this case we have already proved Cauchys Theorem in
an earlier section.
A group in which every element has order a power ( 0) of some xed prime p is called a
p-group. If H is a subgroup of a group G and H is a p-group, H is said to be a p-subgroup of G.
Corollary 5.11. A nite group G is a p-group if and only if [G[ is a power of p.
Proof. If G is a p-group and q is a prime which divides [G[, then G contains an element of
order q by Cauchys Theorem. Since every element of G has order a power of p, then q = p. Hence
[G[ is a power of p. The converse is a consequence of Lagranges Theorem.
Corollary 5.12. Let G be a nite p-group, with p a prime. Then Z(G) ,= 1.
Proof. In the class equation, we have that 1 < [G: C
G
(x
i
)] divides [G[ = p
k
, where k > 0.
Hence p divides [G: C
G
(x
i
)] for 1 i m. Therefore we have that p divides [Z(G)[. Thus
Z(G) ,= 1.
The above theorem is false for innite p-groups, p a prime.
Corollary 5.13. A nite p-group G is solvable.
Proof. We use induction on the order of the group G. According to Corollary 5.12, the center
Z(G) is nontrivial, and so the induction hypothesis shows that G/Z(G) is solvable. Because Z(G)
is solvable, Theorem 4.19 shows that G is solvable.
Corollary 5.14. Let G be a group with order p
2
, with p a prime. Then G is abelian. More
precisely, G is isomorphic to either Z
p
2 or Z
p
Z
p
.
Proof.
Groups acting on themselves by translation and Cayleys Theorem. Let H be a
subgroup of the group G. Consider the set S consisting of all the left cosets of H, i.e. S =
xH[ x G. Then G acts on S by left translation, that is g xH = gxH. It is easy to check
that this is indeed an action of G on S. The homomorphism aorded by this action is dened
by (g) = t
g
, where t
g
(xH) = gxH for all g, x G. This action is clearly transitive
1
as H = S.
Moreover the stabilizer of H is G
H
= g G[gH = H = H. In conclusion the kernel Ker() of
the permutation representation of G aorded by this action is
Ker() =

xHS
G
xH
=

xG
xG
H
x
1
=

xG
xHx
1
= core
H
(G) = H
G
H.
Also, by the First Isomorphism Theorem, G/Ker() Sym(S). In particular we have just shown
that core(H) = Ker() is a normal subgroup of G contained in H. Actually, it is the largest such
subgroup: indeed, if K G with K H then
K =

xG
xKx
1

xG
xHx
1
= Ker().
1
Alternatively, for any x, y G one has g xH = yH with g = yx
1
.
20 5. GROUP ACTIONS
In conclusion, we have proved the following theorem.
Theorem 5.15. Let H be a subgroup of G, N =

xG
xHx
1
and S = xH[ x G. Then
N is the largest normal subgroup of G contained in H, and G/N is isomorphic to a subgroup of
Sym(S).
If we let H = 1, the corresponding action is called the left regular action of G onto itself. In
this case the previous theorem yield:
Corollary 5.16 (Cayleys Theorem). Every group G is isomorphic to a subgroup of a group
of permutations. In particular, if [G[ = n then G is isomorphic to a subgroup of S
n
.
Corollary 5.17. Let G be a nite group and let H be a subgroup of G of index k. If 1
is the only normal subgroup of G contained in H then G is isomorphic to a subgroup of S
k
. In
particular [G[ divides k!.
Corollary 5.18. Let G be a nite group and let p be the smallest prime dividing [G[. Then
every subgroup of index p is normal in G.
Proof. Let H be a subgroup of G with [G: H] = p and let N be as in Theorem 5.15.
Suppose [H: N] = q and observe that by the index formula one has pq = [G: H][H: N] = [G: N].
In particular, every divisor of q is larger than p. On the other hand, by Theorem 5.15 we have that
[G: N] = [G/N[ divides [S
p
[ = p! and hence q divides (p 1)!. This is impossible unless q = 1.
Thus H = N is a normal subgroup of G.
Corollary 5.19. Let H be a subgroup of a nite group G with [G: H] = 2. Then H G.
Example 5.20.
The above corollary is not an existence result, as A
4
has no subgroups of index 2.
Let G = A
4
and let H = (1 2 3). Note that [G: H] = 2
2
. Also, H is not normal in G.
Let H = D
8
be a subgroup of S
4
. Then [S
4
: H] = 3 but H , S
4
.
Let H = K
4
be a subgroup of G = A
4
. Then [G: H] = 3 and H G.
Corollary 5.21. Let G be a non-abelian group of order 6. Then G

= S
3
.
Proof. There exists g G with [g[ = 2 (this follows from Cauchys Theorem). Let H = 1, g.
Suppose HG, then for all x G one has xhx
1
H = 1, g. Hence xgx
1
= g, or equivalently
xg = gx. Thus H Z(G) and therefore [G/Z(G)[ 3. But this is impossible as otherwise G is
abelian. Thus H is not normal in G, hence does not contain any normal subgroup of G other than
1. Thus, since [G: H] = 3, by Theorem 5.15 one has that G S
3
. Since they both have order
6 we conclude that G

= S
3
.
2. Transpositions, Alternating groups, Conjugacy in S
n
and the Simplicity of A
n
Proposition 5.22. Let , be elements of S
n
and assume that
= (a
1
a
2
. . . a
k
1
)(b
1
b
2
. . . b
k
2
)
is the cycle decomposition of . Then the cycle decomposition of
1
is

1
= ((a
1
) (a
2
) . . . (a
k
1
))((b
1
) (b
2
) . . . (b
k
2
))
That is,
1
is obtained from by replacing each entry i in the cycle decomposition for by
the entry (i).
Proof. Let i, j 1, 2, . . . , n and let (i) = j. Then
1
((i)) = (j). Thus, if the
ordered pair i, j appears in the cycle decomposition of , then the ordered pair (i), (j) appears
in the cycle decomposition of
1
.
Example 5.23. Let = (1 2)(3 4 5)(6 7 8 9) and = (1 3 5 7)(2 4 6 8) be in S
9
. Then

1
= (3 4)(5 6 7)(1 2 9 8).
2. TRANSPOSITIONS, ALTERNATING GROUPS, CONJUGACY IN S
n
AND THE SIMPLICITY OF A
n
21
Let S
n
be the product of disjoint cycles of lengths n
1
, n
2
, . . . , n
r
where n
1
n
2
. . . n
r
(including its 1-cycles). Then the integers n
1
, n
2
, . . . , n
r
are called the cycle type of .
Let n Z
+
. A partition of n is any nondecreasing sequence of positive integers with sum = n.
Let S
n
and let n
1
, n
2
, . . . , n
r
be the cycle type of . Then n
1
+n
2
+. . . +n
r
= n and
thus the positive integers n
1
, n
2
, . . . , n
r
is a partition of n.
The cycle type of S
n
is unique. For example, an m-cycle in S
n
has cycle type
1, 1, . . . , 1, m, where m is preceded by n m ones.
Proposition 5.24.
(1) Two elements of S
n
are conjugate if and only if they have the same cycle type.
(2) The number of conjugacy classes of S
n
equals the number of partitions of n.
Proof. By the previous proposition, conjugate permutations have the same cycle type. Con-
versely, assume that
1
and
2
are permutations of the same cycle type. Order the cycle in
nondecreasing length, including the 1-cycles (if several cycles of
1
and
2
have the same length
then there will be several ways of doing this). Ignoring parentheses, each cycle decomposition is a
list in which the integers from 1 to n appear exactly once. Dene to be the permutation which
maps the i-th integer in the list for
1
to the i-th integer in the list for
2
. The previous proposition
assures us that
1

1
=
2
. This proves (1). There is a bijection between the conjugacy classes
of S
n
and the permissible cycle types for the permutations in S
n
. The latter ones correspond to
the partitions of n. Hence (2) follows.
Example 5.25.
Let
1
= (1)(3 5)(8 9)(2 4 7 6) and
2
= (3)(4 7)(8 1)(5 2 6 9). Then
1

1
=
2
, where
= (1 3 4 2 5 7 6 9)(8). However, we also have
1

1
=
2
, where = (1 3 8 4 2 5)(6 9 7).
In fact we can rewrite
2
= (3)(8 1)(4 7)(5 2 6 9).
Note that the two cycles
1
= (1 2 3 4 5) and
2
= (1 3 5 2 4) of A
5
are conjugates in
S
5
, but not in A
5
. Indeed, = (2 3 5 4).
Consider the case n = 5. Then we have
Partition of 5 Representative of Conjugacy Class
1, 1, 1, 1, 1 1
1, 1, 1, 2 (1 2)
1, 1, 3 (1 2 3)
1, 4 (1 2 3 4)
1, 2, 2 (1 2)(3 4)
2, 3 (1 2)(3 4 5)
5 (1 2 3 4 5)
Proposition 5.26. Let be an m-cycle in S
n
. Then [C
S
n
()[ = m(n m)!.
More precisely one has C
S
n
() =
i
[ 0 i m1, S
nm
.
Proof. Let = (a
1
a
2
. . . a
m
) be an m-cycle. The number of conjugates of , i.e. the
number of m-cycles, is
n(n 1) (n m+ 1)
m
= [[ = [S
n
: C
S
n
()] =
[S
n
[
[C
S
n
()[
.
So that [C
S
n
()[ = m(n m)! Now observe that the elements 1, ,
2
, . . . ,
m1
clearly commute
with . Also, commutes with any permutation in S
n
whose cycle are disjoint to and there
are (nm)! permutations of this type (the full symmetric group on the numbers not appearing in
). The product of the elements of these two types already accounts for the m(n m)! elements
commuting with .
We recall that a group G is called simple if 1 and G are the only normal subgroups of G.
Notice that 1 and groups of prime order are simple. In the abelian case these are the only simple
groups.
Lemma 5.27.
(a) For n 3, A
n
is generated by the 3-cycles.
22 5. GROUP ACTIONS
(b) For n 5, any two 3-cycles are conjugate in A
n
.
Proof. (a) Clearly every 3-cycle is in A
n
. To show the converse, we may assume n 4 since
the case n = 3 is clear. Let a, b, c, d be distinct elements in 1, 2, . . . , n. Notice that:
(a b)(a b) = (a b c)
3
(a b)(b c) = (a b c)
(a b)(c d) = (a b)(b c)(b c)(c d) = (a b c)(b c d).
(b) Let = (a b c) and be two 3-cycles. We already know that they are conjugates in S
n
. Thus
there exists S
n
with =
1
. If A
n
we are done. Otherwise, let a, b, c, d, e be distinct
elements in 1, 2, . . . , n. (Recall that n 5.) Let = (d e). Now A
n
, and since = (a b c)
and (d e) are disjoint we have

1
= (d e)(d e)
1
= (d e)(d e)
1
=
1
= ,
as we claimed.
Theorem 5.28. For n 5, A
n
is simple.
Proof. Let 1 , = H A
n
. It suces to show that, in the light of Lemma 5.27, that H
contains a 3-cycle. This will force H = A
n
. Here is the strategy of the proof. Let 1 ,= H.
Let be any 3-cycle. Then x = (
1

1
) H, since H and H A
n
. On the other hand,
x = (
1
)
1
is a product of two 3-cycles. By choosing suitably, will yield a 3-cycle.
Let a, b, c, d, e denote distinct elements in 1, 2, . . . , n. Then the canonical cycle decomposition
of is: (1) = (a b c d . . .) , i.e. has a cycle of length at least 4. In this case choose
= (a b c). Then x = (
1
)
1
= (b c d)(c b a) = (a d b). (2) = (a b c)(d e . . .) , i.e.
has a cycle of length 3. In this case choose = (a b d). Then x = (
1
)
1
= (b c e)(d b a) =
(a d c e b). Now use the previous case. (3) = (a b)(c d) , i.e. has only cycles of length
2. In this case choose = (a c e). Then x = (
1
)
1
= (b d (e))(e c a) = (b d (e) . . .) .
Now use the previous two cases.
3. Automorphisms
Let G be a group. An isomorphism from G onto itself is called an automorphism of G. Denote
by Aut(G) the set of all automorphisms of G. Recall that Aut(G) S
G
. That is, Aut(G) is a
group under composition.
Proposition 5.29. Let H G. For each g G, dene
g
: H H by h ghg
1
. Then
(1)
g
Aut(H) for each g G.
(2) The mapping : G Aut(H) dened by g
g
is a homomorphism with kernel
C
G
(H).
(3) G/C
G
(H)

=
g
[ g G Aut(H).
(4) G acts by conjugation on H and the mapping : G Aut(H) S
H
dened by g
g
is a permutation representation of G.
Proof. Let g G be xed. Then
g
(h) = ghg
1
H since H G. Consider h
1
, h
2
H.
Then
g
(h
1
h
2
) = g(h
1
h
2
)g
1
= (gh
1
g
1
)(gh
2
g
1
) =
g
(h
1
)
g
(h
2
). So that
g
is a homomorphism
of H into H. Assume that
g
(x) =
g
(y). Then gxg
1
= gyg
1
so x = y. Also
g
(g
1
xg) = x
so that
g
Aut(H). Hence (1) holds. Let g
1
, g
2
G. Then (
g
1

g
2
)(h) =
g
1
(
g
2
(h)) =

g
1
(g
2
hg
1
2
) = (g
1
g
2
)h(g
1
g
2
)
1
=
g
1
g
2
(h) for all h H. Hence the mapping : G Aut(H)
dened by g
g
is a homomorphism. Observe that Ker() = g G[
g
= id
H
= g
G[
g
(h) = ghg
1
for all h H = g G[ gh = hg for all h H = C
G
(H). Thus (2) holds.
Also (3) follows from the First Isomorphism Theorem. Finally, (4) follows from (1)-(3).
Corollary 5.30. Let K G, G a group and let g G. Then K

= gKg
1
.
Corollary 5.31. Let H G, G a group. Then
(1) N
G
(H)/C
G
(H) is isomorphic to a subgroup of Aut(H).
(2) G/Z(G)

= Inn(G) =
g
[ g G, the group of inner automorphisms of G, is isomorphic
to a subgroup of Aut(G).
3. AUTOMORPHISMS 23
Proof. HN
G
(H) and C
G
(H) N
G
(H). By Proposition 5.29 with N
G
(H) replacing G, (1)
follows. In Proposition 5.29 put G = H and note that Z(G) = C
G
(H). Hence (2) follows.
A subgroup H of a group G is called characteristic, denoted H char G, if every automorphism
of G maps H into itself. That is (H) = H for all Aut(G).
(1) Characteristic subgroups are normal.
(2) If H is the unique subgroup of a given order in a group G, then H char G.
(3) If H char K and K char G then H char G.
(4) If H char K and K G then
2
H G.
(5) We already gave examples in which H K and K G, but H need not be normal in G.
Example 5.32.
(i) Z(Q
8
) = x
2
, where Q
8
= x, y [ x
4
= y
4
= 1, x
2
= y
2
, yxy
1
= x
1
. Inn(Q
8
)

=
Q
8
/Z(Q
8
)

= Z
2
Z
2
.
(ii) For n 3, Z(S
n
) = 1. So Inn(S
n
)

= S
n
.
(iii) Let G = A
4
, K = 1, (1 2)(3 4), (1 3)(2 4), (1 4)(2 3) G. H = (1 2)(3 4) K but
H ; A
4
.
(iv) Let G = A
4
, K = 1, (1 2)(3 4), (1 3)(2 4), (1 4)(2 3) G. Let g = (1 2 3). Then

g
Aut(K) but
g
is not an inner-automorphism of K.
Proposition 5.33.
(1) If G

= Z (under addition), then Aut(G)

= Z
2
(under addition).
(2) Let G = x be a cyclic group of order n. Then Aut(G)

= (Z
n
)

and [Aut(G)[ = (n),


where is the Euler -function.
Example 5.34. Let G be a group of order pq, p and q are primes with p q. If p [ (q 1)
then G is abelian, hence cyclic.
Proof. Suppose Z(G) ,= 1. Then G/Z(G) is cyclic so that G is abelian. Hence assume that
Z(G) = 1. By Cauchys Theorem (see Corollary 5.10), G has an element of order q, say x. Let
H = x and note that [G: H] = p so that by Corollary 5.18 one has that H G. Since Z(G) = 1
then C
G
(H) = H, for C
G
(H) = G implies H Z(G). Thus G/H = N
G
(H)/C
G
(H) is a group of
order p and isomorphic to a subgroup of Aut(H), which is isomorphic to Z

q
. But p [ (q 1), a
contradiction.
The next result provides a summary of automorphism groups.
Proposition 5.35.
(1) Let p be an odd prime and let n Z
+
. Then Aut(Z
p
)

= Z
p1
, the cyclic group of order
p 1. More generally, the automorphism group of a cyclic group of order p
n
is cyclic of
order p
n1
(p 1), that is Z

p
n.
(2) For n 3, the automorphism group of the cyclic group of order 2
n
is isomorphic to
Z

2
n

= Z
2
Z
2
n2 and in particular is not cyclic but has a cyclic subgroup of index 2.
(3) Let p be a prime and V an elementary abelian group of order p
n
written additively (that
is, V = x
1
x
2
x
n
, where [x
i
[ = p). Then V is an n-dimensional vector
space over Z
p
and Aut(V )

= GL
n
(Z
p
)

= GL(V ).
(4) For n ,= 6, Aut(S
n
) = Inn(S
n
)

= S
n
. For n = 6, Aut(S
n
)/Inn(S
n
)

= Z
2
.
(5) Aut(D
8
)

= D
8
and Aut(Q
8
)

= S
4
.
(6) Let K be the Klein 4-group then Aut(K)

= S
3
.
(7) Let G be a group of order p
2
. Then G

= Z
p
Z
p
or Z
p
2. Thus Aut(G) is GL
2
(Z
p
) or
Z

p
2
. Note that [GL
2
(Z
p
)[ = p(p 1)
2
(p + 1) and [Z

p
2
[ = (p
2
) = p(p 1).
Example 5.36. Let G be a group of order 45 = 3
2
5. Assume
3
that G has a normal subgroup
P of order 9 (that is, a 3-Sylow subgroup ). Then G is abelian.
2
Let g G and dene
g
to be conjugation in G by g. Clearly,
g
Aut(G) and since KG we also have that

g
(K) = K. Thus
g
Aut(K). As H char K we also have that gHg
1
=
g
(H) = H. Hence H G.
3
Actually, as a consequence of Sylows Theorem see the next section this is always the case.
24 5. GROUP ACTIONS
Proof. Suppose [P[ = 3
2
. By Proposition 5.29, G/C
G
(P) is isomorphic to a subgroup of
Aut(P) which has order 3(3 1)
2
(3 + 1) = 48 or 3 2 = 6. Note that P C
G
(P), as P is an
abelian group, so that [G: C
G
(P)] = 1 or 5. But (5, 48) = (5, 6) = 1 so that C
G
(P) = G. This
means that P Z(G) and G/Z(G) is cyclic. Thus G is abelian.
4. Sylows Theorem
Let p be a prime. If G is a group of order p
n
m with (p, m) = 1, then a subgroup of order p
n
is called a p-Sylow subgroup of G. Syl
p
(G) denotes the set of p-Sylow subgroups of G and n
p
(G)
denotes the number of such subgroups.
Let G be a group acting on a set S. A xed point x of G in S is an element x S such that
g x = x for all g G. Set S
0
= xed points of G in S.
Lemma 5.37. Let H be a nite p-group acting on a nite set S. Then [S[ [S
0
[ mod p.
Proof. Notice that S
0
=

_
[x
j
[=1
x
j
. Thus [S[ = [S
0
[ +

[x
j
[>1
[x
j
[. But by Proposition 5.4(c)
[x
i
[ = [H: H
x
i
]. But H is a p-group and [H: H
x
i
] > 1, hence p divides [H: H
x
i
], hence p divides
[x
i
[. Hence [S[ [S
0
[ mod p.
Theorem 5.38 (Sylows Theorem). Let G be a nite group of order p
n
m with p [ m.
I. G has
4
a subgroup of order p
n
, that is Syl
p
(G) ,= .
II. Let H be a p-subgroup of G and let P be a p-Sylow subgroup of G. Then there exists
g G such that H gPg
1
. In particular, any two p-Sylow subgroups are conjugate.
III. The number n
p
(G) of p-Sylow subgroups divides m and n
p
(G) 1 mod p. Moreover,
n
p
(G) = [G: N
G
(P)] where P Syl
p
(G).
Proof. We induct on the order of the group. If [G[ = 1 we are done. Let us prove the
inductive step. We may assume that n 1. If H is a proper subgroup of G and p
n
divides [H[,
then we are done by induction hypothesis. Thus we may assume that p
n
[ [H[ for any proper
subgroup H of G. Thus p divides [G: H] for all subgroups of G with [G: H] > 1. Now, the class
equation says that
[G[ = [Z(G)[ +
m

i=1
[G: C
G
(x
i
)]
with [G: C
G
(x
i
)] > 1. As p divides both [G: C
G
(x
i
)] and [G[, then p divides also [Z(G)[. By
Cauchys Theorem, Z(G) has a subgroup H of order p. Since H Z(G) we have that H G.
Form G/H. Now p
n1
divides [G/H[. By induction hypothesis there exists a subgroup K of G/H
of order p
n1
. Write K = P/H with H P G. Then [P[ = p
n
. This completes the proof of
part I.
Let S = gP [ g G and let H act on S by translation. Now [S[ = [G: P] = m, thus p [ m,
thus by Lemma 5.37 we have that [S
0
[ , = 0. So there exists a xed point gP of H in S. Hence for
all h H we have hgP = gP, hence g
1
hgP = P. Thus g
1
hg P or, equivalently, h gPg
1
for all h H. Hence H gPg
1
. In particular, let Q be another p-Sylow subgroup of G. By
the previous fact, one has Q gPg
1
for some g G. But [gPg
1
[ = [P[ = p
n
= [Q[. Thus
Q = gPg
1
and part II. is established.
Let S = Syl
p
(G) be the set of all p-Sylow subgroups of G. By part I. we have that S ,= .
Let P S be any p-Sylow subgroup. By part II. we have that S = gPg
1
[ g G and thus
n
p
(G) = [S[ = [G: N
G
(P)], where [G: N
G
(P)] divides [G: P] = m. Now, P acts by conjugation.
Notice that P S
0
(the xed point set under this action). Conversely, if Q S
0
then gQg
1
= Q
for all g P, hence P N
G
(Q). Now P and Q are both p-Sylow subgroups of N
G
(Q). But Q is
normal in N
G
(Q), thus by part II. we have that P = Q. This shows that S
0
= P and [S
0
[ = 1.
Now, by Lemma 5.37 n
p
(G) = [S[ 1 mod p, since P is a p-group.
4
One can actually show a stronger version: G contains a subgroup of order p
i
for each i n and every subgroup
of G of order p
i
(with i < n) is normal in some subgroup of order p
i+1
.
4. SYLOWS THEOREM 25
Corollary 5.39. Let G be a nite group and P a p-Sylow subgroup of G, p a prime. Then
the following statements are equivalent:
(a) P is the unique p-Sylow subgroup of G; i.e. n
p
(G) = 1.
(b) P is normal in G.
(c) P is characteristic in G.
Proof. Suppose that (a) holds. Let g G. Then gPg
1
is a p-Sylow subgroup of G so that
gPg
1
= P and P G. Thus (b) holds. Suppose that (b) holds
5
and let Aut(G). Then (P)
is a normal subgroup of G of order [P[. But [(P)P[ = [(P)[[P[/[(P) P[ and [P[ is the largest
power of p which divides [G[. Also [(P)[ = [P[ so that [(P) P[ = [P[ and (P) = P. Thus (c)
is established. If (c) holds then P G and Syl
p
(G) = P so that (a) is satised.
Sylows Theorem is often used in connection with the following remarks:
Remark 5.40. Let G be a nite group. Assume [G[ = mn with (m, n) = 1 and let H, K G
of orders m and n respectively. Then G

= H K.
Proof. Observe that [H K[ divides (m, n) = 1. Hence H K = 1. By an earlier homework
problem we conclude that HK

= HK. This also shows that [HK[ = mn = [G[. Thus G = HK,
with HK

= H K.
Remark 5.41. Let P Syl
p
(G) and let Q be a p-subgroup of G. Then Q N
G
(P) = Q P.
Proof. Let H = Q N
G
(P) and note that Q P H, since P N
G
(P). Since H Q it is
enough to show H P. By Lagranges Theorem H is a p-subgroup of N
G
(P) and P is a normal
p-Sylow subgroup of N
G
(P). Recall that [PH[ = [P[[H[/[P H[ so that [PH[ is a power of p.
But P N
G
(P) so that PH is a p-subgroup of N
G
(P). But p
n
is the largest power of p dividing
[N
G
(P)[ so that [PH[ = [P[ (note P PH). Hence P = PH so that H P.
Remark 5.42 (Frattinis Argument). Let H be a normal subgroup of a nite group G and let
P be a p-Sylow subgroup of H. Then G = H N
G
(P).
Proof. Let g G. Then gPg
1
gHg
1
= H, and both gPg
1
and P are p-Sylow subgroups
of H. According to Theorem 5.38.II there exists h H such that gPg
1
= hPh
1
or, equivalently,
h
1
gP(h
1
g)
1
= P. It follows that h
1
g N
G
(P) and so g H N
G
(P).
Remark 5.43. Let G be a nite group and let P be a p-Sylow subgroup of G. Then
6
one has
that N
G
(N
G
(P)) = N
G
(P). Furthermore, if K _ P then K _ N
P
(K).
Proof. Put H = N
G
(P) and let g N
G
(H). Then P and gPg
1
are p-Sylow subgroups
of H. But P H so that P = gPg
1
. This means that g H and H = N
G
(H). Hence
the rst part of the statement. Let now K be a proper subgroup of P. Then Z(P) ,= 1. If
Z(P) , K the g Z(P) K normalizes K. So assume Z(P) K. Then [P/Z(P)[ < [P[ and
N
P
(K)/Z(P) = N
P/Z(P)
(K/Z(P)) so the result follows by induction.
Remark 5.44. Let G be a nite group and let p be a prime divisor of [G[. Consider the set
S = P
1
, P
2
, . . . , P
t
consisting of the p-Sylow subgroups of G. By Sylows Theorem we have that
G acts by conjugation on S. Hence we have a permutation representation of G into S
t
with
kernel K. The stabilizer of P
i
in G is N
G
(P
i
) so that the kernel of is
t

i=1
N
G
(P
i
) =

gG
N
G
(gPg
1
), P S.
5
The key ingredient in this prove is that the greatest common divisor between [(P)[ and [G: P] is 1. Then
Problem V. of the rst midterm yields that (P) P. Hence (P) = P, as they have the same number of elements.
6
A subgroup H of a group G such that N
G
(H) = H is said to be self-normalizing.
26 5. GROUP ACTIONS
Examples and Applications of Sylows Theorem.
Example 5.45. Let G be a nite group and let p be a prime.
(1) If p [ [G[, then 1 is the unique p-Sylow subgroup of G. If [G[ = p
n
, with n Z
+
, then
G is the unique p-Sylow subgroup of G.
(2) Assume that G is abelian and P = x G[ [x[ = p
n
for some n Z
+
. Then P =
Syl
p
(G) and often P is called the p-primary component of G.
(3) Let G = S
3
. Then Syl
2
(G) = (1 2), (1 3), (2 3). Note G = (1 2), (1 3) so that
(d) of Corollary 5.39 is certainly needed.
(4) Let G = S
4
and let K
4
= 1, (1 2)(3 4), (1 3)(2 4), (1 4)(2 3). Put P
1
= K
4
(1 2),
P
2
= K
4
(1 3) P
3
= K
4
(1 4). Then Syl
2
(G) = P
1
, P
2
, P
3
and n
2
(G) = 3. Put
Q
1
= (1 2 3), Q
2
= (1 3 4), Q
3
= (1 2 4) and Q
4
= (2 3 4). Then Syl
3
(G) =
Q
1
, Q
2
, Q
3
, Q
4
and n
3
(A
4
) = 4 = n
3
(S
4
).
Example 5.46 (Groups of order pq). Let G be a group of order pq, p and q primes with p < q.
Let Q Syl
q
(G) and P Syl
p
(G). Then
(a) QG.
(b) If P G then G is cyclic.
(c) If p divides q 1, there exists
7
a unique (up to isomorphism) non abelian group G with
order pq and n
p
(G) = q.
Proof. Observe that n
q
(G) = 1+kq for some k 0. Also, by Sylows Theorem, n
q
(G) divides
p. But p < q so that k = 0 and Q G. This proves (a). If P G then we have that G = PQ,
P Q = 1 and [x, y] P Q = 1 for all x P and y Q. Thus G
t
= 1 and G is abelian. Let
P = w and Q = t. Then G = wt and so G is cyclic, thus proving (b). Finally, assume that p
divides q 1. Let Q Syl
q
(S
q
) and note that Q = , [[ = q and is a q-cycle. We also have
that [N
S
q
(Q)[ = q(q 1). By assumption p divides q 1 so that N
S
q
(Q) has a subgroup P of order
p, by Cauchys Theorem. Moreover, QN
S
q
(Q) so that H = PQ is a subgroup of N
S
q
(Q) of order
pq. Indeed, [H[ = [P[[Q[/[P Q[ with P Q = 1 by Lagranges Theorem. On the other hand,
C
S
q
() =
i
[ 1 i q so that H is non-abelian. This proves (c). The essential ingredient in
the uniqueness of the construction of H is that Aut(Z
q
)

= Z
q1
.
Example 5.47 (Groups of order 30). Let G be a group of order 30. We show that G has a
normal subgroup isomorphic to Z
15
. Note that G is solvable.
Proof. Let P Syl
5
(G) and let Q Syl
3
(G). Assume either P or Q is normal in G. Then
PQ has order 15. Then PQ has index 2 and hence is normal in G. So it is enough to show that P
or Q is normal in G. By Sylows theorem we have that n
5
(G) divides 6 and n
5
(G) = 1 + 5k with
k Z. So n
5
(G) = 1 or 6. Similarly, n
3
(G) divides 10 and n
3
(G) = 1 + 3k. So, n
3
(G) = 1 or 10.
We assume that n
3
(G) = 10 and n
5
(G) = 6. Each elements of order 5 lies in a 5-Sylow subgroup
and each 5-Sylow subgroup contains 4 non identity elements of order 5. By Lagranges Theorem
distinct 5-Sylow subgroups have trivial intersection. Thus there are 6 4 = 24 elements of order
5 in G. By similar reasoning there are 2 10 = 20 elements of order 3. This is impossible
8
since
[G[ = 30. Thus either P or Q is normal in G.
First assume P G. Now n
3
(PQ) divides 5 and is congruent to 1 modulo 3. This can only
happen if n
3
(PQ) = 1 so that Q PQ and hence it is characteristic. Thus
9
Q G. Assume
that Q G. Then n
5
(PQ) divides 3 and is congruent to 1 modulo 5. This can only happen if
n
5
(PQ) = 1 so that P PQ and hence characteristic. Thus P G. Then PQ = P Q and P Q
is cyclic of order 15.
Example 5.48 (Groups of order p
2
q). Let G be a group of order p
2
q, where p and q are distinct
primes. We show that G has a normal Sylow subgroup , hence G is solvable.
7
This is called the metacyclic group of order p and q. A typical example is given by S
3
, i.e. p = 2 and q = 3.
8
This type of counting works nicely when the p-Sylow subgroups of a group have order p.
9
Here we use the fact that Qchar PQ and PQG implies that QG. Note that PQG as [G: PQ] = 2.
4. SYLOWS THEOREM 27
Proof. Let P Syl
p
(G) and Q Syl
q
(G). Consider rst the case p > q. By Sylows Theorem
n
p
(G) divides q and n
p
(G) = 1 +kp, for some k Z
+
0. So k = 0 and 1 P G is an abelian
series for G. Next, consider p < q. In n
q
(G) = 1, then Q G. Moreover, 1 Q G is an
abelian series for G, as G/Q is abelian given that [G/Q[ = p
2
. Assume that n
q
(G) ,= 1; that
is, n
q
(G) = 1 + tq, with t Z
+
. Now n
q
(G) divides p
2
so that n
q
(G) = p
2
, as q > p. Thus
tq = p
2
1 = (p 1)(p + 1). Since q is a prime strictly greater than p we conclude that q divides
p + 1. Hence p + 1 = ql for some l Z
+
. But l = 1 as q > p. Therefore, p = 2 and q = 3. This
yields that [G[ = 12 and so the result follows from the next example.
Example 5.49 (Groups of order 12). Let G be a group of order
10
12. We show that G has a
normal 3-Sylow subgroup or G

= A
4
. Hence G is solvable.
Proof. Let Q Syl
3
(G). Then n
3
(G) divides 2
2
and n
3
(G) = 1 + 3k. Assume that Q ;
G. Then n
3
(G) = 4. By an easy counting argument, we have 8 elements of order 3. Since
[G: N
G
(Q)] = n
3
(G) = 4 we have that N
G
(Q) = Q. By Remark 5.44, G acts on the four 3-Sylow
subgroups by conjugation
11
so that this action aords a permutation representation : G S
4
.
The kernel K of this action is a subgroup which normalizes all the 3-Sylow subgroups of G. In
particular, K N
G
(Q) = Q. But Q ; G by assumption so K = 1 and is injective. Thus
G

= (G) S
4
. Since G contains 8 elements of order 3 and there are exactly 8 elements of
order 3 in S
4
which are all in A
4
, (G) A
4
has order at least 8. But [A
4
[ = 12, so that by
Lagranges Theorem (G) = A
4
or G

= A
4
. Let P = K
4
be a 2-Sylow subgroup of A
4
. Then
K
4
= 1, (1 2)(3 4), (1 3)(2 4), (1 4)(2 3) so that K
4
A
4
. Therefore, G has a normal 2-Sylow
subgroup.
Groups of order 60.
Proposition 5.50. Let G be a group of order 60 with more than one 5-Sylow subgroup. Then
G is simple.
Proof. Assume, by way of contradiction, that [G[ = 60, n
5
= n
5
(G) > 1 but there exists a
normal subgroup H of G with 1 ,= H ,= G. By Sylows Theorem n
5
(G) = 1 + 5k, k > 0 and
n
5
(G) divides 12. So n
5
(G) is 3, 4, 6 or 12. But we can not solve 3 = 1 + 5k, 4 = 1 + 5k or
12 = 1 + 5k when k Z
+
. Hence 1 + 5k = 6 (i.e. k = 1). Thus n
5
(G) = 6. Let P Syl
5
(G).
Then n
5
(G) = [G: N
G
(P)] = 6 so [N
G
(P)[ = 10. Assume that 5 divides [H[. Then H contains a
5-Sylow subgroup of G, since [G[ = 5 3 2
2
. Therefore, since H is normal in G, H contains all
six conjugates of P. In particular, [H[ 1 + 6 4 = 25, so that the only possibility is [H[ = 30,
using Lagranges Theorem. This leads to a contradiction since by the previous Example it was
proved that any group of order 30 has a normal (hence unique) 5-Sylow subgroup. This argument
shows that 5 does not divide [H[ for any proper normal subgroup H, 1 _ H _ G. If [H[ = 6 or
12, then H has a normal, and hence characteristic, Sylow subgroup which is normal in G. Here,
we are using the fact that a group of order 6 has a normal 3-Sylow subgroup and a group of order
12 has a normal 3-Sylow subgroup or a normal 2-Sylow subgroup. Replacing H by this subgroup,
if necessary, we may assume that [H[ = 3 or 4. Put G = G/H. Hence [G[ = 20 or 15. We know
that if [G[ = 15, then G has a normal 5-Sylow subgroup. Assume [G[ = 20 = 2
2
5. By Sylows
Theorem, n
5
(G) divides 2
2
and n
5
(G) = 1 +5k. This is only possible when n
5
(G) = 1. Therefore,
in all two cases G has a normal 5-Sylow subgroup, say P. Let H
1
be the complete pre-image of P
in G. Then H
1
G and 5 divides [H
1
[. This contradicts the previous argument and so the proof
is complete.
Corollary 5.51. A
5
is simple.
Proof. [A
5
[ = 60 and the subgroups (1 2 3 4 5) and (1 3 2 4 5) are distinct 5-Sylow
subgroups of A
5
. Therefore A
5
is simple by Proposition 5.50.
Proposition 5.52. Let G be a simple group of order 60. Then G

= A
5
.
10
In total there are 3 non commutative groups of order 12 namely Z
2
S
3
, A
4
and Z
3
Z
4
and 2 commutative
groups of order 12 namely Z
12
and Z
2
Z
2
Z
3
.
11
Note that we could also have G acting by left multiplication on the left cosets of Q and use Theorem 5.15.
28 5. GROUP ACTIONS
Proof. Let G be a simple group of order 60. Let P Syl
2
(G) and put N = N
G
(P). By
Sylows Theorem, n
2
= n
2
(G) = [G: N] = 1 + 2k and n
2
divides 15. So n
2
= 3, 5 or 15.
No proper subgroup H of G has index less than 5. For let H be a proper subgroup of G of index 2,
3 or 4. By Theorem 5.15, G has a normal subgroup K such that G/K is isomorphic to a subgroup
of S
2
, S
3
or S
4
. Thus K = 1, since G is simple. This is impossible, since [G[ = 60 does not divide
4!. In particular, one has that n
2
,= 3.
If n
2
= 5, then G

= A
5
. Indeed, if n
2
= [G: N] = 5 then G acts on the 5 left cosets of N in G by
left multiplication. This aords G a permutation representation of G into S
5
with kernel K. But
G is simple so that K = 1. Hence [(G)[ = 60 and (G) S
5
. Assume by way of contradiction,
that (G) , A
5
. Since [S
5
: (G)] = 2 it follows that (G) S
5
and (G)A
5
= S
5
. By the Second
Isomorphism Theorem (G) A
5
has index 2 in A
5
which means that (G) A
5
A
5
. But A
5
is
simple so that we have a contradiction. This means that (G) A
5
, whence (G) = A
5
. Since
is a monomorphism the result follows in this case.
Suppose that n
2
= 15. If for every pair of 2-Sylow subgroups P and Q of G we have that P Q = 1
then the number of non identity elements in the 2-Sylow subgroups of G would be 3 15 = 45.
Note that n
5
= n
5
(G) divides 12 and n
5
= 1 + 5k. A 5-Sylow subgroup W is not normal in G
since G is simple, so that n
5
> 1. This means that n
5
= 6. Hence the number of non identity
elements in G of order 5 is 4 6 = 24. This means that we have at least 45 +24 +1 = 70 elements
in G, which is impossible. This shows that there exists distinct 2-Sylow subgroups P and Q such
that [P Q[ = 2. Let M = N
G
(P Q). Since [P[ = [Q[ = 4, P and Q are abelian and hence
normalize P Q. Hence P, Q M = N
G
(P Q). Since G is simple, M ,= G. Notice that
[P, Q[ = [P[[Q[/[P Q[ = 4 4/2 = 8 so that [M[ is at least 8 and a 2-Sylow subgroup of G
is contained in M. Hence [M[ = 12, 20 or 60. But M ,= G and if [M[ = 20, then [G: M] = 3
so that as we already saw G has a faithful permutation representation : G S
3
, which is
impossible. Therefore, [M[ = 12 and so [G: M] = 5. Now use again the previous argument to
conclude that G

= A
5
.
4. SYLOWS THEOREM 29
Homework set # 5.
Solve 5 of the following problems, with the following requirement. The rst problem must be
chosen among the problems numbered 1.-4.; the second one among those numbered 5.-8.; the
third one among those numbered 9.-12.; the fourth one among those numbered 13.-16.; the fth
one among those numbered 17.-20.
The due date is on November 9 (Friday), 2001.
1. If Z(G) is of index n, prove that every conjugacy class has at most n elements.
2. Let p be a prime and let G be a group of order p
n
. Prove that G has a subgroup of order
p
m
for every 0 m n.
3. Show that SL(2, 1) acts faithfully and transitively on 1
2
. Show that
G
(0,1)
=
__
1 0
a 1
_
[ a 1
_

= 1.
4. If G is a group of odd order, prove for any identity element g G that g and g
1
are not
conjugate in G.
5. Let G be a group of order 203. Prove that if H is a normal subgroup of order 7 in G then
H is contained in Z(G). Deduce that G is abelian in this case.
6. Let G be a group. Prove that characteristic subgroups are normal. Show that the derived
subgroup G
t
is a characteristic subgroup. Give an example of a normal subgroup that is
not characteristic.
7. Show that Aut(A
4
)

= S
4
.
8. Show that the only elements in S
n
commuting with the n-cycle = (1 2 . . . n) are
elements of the form
i
.
9. Show that S
n
is generated by (1 2) and (1 2 . . . n).
10. Let p be a prime number and let H be a subgroup of S
p
containing a transposition and
acting transitively on 1, 2, . . . , p. Show that H = S
p
.
11. Let G be a group acting transitively on a nite set S with [S[ > 1. Show that there exists
a g G such that g x ,= x for every x S (i.e., g has not xed point).
12. Let G have order 99. Show that G is abelian.
13. Prove that if [G[ = 132 then G is not simple.
14. Let G be a nite group, p a prime, and N the intersection over all p-Sylow subgroups of
G. Show that N is a normal p-subgroup of G, and that every normal p-subgroup of G is
contained in N.
15. Let G be a nite group and P a p-Sylow subgroup. Then N
G
(N
G
(P)) = N
G
(P).
16. Let G be a group of order pqr, with p < q < r primes.
Show that G has a normal subgroup of order qr.
Show that the r-Sylow subgroup of G is normal in G.
17. Let H be a subgroup of order n of a nite group, and assume that n and [G: H] are
relatively prime. Show that if H is normal in G, then H is the only subgroup of G of
order n.
18. Let G be a group of order 2
n
3, n 2. Show that G has a normal 2-subgroup ,= 1.
19. Let G be a group of order 231. Let H be an 11-Sylow subgroup of G and let K be a
7-Sylow subgroup of G. Show that H, K G and that H Z(G).
20. Let G be a group of order n and let : G S
n
be given by the action of G on itself via
translation.
For g G, determine the number and the lengths of the disjoint cycles of the
permutation (g).
Show that (G) , A
n
if and only if n is even and G has a cyclic 2-Sylow subgroup.
If n = 2m, m odd, show that G has a subgroup of index 2.
CHAPTER 6
Introduction To Rings
1. Basic Denitions and Examples
A ring R is a set with two binary operations + and (called addition and multiplication) satisfying
the following axioms:
(a) (R, +) is an abelian group
1
.
(b) is an associative operation: that is (a b) c = a (b c) for all a, b, c R.
(c) the distribution laws hold: that is for all a, b, c R one has (a + b) c = a c + b c and
a (b +c) = a b +a c.
The ring is said to be commutative if a b = b a for all a, b R. The ring is said to have an
identity (element) if there exists an element 1 R such that 1 a = a 1 = a for all a R.
Remark 6.1. Let S be a set with binary operations + and such that: (a) (S, +) is a group;
(b) is associative; (c) the distributive laws hold; (d) S has an identity. Then (S, +) is abelian.
Proof. For any a, b R we have (1 + 1)(a + b) = (a + b) + (a + b) = a + b + a + b and also
(1+1)(a+b) = (1+1)a+(1+1)b = a+a+b +b. Hence a+b +a+b = a+a+b +b. Cancellation
laws hold in the group (S, +) so that b +a = a +b.
Proposition 6.2. Let R be a ring. Then
(a) 0a = a0 = 0 for all a R
(b) (a)b = a(b) = ab for all a, b R. (a is the additive inverse of a.)
(c) (a)(b) = ab.
(d) (na)b = a(nb) = n(an) for every n Z.
(e) If R has an identity, then the identity is unique and a = (1)a for any a R.
Proof. Note that 0a = (0+0)a = 0a+0a. So we have that 0 = 0a0a = 0a. Similarly, a0 = 0.
This proves (a). Now, ab + (a)b = (a + (a))b = 0b = 0 by (a). Hence (a)b = ab. Similarly,
a(b) = ab and the proof of (b) is complete. Observe that (a)(b) = a(b) = (ab) = ab,
by (b) and the uniqueness of the additive inverse. This yields (c). Property (d) follows from the
distributivity and (a) and (b). Finally, assume that 1 and 1
t
are identities of R. Then 1 = 11
t
= 1
t
.
Also, 0 = (1 1)a = a + (1)a. Hence (1)a = a for any a R. This completes our proof.
Let R be a ring. A nonzero element a R is called a left (right) zero divisor provided that there
is a nonzero element b R such that ab = 0 (ba = 0, respectively). The element a is called a zero
divisor if it is both a left and right divisor. Assume that R has an identity 1 ,= 0. An element
u R is called left (right) invertible provided that there is some v R such that vu = 1 (uv = 1,
respectively). The element u is called invertible, or a unit, if it is both left and right invertible. The
element v is called the (left, right) inverse of u. The set of units of R is denoted by U(R) = R

.
Proposition 6.3. Let R be a ring with identity 1 ,= 0. If u is a unit then the left and right
inverses of u coincide. Moreover, (R

, ) is a group under multiplication.


Proof. If v
1
u = 1 and uv
2
= 1, then v
1
= v
1
1 = v
1
(uv
2
) = (v
1
u)v
2
= 1 v
2
= v
2
. This proves
the uniqueness of the inverses. Let u
1
, u
2
R

. Then there are elements v


1
, v
2
R (actually
in R

) such that u
1
v
1
= 1 = v
1
u
1
and u
2
v
2
= 1 = v
2
u
2
. It follows that (u
1
u
2
)(v
2
v
1
) = 1 =
(v
2
v
1
)(u
1
u
2
) so that u
1
u
2
R

. Also, u
1
i
= v
i
R

for i = 1, 2. Thus R

is a group.
Proposition 6.4. Let a, b, c R with a not a zero divisors. If ab = ac, then a = 0 or b = c.
1
The identity element is usually denoted with 0.
31
32 6. INTRODUCTION TO RINGS
Proof. If ab = bc then a(b c) = 0. If a ,= 0 then b c = 0, as a is not a zero divisor.
Let R be a ring with an identity 1 ,= 0. Then
1. R is called an integral domain provided that it is commutative and 0 is the only zero
divisors (i.e., ab = 0 implies a = 0 or b = 0).
2. R is called a division ring (or skew eld) if R

= R 0.
3. R is called a eld if it is a commutative division ring or, equivalently, if it is an integral
domain with R

= R 0.
Corollary 6.5. A nite integral domain is a eld.
Proof. Let R be a nite integral domain, and let a R be a nonzero element. By the
cancellation law, the map R R dened by x ax is an injective map. Since R is nite, this
map is also onto. In particular, there is some b R such that ab = 1. Since R is commutative,
ab = ba = 1. Since a was an arbitrary nonzero element, R is a eld.
Example 6.6.
(1) Z, the ring of integers, is an integral domain that is not a eld: Z

= 1.
(2) The rings , 1 and C are elds.
(3) Z
n
, with n Z
+
, is a ring. It is a eld if and only if n is a prime. Also, it is an integral
domain if and only if n is a prime.
Proof. Dene a multiplication on Z
n
by xy = xy. This operation is well dened,
since x x
t
mod n and y y
t
mod n implies that xy x
t
y
t
mod n. Let x Z
n
. Then
x Z

if and only if xy = 1 for some y Z. This occurs if and only if xy = 1 + sn for


some s Z or, equivalently, 1 = xy sn. This equivalent to saying that (x, n) = 1. In
particular, [Z

n
[ = (n). Hence Z
n
is a eld if and only if 0 < (n) = n 1, that is n is a
prime. Now, If n = p is a prime then Z
p
is a eld so in particular it is an integral domain.
Conversely, suppose that n = pm, with 1 < p, m < n and p a prime. Then p and m are
nonzero in Z
n
. However, p m = n = 0 and so Z
n
has zero divisors.
(4) Let R be an abelian group. Dene a b = 0 for a, b R. Then R becomes a ring, the so
called zero ring.
(5) Let X be a nonempty set and A any ring. Let R = M(X, A) be the set of all functions
from X into A. Dene f +g and fg, where f, g R, by
(f +g)(x) = f(x) +g(x) (fg)(x) = f(x)g(x) x X
Then R becomes a ring. Assume that the ring A has an identity 1 ,= 0. Dene f
1
R
by f
1
(x) = 1 for all x X. Then (ff
1
)(x) = f(x)f
1
(x) = f(x) for all x X. Hence f
1
is
the identity for R.
(6) Let X = [0, 1] be the unit interval and 1 the ring of real numbers. R = M(X, 1) is the
ring of functions in X. ((R) = f R[ f is continuous. ((R) is a ring with identity.
(7) Let G be an abelian group and End(G) = f [ f endomorphism of G. On End(G) dene
f +g by (f +g)(x) = f(x) +g(x) for x G, and fg = f g. Then End(G) is a ring with
(End(G))

= Aut(G). End(G) is called the endomorphism ring of G.


A nonempty subset S of a ring R is called a subring if S is a subgroup of (R, +) and is closed
under multiplication.
Remark 6.7. Let S be a nonempty subset of a ring R. S is a subring of R if and only if
whenever a b, ab S for any a, b S.
Example 6.8.
(1) Let E = 2Z denote the set of even integers. Then E is a subring of Z. But E does not
have an identity and is not an integral domain.
(2) Z(R) = r R[ rx = xr for all x R is called the center of R. It is a subring of R.
(3) Z is a subring of with the same identity but it is not a eld.
2. MORE EXAMPLES OF RINGS 33
2. More Examples of Rings
Polynomial Rings.
Let R be a commutative ring with identity 1. Assume that R is a subring of a commutative ring
L with the same identity 1 as R. An element x L is called an indeterminate over R if whenever
a
0
+ a
1
x + a
2
x
2
+ . . . + a
n
x
n
= 0, with all a
i
R, then a
0
= a
1
= . . . = a
n
= 0. Let x be an
indeterminate over R and let R[x] denot the set of polynomials f(x) = a
0
+ a
1
x + . . . + a
n
x
n
,
with all a
i
R. If a
n
,= 0 then f(x) is said to have degree n and a
n
x
n
is the leading term of the
polynomial. If a
n
= 1 then f(x) is called monic. Let f(x) = a
0
+ a
1
x + a
2
x
2
+ . . . + a
n
x
n
and
g(x) = b
0
+b
1
x +. . . +b
m
x
m
. Assume that m n. Dene
f(x) +g(x) = (a
0
+b
0
) + (a
1
+b
1
)x +. . . + (a
n
+b
n
)x
n
+b
n+1
x
n+1
+. . . +b
m
x
m
.
Now let a
n+1
= a
n+2
= . . . = a
m
= 0 and dene
f(x)g(x) = a
0
b
0
+ (a
0
b
1
+a
1
b
0
)x + (a
0
b
2
+a
1
b
1
+a
2
b
0
)x
2
+. . . ,
where, in general, the coecient of x
k
is
k

i=0
a
i
b
ki
. These operations make sense since R R[x]
L, R is a subring of L. With these operations R[x] becomes a commutative ring with identity 1.
Note that R is just the subring of constant polynomials in R[x].
Example 6.9. Examples include: Z[x], [x], Z
n
[x], 1[x]. In Z
3
[x] the polynomials have
coecients 0, 1, 2 and calculations on the coecients are done modulo 3. For example, if f(x) =
x
2
+ 2x + 1 and g(x) = x
3
+x + 2 then
f(x) +g(x) = x
3
+x f(x)g(x) = x
5
+ 2x
4
+ 2x
3
+x
2
+ 3x + 2.
Proposition 6.10. Let R be an integral domain and let p(x), g(x) R[x]. Then
(a) deg(p(x)g(x)) = deg(p(x)) + deg(g(x)).
(b) U(R[x]) = U(R).
(c) R[x] is an integral domain.
Proof. Since R is an integral domain it has no zero divisors. Let p(x) (resp. g(x)) be a
polynomial in R[x] with leading coecient a
n
x
n
(resp. b
m
x
m
). Then the leading coecient of
p(x)g(x) is a
n
b
m
x
n+m
and a
n
b
m
,= 0. This proves (a) and (c). Let p(x) R[x] be a unit, say
p(x)f(x) = 1. Then, by (a), we have that deg(p(x)f(x)) = deg(p(x)) + deg(f(x)) = 0. Hence
deg(p(x)) = 0 and p(x) R. This proves (b).
Let R be a commutative ring with identity 1 ,= 0 and let S be a subring containing 1. Then
S[x] is a subring of R[x]. In particular, Z[x] is a subring of [x]. Assume that R has zero divisors.
Then so does R[x] since R R[x]. Let f(x) R[x] be a zero divisor. Then there is a non zero
polynomial g(x) R[x] such that f(x)g(x) = 0. From the denition of multiplication in R[x] and
the fact that x is an indeterminate over R, it follows that g(x) = c, a nonzero element of R.
Example 6.11.
(1) The set E[x] = f(x) Z[x] [ coecients of f(x) are even integers is a subring of Z[x].
(2) The set S[x] = f(x) [x] [ only even powers of f(x) appear is a subring of [x].
Matrix Rings.
Let n Z
+
and let R be a ring. Let Mat(n, R) = M
n
(R) denote the set of all n n matrices over
R. Then with the usual + and , M
n
(R) becomes a ring. Also, if R has an identity then so does
M
n
(R). If R = k is a eld then M
n
(k)

= GL
n
(k). If n 2, then M
n
(R) is not commutative.
Moreover, M
n
(R) has zero divisors for all non zero rings R. Indeed, let A be the n n matrix
with entry a
11
= a R and with zero entries anywhere else. Let B be the nn matrix with entry
b
12
= b R and with zero entries anywhere else. Note that BA is the zero matrix while AB is not
zero unless ab = 0. A matrix A = (a
ij
) of M
n
(R) is called a scalar matrix if a
ii
R and a
ij
= 0
if i ,= j. The set of scalar matrices S is a subring of M
n
(R). Also, the map R S dened by
a a1
n
is a ring isomorphism. If R is commutative, then S Z(R), the set of all matrices in
M
n
(R) that commute with each element in M
n
(R).
34 6. INTRODUCTION TO RINGS
Example 6.12.
1. Let S be a subring of R. Then M
n
(S) is a subring of M
n
(R).
2. Let Mat
2
(R) be the ring of 2 2 matrices over the ring R. Let
S =
__
a b
0 c
_
[ a, b, c R
_
.
Then S is a subring of Mat
2
(R). More generally, the collection of all n n upper (lower)
triangular matrices is a subring of M
n
(R).
Group Rings.
Let R be a xed commutative
2
ring with 1 ,= 0 and let G = g
1
, g
2
, . . . , g
n
be a nite group
with the group operation written multiplicatively. Dene the group ring RG = R[G] of G with
coecients in R to be the set of all nite sums a
1
g
1
+a
2
g
2
+. . . +a
n
g
n
, where a
i
R for 1 i n.
If g
1
= 1
G
is the identity of G, then we shall write a
1
g
1
simply as a
1
. It is called the constant term
of the given element. Similarly, we shall write 1g for g G simply as g. The addition is dened
componentwise, while the multiplication is dened as follows
_
n

i=1
a
i
g
i
_
_
_
n

j=1
b
j
g
j
_
_
=
n

k=1
_
_

g
i
g
j
=g
k
a
i
b
j
_
_
g
k
where a
i
, b
j
R and g
i
, g
j
, g
k
G. With these operations RG becomes a ring with identity 1,
where 1 is the identity in R. Note that R[G] is commutative if and only if G is abelian. Note that
the associativity of multiplication in RG follows from associativity of multiplication in G.
Example 6.13. Let D
8
= r, s [ r
4
= s
2
= 1, rs = r
1
s and let Z be the ring of integers.
Typical elements in Z[D
8
] might be = r +r
2
2s and = 3r
2
+rs. Then
+ = r 2r
2
2s +rs
= (r +r
2
2s)(3r
2
+rs) = r(3r
2
+rs) +r
2
(3r
2
+rs) 2s(3r
2
+rs)
= 3r
3
+r
2
s 3 +r
3
s + 6sr
2
2srs = 3r
3
3 + 7r
2
s +r
3
s 2r
3
= 3 5r
3
+ 7r
2
s +r
3
s.
Remark 6.14.
(1.) The map R R[G] dened by r r 1
G
is a ring monomorphism.
(2.) If r R and g G, then rg = gr so that the ring elements commute with each element
in RG. (Recall that R is commutative.)
(3.) The map G R[G] dened by g 1 g is a group monomorphism.
(4.) Each element g G has an inverse in R[G], so that G U(R[G]).
(5.) Assume [G[ = n > 1 and let g G have order m. Then
(1 g)(1 +g +g
2
+. . . +g
m1
) = 1 g
m
= 1 1 = 0.
Hence 1 g is a zero divisor in R[G].
(6.) Z[G] is called the integral group ring; [G] is called the rational group ring.
(7.) The set S

i
a
i
g
i
R[G] [

i
a
i
= 0 is a subring of R[G], but does not have an identity.
(8.) The set C of all elements of R[G] with constant term zero is a subring of R[G].
Hamiltons Real Quaternions.
Let H be subset of the ring M
2
(C)
H =
__
x y
y x
_
[ x, y C
_
where denotes complex conjugation. Then, with the induced operations, His a ( non commutative)
ring. Let H a =
_
x y
y x
_
and write x =
0
+
1
i and y =
2
+
3
i, with
i
1. Further
write = det a = xx + yy =
2
0
+
2
1
+
2
2
+
2
3
. If a ,= 0, then ,= 0 and 1, that
2
Note that we do not need R to be a commutative ring.
3. IDEALS AND RING HOMOMORPHISMS 35
b =
1

_
x y
y x
_
H. Notice that ab = ba =
_
1 0
0 1
_
. Thus H is a (non commutative) division
ring, called the division ring of Hamiltons real quaternions. Write
1 =
_
1 0
0 1
_
I =
_
i 0
0 i
_
J =
_
0 1
1 0
_
K =
_
0 i
i 0
_
.
Then every a H can be uniquely written in the form a =
0
1 +
1
I +
2
J +
3
K. Notice
that I
2
= J
2
= K
2
= 1 and IJ = JI = K, JK = KJ = I, KI = IK = J. Thus
1, I, J, K is a multiplicative group isomorphic to Q
8
.
Remark 6.15. Note that the group ring 1[Q
8
] is not the same ring as the ring of Hamiltons
real quaternions H, even though the latter contains a copy of the quaternion group Q
8
(under
multiplication). As noted above the group ring 1[Q
8
] contains zero divisors while H is a division
ring. Another dierence is that I
2
+1 = 0 in H but not in 1[Q
8
]. Note that I
2
= J
2
= K
2
is the
unique element of order 2 in Q
8
.
3. Ideals and Ring Homomorphisms
Properties of ideals. Let R and I R. We say that I is a left (right) ideal if I is a subgroup
and rx ( resp. xr) belongs to I for every r R and x I. We say that I is an ideal if it is both a
left and a right ideal.
Example 6.16.
1. 0, the trivial ideal, and R, the unit ideal, are R-ideals.
2. (n) = nZ = nz [ z Z is a Z-ideal.
3. is a subring of 1 but, for example,

2 1/5 , so that is not an 1-ideal.


4. Let R be a ring and n 2. The subset of M
n
(R) consisting of all matrices having nonzero
entries in column i is a left ideal in M
n
(R), but not a right ideal (viceversa, for row i).
5. Let F be a eld and let M
n
(F) with n 2. Then the only two sided ideals of M
n
(F) are
0 and M
n
(F) itself. M
n
(F) is not a division ring.
Proposition 6.17. Let R be a ring with identity 1 ,= 0 and let I be a left (right) R-ideal.
(a) I = R if and only if 1 R if and only if I contains a unit.
(b) R is a division ring if and only if R has no left and right ideals other than 0 and R.
Proof. We prove (b). If R is a division ring then every nonzero element is invertible so the
statement is clear. Conversely, let a R and suppose a ,= 0. Then Ra is a left R-ideal ,= 0, hence
Ra = R 1 so that ba = 1 for some b R. Hence a is left invertible. Likewise, a is right invertible,
and hence invertible.
Let R be a ring with identity 1
R
,= 0. Let I
j
[ j S be a family of (left, right) R-ideals.
Then

jS
I
j
is again a (left, right) R-ideal. Let Z R be a subset. The left-ideal generated
by Z is the smallest left R-ideal containing Z. Equivalently, it is the intersection of all left R-
ideals containing Z or
_

nite
r
i
z
i
[ r
i
R, z
i
Z
_
. Likewise for right ideals. Finally, the ideal
generated by Z, denoted by (Z), is the smallest R-ideal containing Z. Equivalently, it is the
intersection of all R-ideals containing Z or (Z) =
_

nite
r
i
z
i
s
i
[ r
i
, s
i
R, z
i
Z
_
. If Z = (z), then
Rz = rz [ r R is the left ideal generated by z. The ideal generated by z is then (z) = RzR =
_

nite
r
i
zs
i
[ r
i
, s
i
R
_
. An R-ideal I is said to be nitely generated if I = (Z) for some nite set
Z; it is said to be principal if I = (z) for some z R. Let a, b R, a commutative ring; we say
that a divides b provided that there exists c R such that b = ac. Then a divides b if and only if
(b) (a).
36 6. INTRODUCTION TO RINGS
Example 6.18.
1. Let F be a eld. Well show later that every ideal in F[x] is principal.
2. Let I = (2, x) Z[x]. We show that I is not principal. Note that Z[x] is an integral
domain. Hence (2, x) = 2a(x)+xb(x) [ a(x), b(x) Z[x]. This means that (2, x) consists
precisely of polynomials with integer coecients whose constant term is even. Assume
by way of contradiction that (2, x) = (g(x)) for some g(x) Z[x]. Since 2 (g(x)) there
is a polynomial p(x) Z[x] such that 2 = p(x)g(x). Now, Z is an integral domain so
deg(g(x)) + deg(p(x)) = 0. So g(x) and p(x) must be constant. In particular, 2 is a
prime so that g(x), p(x) 1, 2. If g(x) = 1 then every polynomial in Z[x] would
be a multiple of g(x), contrary to (g(x)) being a proper ideal. The only possibility is
g(x) = 2. But now x (2) = (2) so that x = 2h(x) with h(x) Z[x]. But this is
impossible since x is an indeterminate element over Z.
A commutative ring R is said to be Noetherian if every R-ideal is nitely generated; it is said to
be a principal ideal ring (domain) if every R-ideal is principal (and R is an integral domain).
Let I
1
, . . . , I
n
be (left, right) R-ideals. Then I
1
I
n
=
_

nite
a
i
1
a
i
n
[, a
i
j
I
j
_
. In particular,
I
n
= I I (n times). If I
1
, . . . , I
n
are (left, right) R-ideals, then so are I
1
+ +I
n
and I
1
I
n
Ring Homomorphisms. Let R and S be rings. A map f : R S is called a homomorphism
(of rings) if
3
for all a, b R then f(a + b) = f(a) + f(b) and f(ab) = f(a)f(b). As before one
considers monomorphisms, epimorphisms, isomorphisms of rings.
Proposition 6.19. Let f : R S be a homomorphism of rings. Then
(a) Im(f) = f(R) = f(r) [ r R is a subring of S;
(b) Ker(f) = x R[ f(x) = 0 is an R-ideal.
Proof. Let f(r
1
), f(r
2
) be elements of Im(f). Then f(r
1
)f(r
2
) = f(r
1
r
2
) and f(r
1
)f(r
2
) =
f(r
1
r
2
) belong to Im(f). This shows that Im(f) is a subring of S, hence (a). Now, let x
1
, x
2
be
elements in Ker(f) and let r R. Then f(x
1
x
2
) = f(x
1
) f(x
2
) = 0 0 = 0 so that
x
1
x
2
Ker(f). Also, f(rx) = f(r)f(x) = f(r)0 = 0 and rx Ker(f). Similarly, xr Ker(f)
and so Ker(f) is an R-ideal. This completes the proof of (b).
Example 6.20.
1. Let n Z
+
. The map f : Z Z
n
dened by f(r) = r = [r] is a ring epimorphism.
2. The map f : [x] dened by f(q) = q is a ring monomorphism.
3. Let R be a commutative ring with identity. Dene : R[x] R by (f(x)) = f(0) for
any f(x) R[x]. Then is a ring epimorphism with kernel Ker() = g(x) R[x] [ g(x) =
a
1
x +. . . +a
n
x
n
= (x)R[x], the ideal generated by x.
4. Let x R, where R is a ring. Dene f
x
: R R by f
x
(r) = xr. Observe that f
x
is a group homomorphism of R into R (as groups). Note that f
x
(r
1
r
2
) = xr
1
r
2
while
f
x
(r
1
)f
x
(r
2
) = xr
1
xr
2
for any r
1
, r
2
R. When is f
x
a ring homomorphism?
5. Let R be a eld and f : R S be a nonzero ring homomorphism. Then f is a monomor-
phism. This is because Ker(f) is the 0 ideal.
Let R be a ring, I an R-ideal. In particular I is a subgroup of the additive group R so that we
can form the factor group R/I where the addition is dened by (r
1
+I) +(r
2
+I) = (r
1
+r
2
) +I.
Let : R R/I be the canonical projection. On R/I we dene a multiplication by x y = xy.
Theorem 6.21. With the above operation, R/I is a ring (called a factor ring) and is an
epimorphism of rings with Ker() = I.
Proof. We only need to verify that the multiplication on R/I is well dened. Suppose that
x = x
1
and y = y
1
, or equivalently xx
1
and yy
1
I. Then xyx
1
y
1
= xyxy
1
+xy
1
x
1
y
1
=
x(y y
1
) + (x x
1
)y
1
belongs to I as I is an R-ideal. This shows that xy = x
1
y
1
.
3
In many contexts, people require R and S to have identity elements 1
R
and 1
S
and that f(1
R
) = 1
S
. The
latter condition will always be forced whenever S is an integral domain. Indeed, f(1
R
) = f(1
R
1
R
) = f(1
R
)f(1
R
)
implies f(1
R
)(1
S
f(1
R
)) = 0. Now, if f is not the trivial homomorphism this forces f(1
R
) = 1
S
.
3. IDEALS AND RING HOMOMORPHISMS 37
Remark 6.22. Note that I is an ideal if and only if I is the kernel of a rings homomorphism.
Theorem 6.23 (Isomorphism Theorems for Rings). Let f : R S be a homomorphism of
rings. Then there exists a unique monomorphism of rings f : R/I S such that f = f. In
particular
I. f induces R/Ker(f)

= Im(f).
II. Let J be a subring of R and I an R-ideal. Then J +I = x +y [ x J, y I is subring
of R and the map g : J J + I/I dened by g(x) = x + I is a ring epimorphism with
kernel J I. Hence J/J I

= J +I/I.
III. Let I J be R-ideals. Then J/I = x + I [ x J is an R/I-ideal and the map
h: R/I R/J dened by h(r + I) = r + J is a ring epimorphism with kernel J/I.
Thus (R/I)/(J/I)

= R/J.
Proof. We prove the rst part of the statement. By the First Isomorphism Theorem for
groups, there exists an unique homomorphism of groups f such that f = f. It suces to
show that f is a homomorphism of rings: f(xy) = f(xy) = f((xy)) = f(xy) = f(x)f(y) =
f((x))f((y)) = f(x)f(y). Furthermore, if f(1
R
) = 1
S
then f(1
R
) = 1
S
.
Remark 6.24. Let I be an R-ideal. The correspondence J Y J/I is an inclusion preserving
bijection between the set of subrings (resp. left ideals) J of R containing I and the set of subrings
(resp. left ideals) J/I of R/I. Furthermore, a subring J containing I is an ideal of R if and only
if J/I is an ideal of R/I.
Example 6.25. Let R = Z[x], the ring of polynomials over Z. Let I be the collection of all
polynomials in R whose terms are of degree at least 2 with the zero polynomial. Then I is an ideal
of R. For the sum of any two such polynomials again has terms of degree at least 2 (or it is the
zero polynomial) and the product of a polynomial whose terms are of degree at least 2 with any
polynomial is again a polynomial whose terms are of degree at least 2 (or the zero polynomial).
Let p(x), q(x) Z[x]. Then p(x) + I = q(x) + I if and only if p(x) q(x) I if and only if
p(x) and q(x) have the same constant and rst degree terms. It follows that a complete set of
representatives for the quotient R/I is given by the polynomials a + bx of degree at most 1. Put
R = R/I. Then addition and multiplication is performed by representatives. For example
1 + 3x +4 + 5x = 3 + 8x 1 + 3x 4 + 5x = 4 7x.
Note that x
2
= 0 in R/I while x is nonzero. Hence R/I has zero divisors while Z[x] does not.
Characteristic of a Ring.
Lemma 6.26 (Binomial Theorem). Let R be a commutative ring with 1 ,= 0 and let n, i, k and
p denote integers.
(a) Let a, b be elements of R and let n 0. Then
(a +b)
n
=
n

k=0
_
n
k
_
a
k
b
nk
.
(b) Let p be a prime number, i 0, and assume 0 < k < p
i
. Then p divides
_
p
i
k
_
.
Proof. The proof of (a) follows by induction on n and the fact that
_
n
k
_
+
_
n
k+1
_
=
_
n+1
k+1
_
for
0 k < n. As far as (b) is concerned, since 0 < k p
i
, we may write
_
p
i
k
_
= p
i
/k
_
p
i
1
k1
_
. Thus
k
_
p
i
k
_
= p
i
_
p
i
1
k1
_
, where
_
p
i
k
_
and
_
p
i
1
k1
_
are integers. Since 0 < k < p
i
we have that p
i
[ k; hence,
since p is prime, the fact that p
i
divides k
_
p
i
k
_
implies that p divides
_
p
i
k
_
.
Let R be a ring. If there exists a positive integer n with n 1 = 0 in R, then R is said to have
positive characteristic; the characteristic of R is the smallest such n (write char(R) = n). If there
does not exist a positive integer n with n 1 = 0 in R, then R is said to have characteristic zero
(write char(R) = 0).
Proposition 6.27. Let R be a ring with char(R) = n 0.
38 6. INTRODUCTION TO RINGS
(a) na = 0 for every a R.
(b) If 0 is the only zero divisor in R ,= 0 (for example if R is an integral domain, a eld, a
division ring) then char(R) is either 0 or a prime number.
Proof. Observe that na = n(1 a) = (n 1)a = 0a = 0 for all a R. This proves (a). Suppose
n > 0. Then n ,= 1 since R ,= 0. Thus, if n is not prime then n = n
1
n
2
with 0 < n
1
< n and
0 < n
2
< n. Now 0 = n 1 = (n
1
n
2
) 1 = n
1
(n
2
1) = (n
1
1)(n
2
1). Thus n
1
1 = 0 or n
2
1 = 0,
as 0 is the only zero divisor. But this contradicts the minimality of n. This proves (b).
Let R be a ring. Dene R
0
= n 1 [ n Z. It is the smallest subring of R (called the prime
subring of R). Notice that R
0
Z(R).
Proposition 6.28. Let R be a ring, n 0 an integer. The following are equivalent
(a) char(R) = n.
(b) R
0

= Z
n
.
(a) R has a subring isomorphic to Z
n
.
Proof. Suppose that (a) holds. Let f : Z R with f(i) = i 1. Then f is a homomorphism
of rings with Im(f) = R
0
. Thus by the First Isomorphism Theorem, R
0

= Z/Ker(f). But
Ker(f) = mZ for some m 0. Thus R
0

= Z
m
. Now m = char(Z
m
) = char(R
0
) = char(R) = n.
Thus m = n and R
0

= Z
n
and (b) is established. Clearly (b) implies property (c). Finally, suppose
that (c) holds. Then (a) easily follows since char(Z
n
) = n and the characteristic does not change
when passing to a subring.
Proposition 6.29. Let R be a commutative ring with char(R) = p prime. Let F : R R be
dened by F(a) = a
p
.
(a) For a, b R then (a +b)
p
= a
p
+b
p
.
(b) F is a homomorphism of rings (called the Frobenius homomorphism).
Proof. Part (a) follows from Lemma 6.26. Part (b) is a consequence of (a).
4. The Chinese Remainder Theorem
Let R be a commutative ring with identity 1 ,= 0. Let I and J be two R-ideals. In general the
product IJ is contained in I J, but does not necessarily coincide with it. However, if I +J = R
in which case we say that I and J are comaximal or coprime then IJ = I J. Indeed
I J = (I J)R = (I J)(I +J) = (I J)I + (I J)J. But (I J)I IJ and (I J)J IJ.
It follows that I J IJ and the proof is complete. By induction we can actually prove the
following result.
Lemma 6.30. Let R be a commutative ring with identity 1 ,= 0. Let I
1
, . . . , I
n
be R-ideals which
are comaximal in pairs. Then for every i = 1, . . . , n the ideals I
i
and

j,=i
I
j
are comaximal and
I
1
I
n
= I
1
. . . I
n
.
Proof. Left as an exercise.
The map : R R R given by (r) = (r, . . . , r) is a monomorphism of rings.
Theorem 6.31 (Chinese Remainder Theorem). Let R be a commutative ring with identity 1 ,=
0. Let I
1
, . . . , I
n
be R-ideals. Then induces a monomorpshim R/I
1
. . .I
n
R/I
1
R/I
n
.
If I
i
+I
j
= R for any two i ,= j, then induces an isomorphism
R/I
1
. . . I
n

= R/I
1
R/I
n
.
Proof. induces a homomorphismf : R R/I
1
R/I
n
given by f(a) = (a+I
1
, . . . , a+
I
n
). Notice that Ker(f) = I
1
. . . I
n
. Thus by the First Isomorphism Theorem, f induces a
monomorphism f : R/I
1
. . .I
n
R/I
1
R/I
n
, which is surjective if and only if f is. Thus
we have to show that f is surjective if I
i
+I
j
= R, for i ,= j. So assume I
i
+I
j
= R whenever i ,= j.
We may also assume that n 2. Now R =
n

j=2
(I
1
+ I
j
) I
1
+
n

j=2
I
j
I
1
+
n

j=2
I
j
. Thus we can
4. THE CHINESE REMAINDER THEOREM 39
write 1 = b
1
+ c
1
for some b
1
I
1
and c
1

j,=1
I
j
. Thus f(c
1
) = (1, 0, . . . , 0). Now by symmetry,
for all i there exists c
i
R such that f(c
i
) = (0, . . . , 1, . . . , 0) with 1 in the i-th spot. Now let
(a
1
, . . . , a
n
) be an element in R/I
1
R/I
n
. Then f(a
1
c
1
+. . . +a
n
c
n
) = (a
1
, . . . , a
n
).
Remark 6.32. In view of Lemma 6.30, if I
I
+I
j
= R for any two i ,= j the Chinese Remainder
Theorem can also be stated as: R/I
1
I
n

= R/I
1
R/I
n
.
Corollary 6.33. Let I
1
, . . . , I
n
be R-ideals such that I
i
+ I
j
= R whenever i ,= j. Pick
a
1
, . . . , a
n
in R. Then there exists x R simultaneously solving the congruences
x a
1
mod I
1
, , x a
n
mod I
n
.
Furthermore, such x is uniquely determined modulo I
1
. . . I
n
.
Corollary 6.34. Let m
1
, . . . , m
n
be integers which are pairwise relatively prime, and write
m =
n

i=1
m
i
. Then there is an isomorphism of rings Z
m

= Z
m
1
Z
m
n
. Equivalently, for any
integers a
1
, . . . , a
n
the simultaneous congruences
x a
1
mod m
1
, , x a
n
mod m
n
.
have a solution x, which is unique modulo m.
Proof. Notice that (m
i
) = m
i
Z are Z-ideals with m
1
Z . . . m
n
Z = mZ.
Remark 6.35. The proof of the Chinese Remainder Theorem gives us a method to compute
the solution of a simultaneous system of congruences. Let m
t
i
= m/m
i
be the quotient of m
by m
i
, which is relatively prime to m
i
by assumption. We can therefore nd t
i
and s
i
so that
t
i
m
t
i
+ s
i
m
i
= 1, that is t
i
is the inverse of m
t
i
modulo m
i
. Then the solution to the system of
congruences in Corollary 6.34 is
x = a
1
t
1
m
t
1
+a
2
t
2
m
t
2
+ +a
n
t
n
m
t
n
mod m,
since c
i
= t
i
m
t
i
and b
i
= s
i
m
i
.
Example 6.36. The solution of the simultaneous system of congruences
x 1 mod 8 x 2 mod 55 x 3 mod 81
is x = ... mod 35640. The simultaneous system
x 5 mod 8 x 12 mod 55 x 47 mod 81
has solution x = ... mod 35640.
As a consequence of the Chinese Remainder Theorem, we can now prove all of the properties of
the Eulers -function.
Corollary 6.37. Let m > 1 be an integer, write m = p

1
1
p

n
n
with p
1
, . . . , p
n
distinct
primes and
i
> 0. Then
(a) Z

m

=
n

i=1
(Z
p

i
i
)

.
(b) (m) =
n

i=1
(p

i
i
) = m
m

i=1
_
1
1
p
i
_
Proof. Part (a) follows from Corollary 6.34. As far as (b) is concerned, we use part (a) to prove
the rst equality. For the second equality, we use the fact that (p

i
i
) = p

i
i
p

i
1
i
= p

i
i
(11/p
i
)
by the denition of the -function.
40 6. INTRODUCTION TO RINGS
5. Prime, Primary and Maximal Ideals
Let R be a commutative ring R with identity 1 ,= 0. An R-ideal p is called a prime ideal if p ,= R
and if xy p implies x p or y p. An ideal Q is called a primary ideal if xy Q and x , Q
implies that there exists a positive integer n such that y
n
Q. An R-ideal M is called a maximal
ideal if M is a maximal proper R-ideal. The concepts of prime, primary and maximal ideal are
central to the applications of Commutative Ring Theory to Algebraic Geometry.
Proposition 6.38. Let R be a commutative ring with identity 1 ,= 0 and let I be an R-ideal.
(a) I is a prime ideal if and only if R/I is an integral domain.
(b) M is a maximal ideal if and only if R/M is a eld.
Proof. Assume that I is a prime ideal in R and let x +I and y +I be elements in R/I such
that (x +I)(y +I) = xy +I = I. Then xy I so that x I or y I. Therefore R/I has no zero
divisors, hence it is an integral domain. Conversely, assume that R/I is an integral domain. Let
x, y I with xy I. Then (x + I)(y + I) = I so that x + I = I or y + I = I. Hence x I or
y I and I is a prime ideal. This complete the equivalence in part (a). Now, the ideals I with
M I R correspond bijectively with the nonzero ideals of R/M. So M is maximal if and only
if the only ideals of R/M are 0 and R/M if and only if R/M is a eld. This proves (b).
Corollary 6.39. Every maximal ideal is a prime ideal.
Proof. Let M be a maximal R-ideal. By Proposition 6.38(b) R/M is a eld. Hence by
Proposition 6.38(a) M is a prime ideal.
Remark 6.40. Let R be a commutative ring with 1 ,= 0 and let p ,= R be an R-ideal. Then p
is a prime ideal if and only if IJ p implies I p or J p for any R-ideals I, J.
Proof. If this property holds for any R-ideals I and J then it also holds for the principal
ideals (a) and (b). Hence p is a prime ideal. Conversely, suppose that p is a prime ideal but I , p
and J , p for some ideals I, J with IJ p. Pick a I p and b J p. Then ab IJ p, which
is impossible.
We dene Spec(R), the spectrum of R, to be the set of all prime ideals of R. The maximal spectrum
of R, m-Spec(R), is dened to be the set of all maximal ideals of R. Clearly, m-Spec(R) Spec(R).
Theorem 6.41. Let R be a commutative ring R with identity 1 ,= 0. Then m-Spec(R) ,= .
Proof. Let = I [ I is an R-ideal, I ,= R. Then ,= because 0 since R ,= 0, and
is a partially ordered set
4
with I
1
I
2
if and only if I
1
I
2
. Let = I
j
be a linearly ordered
subset of ; dene J =

j
I
j
. Then J is an R-ideal, and J ,= R since 1 , I
j
for all I
j
, hence
1 , J. Thus J and I
j
J for all I
j
. It follows that every linearly ordered subset of the
nonempty set has an upper bound in . Thus has a maximal element by Zorns Lemma.
Example 6.42.
1. The prime ideals of Z are exactly the ideals nZ where n is a prime number or n = 0.
This is the motivation for the notion of prime ideal. Notice that pZ, where p > 0 a
prime, are exactly the maximal ideals in Z, as Z
p
is a eld. Instead, the prime ideal
0 is not a maximal ideal. In other words, m-Spec(Z) = pZ[ p prime number while
m-Spec(Z) = m-Spec(Z) 0.
2. The ideal (x) is a prime ideal of Z[x], since Z[x]/(x)

= Z. But (x) is not a maximal ideal
as (x) (2, x) Z[x]. However, the ideal I = (2, x) is a maximal ideal in Z[x]. Recall
that (2, x) consists of polynomials of the form f(x) = a
0
+a
1
x +. . . +a
n
x
n
where a
i
Z
and a
0
is even. Let g(x) = b
0
+ b
1
x + . . . + b
n
x
n
Z[x]. If b
0
is even then g(x) I. If
b
0
is odd then b
0
= 1 + 2t for some t Z, and g(x) = 1 + (2t + b
1
x + . . . + b
n
x
n
), where
2t + b
1
x + . . . + b
n
x
n
I. Therefore the only two cosets determined by I in Z[x] are I
and 1 +I. Thus Z[x]/I

= Z
2
so that I is a maximal ideal in Z[x].
4
A partially ordered set is a non empty set together with a binary operation satisfying: (a) reexivity, that
is a a for all a ; (b) anti-symmetry, that is if a b and b a then a = b for all a, b S; (c) transitivity, that
is if a b, b c then a c for all a, b, c R. If, in addition, for any pair (a, b) we have a b or b a then
we say that is a totally ordered set.
5. PRIME, PRIMARY AND MAXIMAL IDEALS 41
3. Let R be a commutative ring with 1 ,= 0 and G = g
1
= 1, g
2
, . . . , g
n
be a nite group.
The map
: R[G] R,
n

i=1
a
i
g
i

n

i=1
a
i
is a homomorphism of R[G] onto R, called the augmentation map. Then I = Ker() =

i
a
i
g
i
R[G] [

i
a
i
= 0 is called the augmentation ideal in R[G]. Note that g
i
g
j
I
for all i ,= j. Also, by the First Isomorphism Theorem one has R[G]/I

= R. Now assume
R = F is a eld. Then the augmentation ideal I of F[G] is a maximal ideal in F[G] since
F[G]/I

= F.
Corollary 6.43. Let R be a commutative ring with identity 1 ,= 0. Every proper R-ideal I is
contained in a maximal R-ideal.
Proof. Apply Theorem 6.41 to R/I.
Corollary 6.44. Let R be a commutative ring with identity 1 ,= 0. Then u R is a unit if
and only if u lies outside each maximal ideal of R.
Proof. Observe that u is a unit if and only if (u) = R. Thus, suppose that u is a unit and
that it is contained in a maximal ideal M. Then, we should have (u) M _ R: a contradiction.
Conversely, pick an element u outside every maximal ideal of R. If u were not a unit of R, then
(u) would be a proper ideal of R. It would follow from Corollary 6.43 that (u) M for some
maximal ideal M of R: again, a contradiction.
Let R be a commutative ring with 1 ,= 0. The Jacobson radical of R, sometimes denoted Jac(R)
is dened to be the intersection of all the maximal ideals of R.
Example 6.45. Note that Jac(Z) = (0).
Proposition 6.46. Let R be a commutative ring with 1 ,= 0 and let r R. Then r Jac(R)
if and only if for each a R the element 1 ra is a unit of R.
Proof. Suppose that r Jac(R) and let a R. If 1ra is not a unit then, by Corollary 6.43,
there exists a maximal ideal M such that 1 ra M. This leads to the contradictory fact that
1 = (1 ra) + ra M, as r Jac(R) M. Conversely, suppose that for each a R and r R
the element 1 ra is a unit. We claim that r M for each maximal ideal M of R. If not, then we
should have M M +(r) R. Hence, by the maximality of M, we deduce that M +(r) = R, so
that there exists b M and a R with b +ar = 1. Hence 1 ar M cannot be a unit of R.
A commutative ring with identity 1 ,= 0 is called a local ring if it has exactly one maximal ideal.
Corollary 6.47. Let R be a commutative ring with 1 ,= 0. If R is local then the unique
maximal ideal of R is precisely the set of non-units of R.
Proof. Apply Proposition 6.44.
Example 6.48. Let x
0
1 and let R = f : 1 1[ f rational and dened at x
0
. Then R
is a local ring with maximal ideal M = f R[ f(x
0
) = 0.
Proof. R is a commutative ring with respect to addition and multiplication of real valued
functions, and : R 1 dened by (f) = f(x
0
) is an epimorphism of rings. Thus, since 1 is
a eld, M = Ker() = f R[ f(x
0
) = 0 is a maximal ideal of R. Let h R M. Then since
h(x
0
) ,= 0 we have that 1/h R, hence h R

. Thus R M = R

, and therefore every proper


R-ideal is contained in M. Thus R is a local ring with unique maximal ideal M.
We plan to use two more times Zorns Lemma, both of which are concerned with the existence of
prime ideals. The rst can be regarded as a sharpening of Theorem 6.41. For that, we need to
introduce a new notion: A subset S of R is called a multiplicative (sub)set of R if 1 S and S is
closed under multiplication.
42 6. INTRODUCTION TO RINGS
Theorem 6.49. Let R be a commutative ring with 1 ,= 0 and let S be a multiplicative subset
of R. Suppose that there exists an R-ideal I disjoint from S, that is I S = . Then the set of
R-ideals
= J [ J R-ideal, J I, J S =
partially ordered by inclusion has at least one maximal element, and any such maximal element of
is a prime ideal of R.
Proof. Clearly, I and so ,= . Let then = J
i
be a non-empty linearly ordered
subset of . Then J =

i
J
i
is an R-ideal such that J I and J S = . Thus J is an upper
bound for in , and so it follows from Zorns Lemma that has at least one maximal element.
Let p be an arbitrary maximal element of . We claim that p is a prime ideal. Since p S =
and 1 S we see that 1 , p and p _ R. Now let a, b , p; we must show that ab , p. Since a , p
we have that I p _ p + (a). By the maximality of p in , we must have (p + (a)) S ,= , and
so there exist s S, r R and t p such that s = t +ra. Similarly, there exist s
t
S, r
t
R and
t
t
p such that s
t
= t
t
+ r
t
b. But then ss
t
= (t + ra)(t
t
+ r
t
b) = (tt
t
+ rat
t
+ r
t
bt) + rr
t
ab. Since
ss
t
S and tt
t
+rat
t
+r
t
bt p we must have that ab , p because p S = .
We now introduce the notion of the radical of an ideal: this is of great importance in Commutative
Algebra and Algebraic Geometry.
Proposition 6.50. Let I be an ideal in a commutative ring R and dene
rad
R
(I) =

I = r R[ r
n
I for some n Z
+

to be the radical
5
of I. Then

I is an R-ideal containing I and with the property that

I =

I.
Proof. Left as an exercise.
Proposition 6.51. Let Q be a primary ideal of a commutative ring R with identity 1 ,= 0.
Then

Q is a prime ideal.
Proof. Write P =

Q. Let a, b R be such that ab P and suppose that a , P. There
exists a positive integer n such that (ab)
n
= a
n
b
n
Q. But a
n
, Q. Since Q is primary, there
exists a positive integer m such that b
nm
= (b
n
)
m
Q. Therefore, b

Q = P and so P is
prime.
Let I be a proper R-ideal of a commutative ring R with identity 1 ,= 0. We dene the variety of
I, denoted Var(I) or better V (I), to be the set p Spec(R) [ p I.
Proposition 6.52. Let R be a commutative ring with identity 1 ,= 0 and let I be a proper
R-ideal. Then

I =

pV (I)
p.
Proof. Let r

I and let p V (I). Then there exists n 0 such that r
n
I p.
Since p is a prime ideal we have that r p. Thus

I

pV (I)
p. To establish the reverse
inclusion, let a

pV (I)
p and suppose that a ,

I. Our supposition means that I S = , where
S = a
n
[ n 0 is a multiplicative subset of R. Hence, by Theorem 6.49 there exists a prime
ideal Q of R such that I Q and Q S = . It follows that Q V (I) so that a Q S. With
this contradiction, the proof is complete.
We are now ready for the other application of Zorns Lemma. This one is concerned with the set
V (I). We know that if I is a proper R-ideal then V (I) ,= . Next, we establish that V (I) actually
contains a minimal member with respect inclusion.
5
If I is the zero ideal of R then

0 = |x R[ x
n
= 0 for some n Z
+
is also called the nilradical N(R) of R.
One has that N(N(R)) = N(R), that is zero is the only nilpotent element in R/N(R). Hence R/N(R) is an example
of a so-called reduced ring.
6. QUOTIENT RINGS (OR RINGS OF FRACTIONS) 43
Theorem 6.53. Let R be a commutative ring with 1 ,= 0 and let I be a proper R-ideal. Then
V (I) has at least one minimal element with respect to inclusion. Such a minimal member is called
6
a minimal prime ideal of I or a minimal prime ideal containing I.
Proof. One nice aspect of the use of Zorns Lemma here is that V (I) is regarded as a partially
ordered set by reverse inclusion, that is for P
1
, P
2
V (I) we have P
1
_ P
2
if and only if P
1
P
2
.
Thus a maximal member of this partially ordered set is just a minimal member of V (I) with
respect to inclusion. Let = P
i
be a linearly ordered (with respect to reverse inclusion) subset
of V (I). Then P =

i
P
i
is a proper R-ideal containing I. We claim that P Spec(R). Let
a R P and b R such that ab P. Since a , P, there exists P
k
such that a , P
k
. We
must show that b P, that is b P
i
for any P
i
. Since is totally ordered, either P
k
P
i
or
P
i
P
k
. In the rst case, the facts that ab P P
k
and a , P
k
imply that b P
k
P
i
; in the
second case, we must have a , P
i
and ab P
i
, so that b P
i
. Thus b P
i
in any event, and since
P
i
was any arbitrary member of , it follows that b P. Thus P V (I) and P is an upper bound
for in our partially ordered set, that is P
i
_ P for any P
i
. We now use Zorns Lemma to
complete the proof.
Corollary 6.54. Let R be a commutative ring with identity 1 ,= 0. Let I be a proper R-ideal
and let Min(I) denote the set of minimal prime ideals of I. Then

I =

pMin(I)
p.
Proof. Left as an exercise.
Lemma 6.55. Let R be a commutative ring with 1 ,= 0 and let I, J and I
j
be R-ideals.
(a) V (I) = V (

I).
(b) V (IJ) = V (I) V (J).
(c) V
_
_
j
I
j
_
=

j
V (I
j
).
(d) V (I) V (J) if and only if

I

J.
Proof. Since the radical of an ideal I is the intersection of the prime ideals containing I,
it is clear that V (I) = V

I), as asserted in (a). Certainly if p I or p J, then p IJ.


Thus V (IJ) V (I) V (J). Conversely, if p IJ then p I or p J by Remark 6.40. Thus
V (IJ) V (I) V (J), and the equality asserted in (b) is established. Observe that p contains

j
I
j
if and only if p contains each I
j
, simply because

j
I
j
is the smallest ideal containing all
of the ideals I
j
. This proves (c). By Proposition 6.52, the radical of I is the intersection of the
set of all prime ideals containing I. So

I

J if and only if V (I) V (J). Thus (d) is also
established.
6. Quotient Rings (or Rings of Fractions)
Here R will always be a commutative ring with 1 ,= 0. Let S be a multiplicative subset
7
of R.
We wish to construct the smallest ring in which the elements of S become units (like the passage
from Z to ).
On the set R S dene a relation by saying that (r, s) (r
t
, s
t
) if and only if there exists
s
1
S with s
1
(s
t
r sr
t
) = 0. This is an equivalence relation. Indeed, (r, s) (r, s), as 1 S;
(r, s) (r
t
, s
t
) implies that (r
t
, s
t
) (r, s), as 0 = 0; nally, (r, s) (r
t
, s
t
) and (r
t
, s
t
) (r
tt
, s
tt
)
implies
8
(r, s) (r
tt
, s
tt
), as S is closed under multiplication. Write
r
s
for the class of (r, s) and write
S
1
R = R
S
= R S/ =
r
s
[ r R, s S. On S
1
R dene an addition and multiplication
9
by
r
s
+
r
t
s
t
=
s
t
r +sr
t
ss
t
r
s
r
t
s
t
=
rr
t
ss
t
.
6
The minimal primes of the zero ideal 0 of R are sometimes referred to as the minimal prime ideals of R.
7
For example, Z \ |0 is a multiplicative subset of Z.
8
We have s
1
(s

r sr

) = 0 and s
2
(s

) = 0 for some s
1
, s
2
S. Multiply the rst equation by s
2
s

and
the second one by s
1
s. Then after adding and simplifying we obtain s
1
s
2
s

(s

r sr

) = 0.
9
Notice that
rs

ss

=
r
s
and
r
s
=
r
1
(
s
1
)
1
.
44 6. INTRODUCTION TO RINGS
S
1
R will be called the quotient ring (ring of fractions) of R with respect to S.
Theorem 6.56.
(a) S
1
R is a commutative ring with
0
1
as zero and
1
1
as one.
(b) The map
S
: R S
1
R, dened by
S
(r) =
r
1
, is a homomorphism of rings.
(c)
S
(S) (S
1
R)

.
Proof. As for (a) we only check that the addition is well dened. The other properties are
left as an exercise. Assume that
r
s
=
r
1
s
1
and
r

=
r

1
s

1
. Then s
2
(sr
1
s
1
r) = 0 and s
3
(s
t
r
t
1
s
t
1
r
t
) = 0
for some s
2
, s
3
S. Multiply the rst equation by s
3
s
t
s
t
1
and the second one by s
2
ss
1
. Then,
after adding, we obtain s
2
s
3
[s
t
s
t
1
(sr
1
s
1
r) + ss
1
(s
t
r
t
1
s
t
1
r
t
)] = 0, that can be rearranged into
s
2
s
3
[ss
t
(s
t
1
r
1
+s
1
r
t
1
) s
1
s
t
1
(s
t
r +sr
t
)] = 0 so that
s
t
r +sr
t
ss
t
=
s
t
1
r
1
+s
1
r
t
1
s
1
s
t
1
.
Now,
S
(r + r
t
) =
r+r

1
=
r
1
+
r

1
=
S
(r) +
S
(r
t
) and
S
(rr
t
) =
rr

1
=
r
1
r

1
=
S
(r)
S
(r
t
) and

S
(1) =
1
1
. This proves (b). Finally,
s
1
1
s
=
s
s
=
1
1
. Hence
s
1
(S
1
R)

for every s S. This


completes the proof of (c).
Theorem 6.57. Let S be a multiplicative subset of R. For every homomorphism f : R T of
commutative rings with f(S) T

, there exists a unique homomorphism of rings f : S


1
R T
such that f
S
= f. The property uniquely determines S
1
R up to isomorphism.
Proof. Dene f(
r
s
) = f(r)(f(s))
1
, as f(s) T

. We rst show that f is well-dened.


Indeed, let
r
s
=
r

or s
1
(s
t
rsr
t
) = 0 for some s
1
S. Thus f(s
1
)(f(s
t
)f(r)f(s)f(r
t
)) = 0, hence
f(s
t
)f(r)f(s)f(r
t
) = 0 since f(s
1
) is a unit. Hence f(
r
s
) = f(r)(f(s))
1
= f(r
t
)(f(s
t
))
1
= f(
r

).
Next, we show that f is a homomorphism. Indeed, f(
r
s
+
r

) = f(
s

r+sr

ss

) = f(s
t
r+sr
t
)(f(ss
t
))
1
=
f(r)(f(s))
1
+ f(r
t
)(f(s
t
))
1
= f(
r
s
) + f(
r

); similarly, we obtain that f(


r
s
r

) = f(
r
s
)f(
r

) and
f(
1
1
) = f(1)(f(1))
1
= 1. Obviously, f
S
= f. To prove the uniqueness, let f
t
be a second
map. Then for every
r
s
S
1
R we have that f(s)f(
r
s
) = f(
s
1
)f(
r
s
) = f(
r
1
) = f(r) = f
t
(
r
1
) =
f
t
(
s
1
)f
t
(
r
s
) = f(s)f
t
(
r
s
). Now f(s)f(
r
s
) = f(s)f
t
(
r
s
) implies f(
r
s
) = f
t
(
r
s
) since f(s) is a unit.
Proposition 6.58.
(a) Ker(
S
) = r [ sr = 0 for some s S.
(b) S
1
R ,= 0 if and only if 0 , S.
(c)
S
is a monomorphism if and only if S contains no zero divisors.
Proof. We know that Ker(
S
) = r R[
r
1
=
0
1
= r R[ sr = 0 for some s S.
This gives (a). Observe that S
1
R = 0 if an only if
S
= 0. This is equivalent to asking that
1 Ker(
S
) or that s = 0 for all s S. This proves (b). Finally,
S
is a monomorphism if and
only if Ker(
S
) = 0. But this is equivalent to asking that S contains no zero divisors, hence (c) is
established.
Example 6.59.
(1) Let S = non zero divisors of R = R zero divisors of R. Then S is a multiplicative
subset of R: notice that 1 S and s
1
s
2
x = 0 implies s
2
x = 0 which implies x = 0.
Write Quot(R) = S
1
R and call this the total ring of quotients of R. Notice that
S
is a
monomorphism by Proposition 6.58(c), hence R Quot(R).
(2) If R is a domain, then Quot(R) is a eld, called the quotient eld (eld of fractions) of
R. For instance, = Quot(Z).
Proof. In this situation we have S = R0. Let 0 ,=
r
s
S
1
R. Then r ,= 0, hence
r S and
s
r
S
1
R.
(3) Let x R and set S = x
n
[ n 0. Then S is a multiplicative subset. Write R
x
= S
1
R.
7. AFFINE SCHEMES 45
(4) Let p be a prime ideal and set S = R p. Then S is a multiplicative subset of R, as 1 , p
and s
1
, s
2
, p implies that s
1
s
2
, p. Write R
p
= S
1
R and call this the localization of R
at p. For example, let p be a prime number. Then Z
(p)
=
a
b
[ a, b Z, p [ b .
Proposition 6.60. Let i : R K be a monomorphism, where K is a eld
10
and let 0 , S be
a multiplicative subset of R. Then by Theorem 6.57, there exists a homomorphism : R
S
K
with
S
= i. This is a monomorphism (in particular, Quot(R) K).
Proof. We need to show that Ker() = 0. So let
r
s
Ker(). Then 0 = i(s)(
r
s
) = (
r
1
) = i(r).
Thus r = 0, hence
r
s
= 0.
Corollary 6.61. Let 0 , S be a multiplicative subset of a domain R. Then S
1
R is a domain.
Proof. By Example 6.59 we have that R K = Quot(R), where the latter is a eld. By
Proposition 6.60, S
1
R K, thus S
1
R is a domain.
Let I be an R-ideal, S a multiplicative subset of R, q Spec(R). Write S
1
I = (S
1
R)I and
I
q
= R
q
I and notice that S
1
I =
a
s
[ a I, s S.
Theorem 6.62. Let S be a multiplicative subset of a commutative ring R, J an S
1
R-ideal
and p a prime ideal of R.
(a) S
1
(J R) = J.
(b) If p S = , then S
1
p is a prime ideal of S
1
R and (S
1
p) R = p.
Proof. Notice that J R is the contraction of J to R, that is J R =
1
S
(J). Now,
S
1
(J R) = S
1
(
S
(
1
S
(J))) J since J is an S
1
R-ideal. Conversely, let
r
s
J. Then
r
1
=
s
1
r
s
J, hence r J R, hence
r
s
S
1
(J R). This proves (a). As far as (b) is
concerned, we rst show that (S
1
p) R = p. Notice that (S
1
p) R =
1
S
(S
1

S
(p)) p.
Conversely, let a be in (S
1
p) R. Then a R with
a
1
S
1
p, hence
a
1
=
b
s
for some b p
and s S. Thus s
1
sa = s
1
b for some s
1
S. Thus s
2
a p for some s
2
S. But s
2
, p since
S p = , thus a p since s
2
a p and p is prime. Suppose now that S
1
p = S
1
R. Then
p = (S
1
p) R = (S
1
R) R = R, which is impossible. Hence S
1
p is a proper ideal of S
1
R.
Finally, suppose that
r
s
r

S
1
p. Then
rr

1
=
ss

1
r
s
r

S
1
p so that rr
t
(S
1
p) R = p. Now
since p is a prime ideal of R one has that r p or r
t
p. Thus
r
s
or
r

is in S
1
p. Hence S
1
p is
a prime ideal.
Theorem 6.63. There is a one-to-one inclusion preserving correspondence between Spec(S
1
R)
and p Spec(R) [ p S = , given by q q R and S
1
p . p.
Proof. If q Spec(S
1
R) then q ,= S
1
R, hence
1
S
(q) S = since otherwise q would
contain a unit in S
1
R. The rest follows from Theorem 6.62(a,b).
Corollary 6.64. For q Spec(R), the correspondence of Theorem 6.63 gives a one-to-one
correspondence between Spec(R
q
) and p Spec(R) [ p q. In particular, R
q
is a local ring with
maximal ideal qR
q
. (This motivates the term localization.)
Proof. p (R q) = is and only if p q. Now use Theorem 6.63.
On the other hand, notice that there is a one-to-one correspondence between Spec(R/q) and
p Spec(R) [ p q.
Example 6.65. Let p be a prime number, n > 0.
(1) Z
(p)
is a local ring with Spec(Z
(p)
) = 0, pZ
(p)
.
(2) Z/(p
n
) is a local ring with Spec(Z/(p
n
)) = (p)/(p
n
).
7. Ane Schemes
The spectrum of a ring, equipped with its Zariski topology and the structure sheaf O, is what
Grothendieck called an ane scheme. The theory of schemes revolutionized Algebraic Geometry
and Grothendieck was awarded a Fields medal in 1966 for this huge body of work.
10
Thus, R is a domain!
46 6. INTRODUCTION TO RINGS
The Zariski Topology on Spec(R). Let R be a commutative ring with 1 ,= 0. We dene a
point of Spec(R) to be a prime ideal of R. To avoid confusion sometimes we write [p] for the point
of Spec(R) corresponding to the prime p of R. We will adopt the usual convention that R itself is
not a prime ideal. Of course, the zero ideal (0) is a prime if R is a domain.
Now we can equip Spec(R) with a topology: the Zariski topology
11
. We take the subsets of the
form V (I), where I is an R-ideal, to be the closed subsets. Note that V (R) = , V (0) = Spec(R)
and Lemma 6.55 shows that nite unions and arbitrary intersections of sets of the form V (I) are
again of that form. Hence they do form the set of closed sets for a topology on Spec(R). The
closed subset V (I) in Spec(R) is irreducible
12
if and only if

I is a prime ideal. An open set in the
Zariski topology is simply the complement of one of the sets V (I). The open sets corresponding
to principal ideals play a special role, essentially because they are again the spectra of rings. If
f R, we dene the basic (or distinguished) open subset of X = Spec(R) associated with f to be
X
f
= Spec(R) V (f) = p Spec(R) [ f , p.
Proposition 6.66. Let f R and let X
f
be the corresponding basic open set in X = Spec(R).
(a) X
f
= X if and only if f is a unit and X
f
= if and only if f is nilpotent.
(b) X
f
X
g
= X
fg
. In particular, any basic open set that is a subset of the basic open set
X
f
has the form X
fg
for suitable g.
(c) X
f
X
g
1
. . .X
g
n
if and only if f

(g
1
, . . . , g
n
). X
f
= X
g
if and only if

(f) =

(g).
(d) The basic open sets form a basis for the Zarisky topology on X.
(e) There exists an homeomorphism X
f

= Spec(R
f
), where R
f
is the localization of R at f.
(f) The spectrum of any ring is quasicompact
13
; in particular X
f
is quasicompact.
Proof. We leave (a) and (b) as exercises. As far as (c) is concerned, we note that X
g
1
. . .X
g
n
consists of the prime ideals p not containing at least one of g
1
, . . . , g
n
. Hence X
g
1
. . . X
g
n
is
the complement of the closed set V (g
1
, . . . , g
n
) consisting of the primes p that contain the ideal
generated by g
1
, . . . , g
n
. If (g
1
, . . . , g
n
) = R then X
g
1
. . . X
g
n
= X and there is nothing
to prove. Otherwise, X
f
X
g
1
. . . X
g
n
if and only if every prime p with f , p also satises
p , V ((g
1
, . . . , g
n
). This latter condition is equivalent to the statement that if the prime p contains
the ideal (g
1
, . . . , g
n
) then p also contains f, i.e., f is contained in the intersection of all the primes
p containing (g
1
, . . . , g
n
). Since this latter set is

(g
1
, . . . , g
n
), this proves (c). If U = X V (I) is
a Zariski open subset of X = Spec(R), then U is the union of the sets X
f
with f I, which proves
(d). The natural ring homomorphism from R to the localization R
f
establishes a bijection between
the prime ideals in R
f
and the prime ideals in R not containing (f). The corresponding Zariski
continuous map from Spec(R
f
) to Spec(R) is therefore continuous and bijective. Since every ideal
of R
f
is the extension of some ideal of R, it follows that the inverse map is also continuous, which
proves (e). In (f), every open set is the union of principal open sets by (d), so it suces to prove
that if X is covered by principal open sets X
g
i
then X is a nite union of some of the X
g
i
. If
the ideal I generated by the g
i
were a proper ideal in R, then I would be contained in some
maximal ideal M. But in this case the element M in X = Spec(R) would not be contained in
any principal open set X
g
i
, contradicting the assumption that X is covered by the X
g
i
. Hence
I = R and so 1 R can be written as a nite sum 1 = a
1
g
i
1
+ + a
n
g
i
n
. Consider the nite
union X
g
i
1
. . . X
g
i
n
. Any point p in X not contained in this union would be a prime in R that
contains g
i
1
, . . . , g
i
n
, hence would contain 1, a contradiction. It follows that X = X
g
i
1
. . . X
g
i
n
as needed. The second part of (f) follows from (e).
Let : R S be a ring homomorphism mapping 1
R
to 1
S
. For P Spec(S), the ideal P R =

1
(P) is a point of Spec(R). If

(P) = p, that is P R = p, we say that P lies over p. Write

: Y = Spec(S) X = Spec(R) to denote the map sending P

(P) = P R. One easily


11
Such topologies were rst introduced by M.H. Stone for Boolean rings and were considered by N. Jacobson
for the primitive ideals of an arbitrary ring.
12
A closed subset of a topological space is irreducible if it is not the union of two proper closed subsets, or,
equivalently, if every nonempty open set is dense.
13
Every open cover has a nite subcover. The quasi is there because the space is not necessarily Hausdor. In
contrast with the usual situation, the continuous image of an ane scheme need not be closed. A better notion than
the one of compactness is the one of properness.
7. AFFINE SCHEMES 47
sees that (

)
1
(V (I)) = V ((I)S), so that

is continuous. Moreover, if f R and X


f
denotes
the corresponding basic set in X we have that (

)
1
(X
f
) = Y
(f)
. If : S T is another ring
homomorphism then obviously ( )

. Hence the correspondence R Spec(R) and


denes a controvariant functor from the category of rings to the category of topological
spaces. Notice that if M m-Spec(S) then it is not necessarily the case that

(M) m-Spec(R).
For instance, let : Z and take M = 0, the zero ideal of . Then

(M) = 0, which is
not a maximal ideal of Z. Thus the correspondence R m-Spec(R) is not functorial. This is one
reason for thinking of Spec(R) as more important that m-Spec(R). On the other hand, one could
say that Spec(R) contains too many points. The only points of Spec(R) that are closed are those
corresponding to maximal ideals of R. In general, it is clear that the smallest closed set containing
a given point [p] must be V (p). So that Spec(R) is almost never a Hausdor space: the open sets
are simply too large!
Next, we shall show that Spec(R) is disconnected
14
if and only if R contains an idempotent ,= 0, 1.
This will follow from a considerably stronger result, which gives a bijection of the set of idempotents
of R and the set of open and closed subsets of Spec(R). To obtain this we shall need the following
result on lifting of idempotents.
Lemma 6.67. Let R be a ring that is not necessarily commutative with 1 ,= 0, N a nil ideal in
R, and u = u + N an idempotent element of R = R/N. Then there exists an idempotent e in R
such that e = u. Moreover, e is unique if R is commutative.
Proof.
If e and f are idempotents in R, then so are e
t
= 1e, ef, and e f = 1(1e)(1f) = e+f ef.
It is readily veried that the set E of idempotents of R is a Boolean algebra with the composition
e f = ef and e f = e f.
Theorem 6.68. If e is an idempotent in R, then X
e
is an open and closed subspace of X =
Spec(R) and the map e X
e
is an isomorphism of the Boolean algebra E onto the Boolean algebra
of open and closed subsets of X.
Proof.
Corollary 6.69. Spec(R) is connected if and only if R contains no idempotents ,= 0, 1.
The Structure Sheaf on Spec(R). For each prime ideal p R, let R
p
be the localization
of R at p. Let U be a Zariski open subset of Spec(R). If U = we dene O(U) = 0; otherwise,
we dene O(U) to be the set of functions s: U

pU
R
p
from U to the disjoint union of the
localizations R
p
for p U such that
i. s(p) R
p
for each p U;
ii. for each p U there is a neighborhood X
f
of p, contained in U, and elements a/f
n
R
f
,
such that s(q) = a/f
n
R
q
for each q X
f
.
Now it is clear that sums and products of such functions are again such, and that the element 1
which gives 1 in each R
p
is an identity. Thus O(U) is a commutative ring with identity. If V U
are two open sets, the natural restriction map O(U) O(V ) is a homomorphism of rings. It is
then clear that O is a presheaf. Finally, it is clear from the local nature of the denition that O is
a sheaf. More precisely, the collection of rings O(U) for the Zariski open sets of X together with
the restriction maps O(U) O(V ) for V U is called the structure sheaf on X, and is denoted
simply by O, or O
X
. The elements s O(U) are called the sections of O over U. The elements
of (O)(X) are called the global sections of O.
Theorem 6.70. Let R be a ring and consider the ane scheme (X = Spec(R), O
X
). For any
element f R the ring O(X
f
)

= R
f
. In particular O(X)

= R.
Proof. Suppose that a/f
n
is an element of the localization R
f
. Then the map dened by
s(q) = a/f
n
R
q
for q X
f
gives an element in O(X
f
) and it is immediate that the resulting
map from R
f
to O(X
f
) is a ring homomorphism. Suppose that a/f
n
= b/f
m
in R
q
for every
14
A space X is disconnected if it contains an open and closed subset ,= , ,= X.
48 6. INTRODUCTION TO RINGS
q X
f
, i.e., g(af
m
bf
n
) = 0 in R for some g , q. If I is the ideal in R of elements r R with
r(af
m
bf
n
) = 0, it follows from g I that I is not contained in q for any q X
f
. Put another
way, every prime ideal of R containing I also contains f. Hence f is contained in the intersection
of all the prime ideals of R containing I, which is to say that f

I. Then f
N
I for some
integer N 0, and so f
N
(af
m
bf
n
) = 0 in R. But this shows that a/f
n
= b/f
m
in R
f
and so
the map is injective. Suppose now that s O(X
f
). Then by denition X
f
can be covered by
basic open sets X
g
i
on which s(q) = a
i
/g
n
i
i
R
q
for every q X
g
i
. By Proposition 6.66(f), we may
take a nite number of the g
i
and then by taking dierent a
i
we may assume all the n
i
are equal
(since a
i
/g
n
i
i
= (a
i
g
nn
i
i
)/g
n
i
if n is the maximum of the n
i
). Since s(q) = a
i
/g
n
i
= a
j
/g
n
j
in R
q
for
all q X
g
i
g
j
= X
g
i
X
g
j
, the injectivity of (applied to R
g
i
g
j
) shows that a
i
/g
n
i
= a
j
/g
n
j
in R
g
i
g
j
.
This means that g
i
g
N
j
(a
i
g
n
j
a
j
g
n
i
) = 0, i.e., a
i
g
N
i
g
n+N
j
= a
j
g
n+N
i
g
N
j
in R for some N 0, and
we may assume N suciently large that this holds for every i and j. Since X
f
is the union of the
X
g
i
= X
g
n+N
i
, f is contained in the radical of the ideal generated by the g
n
i
by Proposition 6.66(c),
say f
M
=

i
b
i
g
n+N
i
for some M 1 and b
i
R. Dene a =

i
b
i
a
i
g
N
i
R. Then
g
N
j
a
j
f
M
=

i
b
i
(a
j
g
n+N
i
g
N
j
) =

i
b
i
(a
i
g
N
i
g
n+N
j
) = g
n+N
j
a.
It follows that a/f
M
= a
j
/g
n
j
in R
g
j
, and so the element in O(X
f
) dened by a/f
M
in R
f
agrees
with s on every X
g
j
, and so on all of X
f
since these open sets cover X
f
. Hence the map gives
an isomorphism R
f

= O(X
f
). Taking f = 1 gives R

= O(R), completing the proof.
7. AFFINE SCHEMES 49
Homework set # 6.
Solve 5 of the following problems, with the following requirement. The rst problem must be
chosen among the problems numbered 1.-4.; the second one among those numbered 5.-8.; the
third one among those numbered 9.-12.; the fourth one among those numbered 13.-16.; the fth
one among those numbered 17.-20.
The due date is on November 30 (Friday), 2001.
1. Prove that if R is an integral domain and x
2
= 1 for some x R then x = 1.
2. Prove that any subring of a eld which contains the identity is an integral domain.
3. A ring R is called a Boolean ring if a
2
= a for all a R.
Prove that every Boolean ring is commutative.
Prove that the only Boolean ring that is an integral domain is Z
2
.
4. The center Z(R) of a ring R is z R[ zr = rz for all r R.
Prove that the center of a ring is a subring that contains the identity if R does.
Prove that the center of a division ring is a eld.
5. Let x be a nilpotent element of a commutative ring R with 1 ,= 0, that is x
m
= 0 for some
m Z
+
.
Prove that x is either zero or a zero divisor.
Prove that rx is nilpotent for all r R.
Prove that 1 +x is a unit in R.
Prove that the sum of a nilpotent element and a unit is a unit.
6. Let K be a eld. A discrete valuation on K is a function : K

Z satisfying
(i) (xy) = (x) +(y) for all x, y K

;
(ii) is surjective;
(iii) (x +y) min((x), (y)) for all x, y K

with x +y ,= 0.
The set R = x K

[ (x) 0 0 is called the valuation ring of .


Prove that R is a subring of K which contains the identity.
Prove that for each nonzero element x K either x or x
1
is in R.
Prove that an element x is a unit of R if and only if (x) = 0.
7. Let R be a ring with more than one element such that for each non zero a R there is a
unique b R such that aba = a. Prove that:
R has no zero divisors.
bab = b.
R has an identity.
R is a division ring.
8. Dene the set R[[x]] of formal power series in the indeterminate x with coecients from
R to be all formal innite sums

n=0
a
n
x
n
= a
0
+a
1
x +a
2
x
2
+
Dene addition and multiplication of power series in the same way as for power series
with real or complex coecients. (Formal indicates that convergence is not considered.)
Show that 1 x is a unit in R[[x]].
Prove that

n=0
a
n
x
n
is a unit in R[[x]] if and only if a
0
is a unit in R.
Prove that if R is an integral domain then the ring of formal power series R[[x]] is
also an integral domain.
9. Let : R S be a homomorphism of commutative rings.
Prove that if P is a prime ideal of S then either
1
(P) = R or
1
(R) is a prime
ideal of R. In particular, if R is a subring of S and is the inclusion homomorphism
deduce that if P is a prime ideal of S then P R = R or P R is a prime ideal of R.
Prove that if M is a maximal ideal of S and is surjective then
1
(M) is a maximal
ideal of R. This need not be the case if is not surjective.
10. Find all ring homomorphisms from Z to Z
30
. In each case describe the kernel and the
image.
50 6. INTRODUCTION TO RINGS
11. The Prime Avoidance Theorem: Let P
1
, . . . , P
n
, where n 2, be ideals of the commutative
ring R such that at most 2 of P
1
, . . . , P
n
are not prime ideals. Let I be an R-ideal such
that
I
n
_
i=1
P
i
.
Then show that I P
i
for some i with 1 i n. In particular, if for each i = 1, . . . , n we
have that I , P
i
, then there exists c I

i
P
i
so that c avoids all the ideals P
1
, . . . , P
n
,
most of which are prime ideals.
12. Let f : R S be a homomorphism of commutative rings. Then we have an induced map
a
f : Spec(S) Spec(R) given by
a
f(q) = q R, where q R means f
1
(q). Notice
that if M m-Spec(S) then it is not necessarily the case that f
1
(M) m-Spec(R). For
instance, let i : Z and take M = 0, the zero ideal of . Then i
1
(M) = 0, which is
not a maximal ideal of Z.
13. Let I, J and K be ideals of R.
Prove that IJ is an ideal contained in I J.
Prove that if R is commutative and if I +J = R then IJ = I J.
Prove that I(J +K) = IJ +IK and (I +J)K = IK +JK.
Prove that if J I then I (J +K) = J + (I K).
14. Let R and S be nonzero rings with identity 1
R
and 1
S
respectively. Let : R S be
a nonzero homomorphism of rings.
If S is an integral domain prove that if (1
R
) = 1
S
.
Prove that if (1
R
) = 1
S
then (u) is a unit in S and (u
1
) = (u)
1
for each unit
u of R.
15. Let R be an integral domain. Prove that (a) = (b) for some elements a, b R if and only
if a = ub for some unit u of R.
16. Let x
2
+x + 1 be an element of the polynomial ring E = F
2
[x] and use the bar notation
to denote passage to the quotient ring F
2
[x]/(x
2
+x + 1).
Prove that E has 4 elements: 0, 1, x, 1 +x.
Prove that the additive group E is isomorphic to the Klein 4-group.
Prove that E

is isomorphic to the cyclic group of order 3.


Deduce that E is a eld.
17. Assume R is commutative. Prove that if P is a prime ideal of R and P contains no zero
divisors then R is an integral domain.
18. Assume R is commutative. Let I and J be ideals of R and assume P is a prime ideal of
R that contains I J. Prove that either I or J is contained in P.
19. Let R be a nite commutative ring with identity. Prove that every prime ideal of R is a
maximal ideal.
20. Let R be a Boolean ring.
Prove that a nonzero Boolean ring has characteristic 2.
Prove that every prime ideal of R is a maximal ideal.
Prove that every nitely generated ideal of R is principal.
CHAPTER 7
Factorization in Rings
1. UFDs, PIDs and Euclidean Domains
In this section R will be an integral domain with 1 ,= 0. Let a and b be elements in R. We say
that a divides b, and write a[b, if b = ca for some c R. We say that a and b are associates, and
write a b, if b = ua for u R

.
Remark 7.1.
1. a[b if and only if (b) (a).
2. a b if and only if a[b and b[a if and only if (a) = (b).
An element c R is called irreducible in R if c ,= 0, c , 1 and c = ab with a, b R implies a 1
or b 1. In other words, c is a nonzero element of R which is not a unit in R and whenever c = ab
then at least one of a or b must be a unit in R.
An element p R is called a prime (element) if p ,= 0, p , 1 and whenever p[ab with a, b R then
either p[a or p[b.
Example 7.2. Let n Z. Then n is irreducible if and only if n is a prime element if and only
if [n[ is a prime number.
Proposition 7.3. Let c, p be elements of an integral domain R.
(a) p is a prime element if and only if p ,= 0 and (p) is a prime ideal.
(b) c is irreducible if and only if c ,= 0 and (c) is maximal in the set of all proper principal
R-ideals.
(c) Every prime element is irreducible.
(d) If R is a principal ideal domain (PID), then every irreducible element is prime.
Proof. An element p is prime if and only if p ,= 0, p , 1 and whenever p[ab with ab R then
either p[a or p[b. This is equivalent to saying that p ,= 0, (p) ,= R, and ab (p) implies that a (p)
or b (p). Hence this is equivalent to p ,= 0 and (p) is a prime ideal. This proves property (a).
Let us suppose now that c is irreducible. This is equivalent to saying that c ,= 0, c , 1, a[c implies
a 1 or a c. This is equivalent to saying that c ,= 0, (c) ,= R, and (c) (a) implies (a) = R or
(a) = (c). This is equivalent to saying that c ,= 0 and (c) is a maximal proper principal ideal. This
establishes property (b). Suppose now that p is a prime element. Then p ,= 0 and p , 1. Further
suppose p = ab. Then p[a or p[b. Assume that p[b. Then b = tp for some t R. Thus p = atp
and so (1 at)p = 0. Therefore 1 = at, or a 1. Thus p is irreducible and property (c) is proved.
Finally, let R be a PID and let c be an irreducible element. Then by property (b), c ,= 0 and (c)
is a maximal proper principal ideal, hence c ,= 0 and (c) is a maximal proper ideal (since R is a
PID). Hence c ,= 0 and (c) is a prime ideal, hence c is a prime element by part (a) and this yields
the proof of (d).
A ring R is called a unique factorization domain (UFD or factorial) if every a R, a ,= 0, a , 1
can be written as a product of prime elements.
Proposition 7.4. Prime factorizations are automatically unique in the following sense: Let
a p
1
p
n
and a q
1
q
m
, where p
i
and q
i
are prime elements. Then n = m and after
changing the order of the factors, if needed, p
i
q
i
for all i.
Proof. We may assume n m and induct on m, the assertion being trivial for m = 0. We
may assume m > 0 and then n > 0. Now q
m
[a = p
1
p
n
, hence q
m
[p
i
for some i (since q
m
is prime), hence q
m
p
i
(since p
i
is prime hence irreducible and q
m
, 1). Now after changing
51
52 7. FACTORIZATION IN RINGS
the order of the factors, q
m
p
n
. Thus p
1
p
n1
q
1
q
m1
and we are done by induction
hypothesis.
Proposition 7.5. R is a UFD if and only if the following two conditions hold:
(a) every a R such that a ,= 0 and a , 1 can be written as a product of irreducible elements;
(b) every irreducible element of R is a prime element.
Proof. If the two conditions holds then R is clearly a UFD, by denition. Conversely, we only
need to verify property (b). So let c be irreducible. Since c ,= 0, c , 1, c has a prime factorization
c = p
1
p
n
, p
i
prime. Since c is irreducible, n = 1 and thus c = p
1
is prime.
R is said to satisfy the ascending chain condition (acc) for principal ideals if for every chain of
principal ideals (a
1
) (a
2
) there exists n with (a
i
) = (a
i+1
) for i n.
Proposition 7.6. If R satises acc for principal ideals then for every a R, a ,= 0, a , 1,
can be written as a product of irreducible elements.
Proof. Suppose the assertion is false, then
S = (a) [ (a) ,= 0, (a) ,= R, a not a product of irreducibles
is not empty. If S has no maximal element, then one can construct inductively a strictly increasing
chain in S, (a
1
) _ (a
2
) _ , which is impossible by our assumption on R. Thus S has a maximal
element (c). Since c S, c cannot be irreducible, hence since c ,= 0, c , 1, we have c = ab with
a , 1 and b , 1, or equivalently, (c) _ (a) _ R and (c) _ (b) _ R. Now by the maximality of (c)
in S, a and b are products of irreducibles, and hence so is c = ab. This is a contradiction.
Proposition 7.7. Every Noetherian ring
1
satises acc for principal ideals.
Proof. Let (a
1
) (a
2
) be an ascending chain of principal ideals, and set I =
_
i
(a
i
).
Then I is an ideal, hence nitely generated. Write I = (f
1
, . . . , f
s
). Now f
j
(a
ij
); hence f
j
(a
n
)
for all j, for some n. Thus I (a
n
), which gives (a
i
) = (a
n
) for i n.
Theorem 7.8. Let R be a Noetherian domain. Then R is a UFD if and only if every irreducible
element of R is prime.
Proof. Use Proposition 7.7, Proposition 7.6 and Proposition 7.5.
Corollary 7.9. Every PID is a UFD.
Proof. Use Theorem 7.8 and Proposition 7.3(d).
Let a and be elements of R. An element d R is called greatest common divisor of a and b if
d[a and d[b, whenever c[a and c[b then c[d. If d exists, then d is unique up to associates. Write
d gcd(a, b). An element m R is called least common multiple of a and b if a[m and b[m, and
whenever a[c and b[c, then m[c. If m exists, then it is unique up to associates. Write m lcm(a, b).
We say that a and b are relatively prime if gcd(a, b) 1.
Proposition 7.10. Let a, b be in R.
(a) If R is a UFD the gcd(a, b) and lcm(a, b) exist: For nonzero nonunits a, b write a
p

1
1
p

n
n
and b p

1
1
p

n
n
with
i
0,
i
0 and p
1
, . . . , p
n
non associate primes;
then gcd(a, b) p

1
1
p

n
n
and lcm(a, b) p

1
1
p

n
n
with
i
= min
i
,
i
and
i
=
max
i
,
i
.
(b) If R is a PID then (a) + (b) = (gcd) and (a) (b) = (lcm).
Proof. For the proof of the second statement, the ideal generated by the gcd is the smallest
principal ideal containing (a) and (b), while the ideal generated by the lcm is the largest principal
ideal contained in (a) and (b).
1
In particular, any PID satises the ascending chain condition on ideals.
1. UFDS, PIDS AND EUCLIDEAN DOMAINS 53
An integral domain R is called a Euclidean domain if there exists a function N: R 0 N,
called a norm on R, such that
i. if for a ,= 0 and b ,= 0, a[b then N(a) N(b);
ii. if b ,= 0, then there exists q and r in R with a = qb + r where either r = 0 or else
N(r) < N(b). Here q is the quotient and r is the remainder.
Theorem 7.11. Every Euclidean domain is a PID. Hence a UFD.
Proof. Let I be an R-ideal. We may assume I ,= 0. Thus N(I 0) = N(a) [ a I, a ,= 0
is a nonempty subset of N, and hence has a smallest element, say N(b) with b I 0. We claim
that I = (b). So let a I, and write a = qb + r as above. If r ,= 0, then N(r) < N(b). But
r = a qb I 0. This contradicts the minimality of N(b). Thus r = 0, hence a (b).
Remark 7.12 (Euclidean Algorithm). The Division Algorithm can be used to nd gcd(a, b).
Proof. If a, b R and b ,= 0 by a repeated use of the Division Algorithm we have
a = q
0
b +r
1
with r
1
= 0 or N(r
1
) < N(b);
b = q
1
r
1
+r
2
with r
2
= 0 or N(r
2
) < N(r
1
);
r
1
= q
2
r
2
+r
3
with r
3
= 0 or N(r
3
) < N(r
2
);

r
n2
= q
n1
r
n1
+r
n
with r
n
= 0 or N(r
n
) < N(r
n1
);
r
n1
= q
n
r
n
+ 0 with r
n+1
= 0.
Such an r
n
exists since N(b) > N(r
1
) > N(r
2
) > > N(r
n
) is a decreasing sequence on
nonnegative integers if the remainders are nonzero, and such sequence cannot continue indenitely.
Then r
n
gcd(a, b). That is, r
n
[a and r
n
[b (use the equations backwards) and whenever c[a and
c[b then c[r
n
(use the equations forward!). In particular, there exist (non unique!) x, y R so that
r
n
= ax +by.
Remark 7.13. It is known but not easy to prove that the quadratic integer ring R =
Z[

D] is a PID if and only if D = 1, 2, 3, 7, 11, 19, 43, 67, 163. If D = 1, 2, 3,


7, 11 then R is also an Euclidean domain. Well discuss some of this next.
Example 7.14.
(1) A eld k is a Euclidean domain. Take N 0 and observe that for every a, b k with
b ,= 0 we have a = qb + 0, where q = ab
1
.
(2) Z is a Euclidean domain. Take N = [ [. Hence Z is a UFD (Fundamental Theorem of
Arithmetic).
Proof. Let a, b be two nonzero integers and suppose rst that b > 0. The half open
intervals [nb, (n + 1)b), with n Z, partition the real line and so a is in one of them,
say a [kb, (k + 1)b). For q = k we have a qb = r [0, [b[), as needed. If b < 0, by
the previous argument there is an integer q such that a = q(b) +r with either r = 0 or
[r[ < [ b[; then a = (q)b +r satises the requirements of the Division Algorithm for a
and b.
Note that this proof always produce a positive remainder r. If for example b > 0 and q, r
are as above with r > 0, then a = q
t
b + r
t
with q
t
= q + 1 and r
t
= r b also satisfy the
conditions of the Division Algorithm applied to a, b. The quotient and the remainder are
unique if we require the remainder to be nonnegative. Indeed, suppose that a = qb +r =
q
t
b + r
t
with 0 r, r
t
[b[ and assume that r r
t
. Then one has 0 r
t
r = b(q q
t
).
After taking the absolute values we have that [b[ [q q
t
[ = r
t
r r
t
< [b[. This is
possible only if [q q
t
[ < 1, hence q = q
t
. It then follows that r = r
t
.
(3) If k is a eld then k[X] is a Euclidean domain (see Corollary 7.27). Take N = deg().
(4) Let Z[

1] = Z[i] = a + ib [ a, b Z C, the Gaussian integers, form a Euclidean


domain. Take N(a +ib) = (a +ib)(a ib) = a
2
+b
2
.
54 7. FACTORIZATION IN RINGS
Proof. For x = a + ib R = Z[i] notice that N(x) = xx = a
2
+ b
2
N
0
. Now
N: R 0 N and N is multiplicative. Moreover, let x, y be in R and assume x ,= 0.
Since C is a eld, y/x C, hence y/x = + i with , 1. Let a, b Z with
[a[ 1/2 and [ b[ 1/2. Set q = a +ib and r = x((a) +i( b)). Now q R,
y = qx +r, and hence r R. Either r = 0, or else N(r) = N(x)N(( a) +i( b))
N(x)((1/2)
2
+ (1/2)
2
) = N(x)1/2 < N(x), since N(x) N.
(5) Let = (1 +

19)/2 and R = Z[] = a + b [ a, b Z. Then R is a PID but not an


Euclidean Domain.
Proof. See Dummit and Foote.
(6) Z[X] is a UFD (see Theorem 7.42) but it is not a PID since (2, X) is not a principal ideal.
(7) Let k be a eld. Then k[X
1
, X
2
] is a UFD, but not a PID (see Corollary 7.43).
(8) The subring Z[2i] = a+2ib [ a, b Z is an integral domain. Both 2 and 2i are irreducibles
in Z[2i] and 4 = 2 2 = 2i 2i. Hence Z[2i] is not a UFD. Note that i , Z[2i] and 2 and
2i are associates in Z[i].
(9) The ring Z[

5] = a+b

5 [, a, b Z is a domain (even Noetherian, see Example 7.51


), but not a UFD.
Proof. Observe that N: R = Z[

5] 0 N given by N(x) = xx is multiplica-


tive. Notice if x = a +b

5 R with N(x) = 1, then a


2
+5b
2
= 1, hence x = 1 R

.
Also, if x[y in R, then N(x)[N(y) in Z. We claim that 2 is irreducible in R. First 2 ,= 0,
and 2 , R

since N(2) ,= 1. Assume 2 = xy with x, y R. Then 4 = N(x)N(y). Thus


N(x) = 1, 2, or 4. Write x = a + b

5. Then N(x) = a
2
+ 5b
2
,= 2. Hence N(x) = 1 or
N(x) = 4, in which case N(y) = 1. Thus x R

or y R

. Therefore 2 is irreducible in
R. We claim now that 2 is not a prime. In R we have that 2[6 = (1 +

5)(1

5).
If 2 is prime, then 2[(1 +

5) or 2[(1

5) in R, thus 4 = N(2)[N(1

5) = 6 in
Z, which is impossible.
2. Polynomial rings
Here R will be a commutative ring. Let R[X] =

iN
0
R = (a
0
, a
1
, . . .) [ a
i
R, almost all a
i
= 0
as an additive abelian group, and dene a multiplication on R[X] by
(a
0
, a
1
, . . .)(b
0
, b
1
, . . .) = (c
0
, c
1
, . . .) with c
n
=
n

i=0
a
i
b
ni
.
Notice that (c
0
, c
1
, . . .) R[X]. With these operations, R[X] is a commutative ring with 1
R[X]
=
(1, 0, 0, . . .), called the polynomial ring (in one variable) over R. Set X = (0, 1, 0, 0, . . .) R[X]
and i : R R[X] with i(a) = (a, 0, 0, . . .).
Remark 7.15.
(a) i : R R[X] is a monomorphism of rings;
(b) X
n
= (0, 0, . . . , 1, 0, . . .) with 1 in the n-th spot;
(c) Every 0 ,= f R[X] can be uniquely written as
n

i=0
a
i
X
i
, where a
i
R and a
n
,= 0.
Proof. As far as (c) is concerned, observe that (0, . . . , a
i
, 0, . . .) = (a
i
, 0, . . .)(0, 0, . . . , 1, 0, . . .)
with 1 in the i-th spot. But that is also equal to i(a
i
)X
i
= a
i
X
i
.
The n in Remark 7.15(c) is called the degree of f, written as deg f. Set deg 0 = . Thus
deg: R[X] N
0
. Then a
n
in Remark 7.15(c) is called the leading coecient of f.
Remark 7.16. Let f and g be in R[X].
(a) deg(fg) deg f + deg g;
(b) If R is a domain then deg(fg) = deg f + deg g for all f and g;
(c) R is a domain if and only if R[X] is a domain;
2. POLYNOMIAL RINGS 55
(d) If R is a domain then (R[X])

= R

.
Proof. Observe that (c) follows from (b). As far as (d) is concerned, since R R[X] it is
clear that R

(R[X])

. Conversely, let f (R[X])

. Then there exists g R[X] with fg = 1.


Thus, by (b), we have that 0 = deg 1 = deg f +deg g. Thus deg f = deg g = 0. Hence f and g are
in R, which means that f R

.
Example 7.17. Let k = Z/2Z and f = X
2
+ X k[X]. Notice that f ,= 0, but that the
polynomial function k k dened by f is 0.
Dene R[X
1
, . . . , X
n
] inductively as R[X
1
, . . . , X
n1
][X
n
] and call this the polynomial ring in n
variables over R. Every f R[X
1
, . . . , X
n
] can be uniquely written as f =

nite
a
i
1
i
n
X
i
1
1
X
i
n
n
with i
j
0 and a
i
1
i
n
R (unique up to deleting 0s). Observe that X
1
, . . . , X
n
in R[X
1
, . . . , X
n
]
are called variables (indeterminates), X
i
1
1
X
i
n
n
are called monomials, f =

nite
a
i
1
i
n
X
i
1
1
X
i
n
n
are called polynomials, a
i
1
i
n
are called coecients of f. Also, the degree of f is given by deg f =
maxi
1
+ +i
n
[, a
i
1
i
n
,= 0 note that max() = .
Remark 7.18. The analogous statement to Remark 7.16 holds for R[X
1
, . . . , X
n
].
Theorem 7.19 (Universal Property of Polynomial Rings). Let : R T be a homomorphism
of commutative rings, and let a
1
, . . . , a
n
T. Then there exists a unique homomorphism of
rings, : R[X
1
, . . . , X
n
] T with
[R
= and (X
i
) = a
i
. This property uniquely determines
R[X
1
, . . . , X
n
] up to isomorphism.
Proof. Every f R[X
1
, . . . , X
n
] can be uniquely written as f =

nite
a
i
1
i
n
X
i
1
1
X
i
n
n
. Set
(f) =

nite
(a
i
1
i
n
)a
i
1
1
a
i
n
n
. This is a well-dened homomorphism or rings with the desired
properties.
In the setting of Theorem 7.19, write f(a
1
, . . . , a
n
) = (f(X
1
, . . . , X
n
)) and call = ev
a
the
evaluation homomorphism.
Corollary 7.20. Let R be a subring of a commutative ring T such that T = R[a
1
, . . . , a
n
].
Then T

= R[X
1
, . . . , X
n
]/I for some R[X
1
, . . . , X
n
]-ideal I.
Proof. It follows from Theorem 7.19 and Theorem 6.23.I.
Example 7.21. Z[i]

= Z[X]/(X
2
+ 1).
Theorem 7.22. Let I be an R-ideal.
(a) IR[X
1
, . . . , X
n
] =
_

nite
a
i
1
i
n
X
i
1
1
X
i
n
n
[ a
i
1
i
n
I
_
;
(b) R[X
1
, . . . , X
n
]/IR[X
1
, . . . , X
n
]

= (R/I)[X
1
, . . . , X
n
];
(c) I is a prime ideal of R if and only if IR[X
1
, . . . , X
n
] is a prime ideal of R[X
1
, . . . , X
n
].
Proof. In order to prove (b), consider the homomorphism: R

R/I (R/I)[X
1
, . . . , X
n
].
By Theorem 7.19, there exists a homomorphism : R[X
1
, . . . , X
n
] (R/I)[X
1
, . . . , X
n
] with
(X
i
) = X
i
. Notice that (

a
i
1
i
n
X
i
1
1
X
i
n
n
) =

(a
i
1
i
n
)X
i
1
1
X
i
n
n
. Thus is an epimor-
phism with Ker() = IR[X
1
, . . . , X
n
] by (a). Now use Theorem 6.23.I. Now, (c) follows from (b)
via Proposition 6.38(a) and Remark 7.16(c).
Proposition 7.23. For a R then R[X]/(X a)

= R.
Proof. The map ev
a
: R[X] R has X a in its kernel and hence it induces the map
: R[X]/(Xa) R. On the other hand, i : R R[X] gives = i : R R[X]/(Xa).
Now is an epimorphism and = id
R
.
Example 7.24.
56 7. FACTORIZATION IN RINGS
(a) R[X
1
, . . . , X
n
]/(X
i+1
, . . . , X
n
)

= R[X
1
, . . . , X
i
]. In particular, (X
i+1
, . . . , X
n
) is a prime
ideal if and only if R is a domain.
(b) Let k be a eld, = (
1
, . . . ,
n
) k
n
and write f(
1
, . . . ,
n
) = f() for any f
k[X
1
, . . . , X
n
]. Then m

= f k[X
1
, . . . , X
n
] [ f() = 0 m-Spec(k[X
1
, . . . , X
n
]).
Moreover, we have that
2
m

= (X
1

1
, . . . , X
n

n
) k[X
1
, . . . , X
n
].
Proof. Let ev

: k[X
1
, . . . , X
n
] k be given by ev

(f) = f(). Then ev

is an
epimorphism of rings with m

= Ker(ev

). Since k is a eld, then m

is a maximal ideal
of k[X
1
, . . . , X
n
]. Clearly, (X
1

1
, . . . , X
n

n
) m

. Conversely, if f m

we can
use Taylors series expansions to obtain
f = f() +
n

i=1
_
f
X
i
_
X=
(X
i

i
) + higher order terms in (X
i

i
).
Since f() = 0 we obtain that f (X
1

1
, . . . , X
n

n
).
Let k be a eld then k(X
1
, . . . , X
n
) = Quot(k[X
1
, . . . , X
n
]) = f/g [ g(X
1
, . . . , X
n
) ,= 0 the eld
of rational functions in n variables over k.
Example 7.25. Let k be a eld and consider the n-tuple = (
1
, . . . ,
n
) k
n
. Then
R = k[X
1
, . . . , X
n
]
(X
1

1
,...,X
n

n
)
is a local ring with R

= f/g [ g() ,= 0 = rational functions
dened at k(X
1
, . . . , X
n
), and the maximal ideal of R is f/g [ g() ,= 0, f() = 0. (See
Example 7.24(b), Corollary 6.64 and Proposition 6.60.)
3. Roots of Polynomials
Here R is a commutative ring.
Theorem 7.26. Let f, g R[X], g ,= 0, and assume that the leading coecient of g is a unit
in R. Then there exist unique q and r in R[X] such that f = qg +r and deg r < deg g.
Proof. We may assume f ,= 0. Write f = a
n
X
n
+ +a
0
with a
n
,= 0 and g = b
d
X
d
+ +b
0
with b
d
R

. We induct on n = deg f. Let n = 0; if d=0 take q = a


0
b
1
0
and r = 0, otherwise
if d > 0 take q = 0 and r = f. Let assume n > 0. If d > n take q = 0 and r = f. Otherwise, if
d n we may write f = a
n
b
1
d
X
nd
g + f
1
with deg f
1
< n. By induction hypothesis there exists
q
1
and r in R[X] with f
1
= q
1
g + r and deg r < d. Now f = (a
n
b
1
d
X
nd
+ q
1
)g + r. We now
show the uniqueness. Suppose f = q
1
g + r
1
= q
2
g + r
2
with deg r
1
< deg g and deg r
2
< deg g.
Then (q
1
q
2
)g = r
2
r
1
has degree < deg g. Thus, since the leading coecient of g is a unit,
q
1
q
2
= 0. Thus q
1
= q
2
and r
1
= r
2
.
Corollary 7.27. Let k be a eld. Then k[X] is a Euclidean domain.
Proof. Take N = deg: k[X] 0 N
0
.
Theorem 7.28. The following are equivalent:
(a) R[X] is a Euclidean domain;
(b) R[X] is a PID;
(c) R is a eld.
Proof. Theorem 7.11 shows that (a) implies (b). Suppose that (b) holds. Since R
R[X], R is a domain. By Example 7.24(a), R[X]/(X)

= R. Thus (X) ,= 0 is a prime ideal
(Proposition 6.38(a)), hence (X) is a maximal ideal (see HW 7.2), hence R

= R[X]/(X) is a eld
(Proposition 6.38(b)). Hence (c) holds. Finally, Corollary 7.27 shows that (c) implies (a).
Notice that by Theorem 7.28, R[X
1
, . . . , X
n
] can never be a PID for n 2.
Proposition 7.29. Let f R[X] and a R. Then:
(a) There exists unique q(X) R[X] with f = q(X) (X a) +f(a).
(b) f(a) = 0 if and only if
3
(X a) [ f.
2
Hilbert Nullstellensatz: If k is algebraically closed then all maximal ideals of k[X
1
, . . . , X
n
] are of the form
(X
1

1
, . . . , X
n

n
) for some (
1
, . . . ,
n
) k
n
.
This fails if k is not algebraically closed: e.g., 1[X]/(X
2
+ 1)

= C.
3
Such an a is called a root of f.
3. ROOTS OF POLYNOMIALS 57
Proof. By Theorem 7.26, since 1 R

and deg(X a) = 1 there exist unique q R[X] and


c R with f = q(X) (X a) + c. Evaluating at X = a yields f(a) = c. This establishes (a).
As far as (b) is concerned, if f(a) = 0 then part (a) implies that X a divides f. The converse is
clear.
Corollary 7.30. Let R be a domain and f ,= 0 in R[X] with deg f = n. Then f has at most
n distinct roots in R.
Proof. We induct on n. If n = 0, then f has no root since f R 0. So let n > 0, and let
a
1
, . . . , a
m
be distinct roots of f. By Proposition 7.29(a) we have that f = q(X)(X a
m
). Now
deg q(X) = n 1. For i m 1 , 0 = f(a
i
) = q(a
i
)(a
i
a
m
), where a
i
a
m
,= 0 since a
i
,= a
m
.
Thus, since R is a domain, q(a
i
) = 0. Hence a
1
, . . . , a
m1
are distinct roots of q. Now by induction
hypothesis, m1 n 1 which gives m n.
Example 7.31. Let R =

iN
Z/2Z and f = X
2
X R[X]. Then deg f = 2, but f has
innitely many roots in R!
Proposition 7.32. Let R be a UFD, K = Quot(R) and let f = a
0
+a
1
X + a
n
X
n
R[X].
If u = c/d K with c and d relatively prime, and u is a root of f, then c divides a
0
and d divides
a
n
.
Proof. Notice that f(u) = 0 implies that
a
0
d
n
= c
_
n

i=1
(a
i
)c
i1
d
ni
_
and a
n
c
n
= d
_
n1

i=0
(a
i
)c
i
d
ni1
_
.
Consequently, if g.c.d.(c, d) = 1 then c[a
0
and d[a
n
.
Theorem 7.33. Any nite (multiplicative) subgroup U of R

, with R a domain, is cyclic.


Proof. Suppose U is not cyclic. Since U is nite and abelian, U would have p
2
distinct
elements satisfying x
p
= 1 for some number
4
p. But this is impossible by Corollary 7.30.
Example 7.34. Let k be a eld.
(a) If k is nite (for example, Z/pZ), then k

is cyclic.
(b) For n 0 let
n
(k) = a k [ a
n
= 1 = a k

[ [a[ [ n be the group of n-th roots of


unity. Then
n
(k) is a subgroup of k

, and by Corollary 7.30, [


n
(k)[ n < . Thus
by Theorem 7.33,
n
(k) is cyclic. Any generator of
n
(k) is called primitive n-th root of
unity
5
. Then (k) =

nN
0

n
(k) = a k

[ [a[ < is called the group of roots of


unity.
Let R be a domain, 0 ,= f R[X], a R a root of f. There exists a well-dened m N with
(Xa)
m
[ f and (Xa)
m+1
[ f, i.e., f = (Xa)
m
g and g(a) ,= 0 see the proof of Corollary 7.30.
We call m the multiplicity of the root a. If m = 1, then a is called a simple root. If m > 1, then a
is called a multiple root. Consider the map D: R[X] R[X] given by
D(f) = f
t
=
n

i=1
ia
i
X
i1
if f =
n

i=0
a
i
X
i
.
Notice that
6
(f +g)
t
= f
t
+g
t
, (fg)
t
= f
t
g +fg
t
and c
t
= 0 for all c R.
Remark 7.35. Let R be a domain, D: R[X] R[X] as above. Then Ker(D) = R if
char(R) = 0 or Ker(D) = R[X
p
] if char(R) = p > 0.
Proposition 7.36. Let R be a domain, 0 ,= f R[X], a R. Then a is a multiple root of f
if and only if f(a) = 0 = f
t
(a).
Proof. Let a be a root and write f = (X a)g, see Proposition 7.29(b). We need to show
that g(a) = 0 if and only if f
t
(a) = 0. But f
t
= g + (X a)g
t
. Thus f
t
(a) = g(a).
4
A nite Abelian group G which is not cyclic contains a subgroup isomorphic to Z
p
Z
p
for some p.
5
Recall that
n
(C) = e
2i
n
).
6
f

is called the derivative of f.


58 7. FACTORIZATION IN RINGS
Example 7.37. Let k be a eld, char(k) = p > 0, a k. Let f = X
p
n
a
p
n
k[X]. Then
X
p
n
a
p
n
= (X a)
p
n
and a is a root of multiplicity p
n
of f.
4. Polynomial Rings over UFDs
Here R will be a UFD. Let 0 ,= f = a
n
X
n
+ + a
0
R[X]. The content of f is dened to be
c(f) = gcd(a
0
, . . . , a
n
). Notice that it is determined up to . We also say that f is primitive
7
if
c(f) 1. Notice that f = c(f) f
1
, with f
1
primitive.
Lemma 7.38 (Gauss). Let R be a UFD. Then c(fg) c(f)c(g). In particular, if f and g are
primitive, then so is fg.
Proof. Write f = c(f)f
1
and g = c(g)g
1
with f
1
and g
1
primitive polynomials. Then fg =
(c(f)c(g))f
1
g
1
. It suces to show that f
1
g
1
is a primitive polynomial. Write f
1
=
n

i=0
a
i
X
i
and
g
1
=
m

i=0
b
i
X
i
. Then f
1
g
1
=
n+m

i=0
c
i
X
i
with c
i
=
i

j=0
a
j
b
ij
. Suppose that f
1
g
1
is not primitive.
Then there exists a prime element p of R with p[c
i
for all i. But since f
1
and g
1
are primitive, there
exists a smallest l with p [ a
l
and a smallest k with p [ b
k
. Now c
l+k
= a
l
b
k
+
l+k

j=0
t
a
j
b
l+kj
a
l
b
k
mod (p). Thus p[c
l+k
implies p[a
l
b
k
. Thus p[a
l
or p[b
k
, which is impossible.
Lemma 7.39. Let R be a UFD, K = Quot(R), f and g in R[X], f primitive in R[X]. Then
f[g in R[X] if and only if f[g in K[X].
Proof. We only need to prove the converse. Suppose g = fh for some h K[X]. Thus for
some a R, a ,= 0, we have ag = f(ah) with q = ah R[X]. Now, ag = fq and by Lemma 7.38
we have ac(g) c(ag) c(fq) c(f)c(q) c(q). Thus a[c(q) in R, hence q/a R[X]. Now
g = f(q/a) with q/a R[X].
Lemma 7.40. Let R be a UFD, K = Quot(R), f R[X] primitive in R[X]. Then f is a prime
element of R[X] if and only if f is a prime element of K[X].
Proof. Notice that in either case deg f 1 (since f ,= 0, and either f is primitive in R[X]
with f , R

or else f , K

). Thus f is prime in R[X] if and only if for all g, h R[X] with f[gh
then f[g or f[h (division in R[X]). By Lemma 7.39 this occurs if and only if for all g, h R[X]
with f[gh then f[g or f[h (division in K[X]). But this happens if and only if for all g, h K[X]
with f[gh then f[g or f[h (since every element of K[X] is associate to an element of R[X]). This
is equivalent to f being prime in K[X].
Lemma 7.41. Let R be a domain, p R. Then p is a prime element in R if and only if p is a
prime element in R[X].
Proof. In either case p ,= 0. But then p is a prime element in R if and only if pR is a prime
ideal in R. By Theorem 7.22(c) this occurs if and only if pR[X] is a prime ideal in R[X] and,
hence, if and only if p is a prime element in R[X].
Theorem 7.42 (Gauss). Let R be a UFD. Then R[X
1
, . . . , X
n
] is a UFD.
Proof. By induction on n, it suces to show that R[X] is a UFD. So let f R[X] be a
nonzero nonunit. We show by induction on m = deg f that f is a product of prime elements
in R[X]. If m = 0, then f R is a product of prime elements in R (since R is a UFD), and
hence of prime elements in R[X], by Lemma 7.41. If m > 0, then by induction hypothesis, it
suces to show that there exists a prime element p R[X] with deg p > 0 and p[f in R[X]. So
let K = Quot(R). Since K[X] is a UFD (by Theorem 7.28 and Corollary 7.9) and deg f > 0,
there exists a prime element q of K[X] with q[f in K[X]. But q p for some p R[X] which is
primitive in R[X]. Now p is a prime element in K[X], hence deg p > 0 and p is a prime element
7
For example if f is monic, i.e., if a
n
= 1.
4. POLYNOMIAL RINGS OVER UFDS 59
in R[X] (since p R[X] is primitive in R[X], use Lemma 7.40). Also, p[f in K[X], hence p[f in
R[X] (since p R[X] is primitive in R[X], use Lemma 7.39).
Corollary 7.43. Let k be a eld. Then k[X
1
, . . . , X
n
] is a UFD (but not a PID if n > 1).
Corollary 7.44. Let R be a UFD, K = Quot(R), and f R[X]. Then f is irreducible in
R[X] if and only if either f R and f is irreducible in R or else f is primitive in R[X] and
irreducible in K[X].
Proof. Notice that since R[X] and K[X] are UFDs, primeness and irreducibility are equiv-
alent (Theorem 7.42 and Proposition 7.5). Furthermore if f , R and irreducible in R[X], then f
is primitive in R[X]. Now use Lemma 7.41 and Lemma 7.40.
Notice that if k is a eld then f k[X] is irreducible if and only if f , k and f = gh implies g k
or h k.
Theorem 7.45 (Eisensteins Criterion). Let R be a UFD, K = Quot(R), f = a
n
X
n
+ +a
0
with n > 0. Assume that there exists a prime element p of R with p [ a
n
, p [ a
i
for all i < n, and
p
2
[ a
0
. Then f is irreducible in K[X]. If f is primitive in R[X], then f is irreducible in R[X].
Proof. Write f = c(f)f
1
where f
1
is primitive in R[X]. Since p [ a
n
then p [ c(f) and
therefore f
1
satises the same assumptions as f. Thus we may assume that f = f
1
is primitive
in R[X]. By Lemma 7.40, we only need to prove that f is irreducible in R[X]. Since f , R and
f is primitive it suces to prove that if f = gh, then deg g = 0 or deg h = 0. Suppose f = gh,
where g = b
s
X
s
+ + b
0
and h = c
t
X
t
+ + c
0
with deg g = s > 0 and deg h = t > 0. Since
p[a
0
= b
0
c
0
and p is prime, we have p[b
0
or p[c
0
; say p[b
0
. Since p [ a
n
= b
s
c
t
, we have that p [ b
s
.
Let k = minj [ p [ b
j
. Then 1 k s < s + t = n. Since k < n, p[a
k
= b
0
c
k
+ + b
k
c
0
. Thus
by the minimality of k, p[b
k
c
0
. Thus, since p [ b
k
we have that p[c
0
. But then p
2
[b
0
c
0
= a
0
, which
is a contradiction.
Example 7.46.
(a) Let f = Z
4
+XY
2
Z
2
+X
2
Y Z+XY
3
k[X, Y, Z], where k is a eld. Then f is irreducible.
Proof. Let R = k[X, Y ]. Then R is a UFD by Theorem 7.42 and X is a prime
element of R by Example 7.24(a). Now f R[Z] satises the assumptions of Theorem 7.45
with p = X. Furthermore f R[Z] is monic, hence primitive. Thus by Theorem 7.45, f
is irreducible in R[Z] = k[X, Y, Z].
(b) Let p be a prime number and n > 0. Then X
n
p is irreducible in [X] and Z[X].
(c) Let f(X) = X
4
+ 1 Z[X]. Now Eisensteins Criterion does not apply directly to f(X).
Consider g(X) = f(X + 1) = (X + 1)
4
+ 1 = X
4
+ 4X
3
+ 6X
2
+ 4X + 2. Then g(X) is
irreducible in Z[X] with p = 2. Hence f(X) is irreducible in Z[X] since any factorization
of f(X) into nonconstant polynomials in Z[X] leads to a nonconstant factorization of
g(X).
(d) Let p be a prime number. Then f =
p1

i=0
X
i
is irreducible in [X] and Z[X].
Proof. Notice that ev
X+1
: [X] [X] is an automorphism of rings since ev
X+1

ev
X1
= id and ev
X1
ev
X+1
= id. Thus it suces to show that ev
X+1
(f) = f(X + 1)
is irreducible in [X]. Applying ev
X+1
to the equation (X 1)f(X) = X
p
1, we
obtain Xf(X + 1) = (X + 1)
p
1 =
p

k=1
_
p
k
_
x
k
. Thus f(X + 1) =
p

k=1
_
p
k
_
X
k1
=
X
p1
+
p1

k=2
_
p
k
_
X
k1
+p, which is irreducible by Theorem 7.45.
Theorem 7.47 (Reduction Criterion). Let R be a domain, K = Quot(R), f = a
n
X
n
+ +a
0

R[X] with deg f = n > 0. Let p be a prime ideal of R with a
n
, p, L = Quot(R/p), f the image
of f in (R/p)[X]. If f is irreducible in L[X], then f cannot be written in the form f = gh with
g, h R[X] of degree > 0. If in addition R is a UFD then f is irreducible in K[X].
60 7. FACTORIZATION IN RINGS
Proof. Suppose f = gh with g = b
m
X
m
+ +b
0
, b
m
,= 0, and h = c
k
X
k
+ +c
0
, c
k
,= 0.
Then a
n
= b
m
c
k
; thus in R/p we have 0 ,= a
n
= b
m
c
k
, hence b
m
,= 0 and c
k
,= 0. Thus f = gh
with deg g = m and deg h = k. Since f is irreducible in L[X], it follows that m = 0 or k = 0. If R
is a UFD, then f is irreducible in K[X] by Lemma 7.39.
Example 7.48. The polynomial f = X
5
X
2
+ 1 is irreducible in [X] (and in Z[X]).
Proof. To see this, we apply Theorem 7.47 with R = Z and p = 2Z, and we need to show
that f is irreducible in (Z/2Z)[X]. Suppose not, then f would have a divisor of degree 1 or 2 in
(Z/2Z)[X]. But degree 1 is impossible since f has no root in Z/2Z. Thus f would have a divisor
of degree 2 in (Z/2Z)[X] without a root in Z/2Z. But the only such polynomial is X
2
+ X + 1
and X
2
+X + 1 [ f since X
5
X
2
+ 1 = (X
3
+X
2
)(X
2
+X + 1) + 1.
5. Hilberts Basis Theorem
Here R is a commutative ring. The following result, due to Hilbert, says that if every ideal in a
ring R has a nite basis (i.e., is nitely generated) then so does every ideal in the polynomial ring
R[X]. Hilberts result connected the theory of invariants to the eld of algebraic functions and
algebraic varieties.
Theorem 7.49 (Hilberts Basis Theorem). If R is Noetherian then so is R[X
1
, . . . , X
n
].
Proof. It suces to prove that R[X] is Noetherian. Let J be any R[X]-ideal. We need to show
that J is nitely generated. For every integer i 0 we dene the subset of R, I
i
= 0 leading
coecients of polynomials of degree i in J. Since J is an ideal in R[X], I
i
are ideals in R.
Furthermore
8
we have that I
0
I
1
I
i
I
i+1
. Hence I =
_
i0
I
i
is an R-ideal. Since
R is a Noetherian ring we have that I is nitely generated, say by h
1
, . . . , h
l
. Now for some n,
h
1
, . . . , h
l
I
n
so that I
0
I
1
I
n
= I
n+1
= I
n+2
= . Since R is Noetherian the
I
i
s are nitely generated R-ideals, say I
i
= (a
i1
, . . . , a
is
i
). Let f
ij
be polynomials in J with f
ij
=
a
ij
X
i
+ lower terms. We claim that J is generated by the nite set S = f
ij
[ 1 j s, 0 i n.
Obviously, (S) J. Suppose (S) ,= J. Then there exists f J of smallest possible degree i 0
with f , (S). Write f = aX
i
+ lower terms; now a I
i
. If i n then a = r
1
a
i1
+ + r
s
i
a
is
i
for some r
j
R. Since f
ij
= a
ij
X
i
+ lower terms, we have that f r
1
f
i1
r
s
i
f
is
i
has degree
i 1. Since this polynomial is in J, by the minimality of i, it is in (S). Thus f (S) notice
that f
ij
(S) since i n. Suppose now that i > n. Since I
i
= I
n
, a = ra
n1
+ + r
s
n
a
ns
n
for
some r
j
R. Since f
nj
= a
nj
X
n
+ lower terms we have that f r
1
X
in
f
n1
r
s
n
X
in
f
ns
n
has degree i 1. Thus, as above, f (S) notice that f
nj
S.
Corollary 7.50. Let R be a subring of a commutative ring T such that T = R[a
1
, . . . , a
n
]. If
R is Noetherian then so is T.
Proof. By Corollary 7.20, T

= R[X
1
, . . . , X
n
]/I for some n and some ideal I of R[X
1
, . . . , X
n
].

Example 7.51. Z[

5]

= Z[X]/(X
2
+ 5) is a Noetherian domain.
Let k be a eld. For s subset S k[X
1
, . . . , X
n
] write V (S) = a k
n
[ f(a) = 0 for all f
S k
n
and call this an algebraic set. For f k[X
1
, . . . , X
n
] write V (f) = V (f) and call this a
hypersurface. Notice that V ((S)) = V (S) and V (S
1
S
2
) = V (S
1
) V (S
2
).
Corollary 7.52. Every algebraic set in k
n
is the intersection of nitely many hypersurfaces
in k
n
.
Proof. Let V (S) k
n
be an algebraic set. By Theorem 7.49, k[X
1
, . . . , X
n
] is a Noetherian
ring, thus (S) is a nitely generated ideal in this ring, say (S) = (f
1
, . . . , f
s
). Now V (S) =
V ((S)) = V ((f
1
, . . . , f
s
)) = V (f
1
, . . . , f
s
) = V (f
1
) . . . V (f
s
).
One can prove the non trivial fact that every algebraic set in k
n
is the intersection of (at most) n
hypersurfaces in k
n
(Storch, Eisenbud-Evans).
8
Since for 0 ,= a I
i
, a is the leading coecient of some f J with deg f = i, then a is the leading coecient
of Xf J with deg Xf = i + 1, thus a I
i+1
.
5. HILBERTS BASIS THEOREM 61
Homework set # 7.
Choose ve of the following problems. The due date is on January 28 (Monday), 2002.
1. Let R = f : [0, 1] 1[ f continuous. Then R is a commutative ring with the usual
addition and multiplication of functions as operations.
Show that R is not Noetherian.
Determine m-Spec(R).
2. Let R be a PID. Show that every non-zero prime ideal of R is a maximal ideal.
3. Let R be a commutative ring with 1 ,= 0. A homomorphism D: R R of additive
groups satisfying D(ab) = aD(b) +bD(a) for every a, b R is called a derivation on R.
Show that Ker(D) is a subring of R containing 1.
Show that 0 is the only derivation on Z[i].
4. Let S , 0 be a multiplicative subset of R. Show that if R is a UFD, then S
1
R is a UFD.
5. Factor 3 + 4i into irreducible elements in Z[i].
Find a generator for the ideal (85, 1 + 13i) in Z[i]. In other words, nd the greatest
common divisor for 85 and 1 + 13i.
6. Let R be a domain. If an l.c.m. of a and b exists then so does a g.c.d. (Notice that the
converse does not hold in general.)
7. If R is a UFD, show that the intersection of an arbitrary collection of principal ideals is
again principal.
8. If R is a UFD, show that every non-zero minimal prime ideal is principal.
9. An integral domain R is a UFD if and only if the ascending chain condition holds for
principal ideals, and any two elements of R has an l.c.m.
Homework set # 8.
Choose ve of the following problems. The due date is on February 11 (Monday), 2002.
1. Is the polynomial x
4
5 irreducible in Z
3
[X]? If not, nd a factorization.
2. Let m be a maximal ideal of Z[X] such that mZ ,= 0. Show that m = (p, f), where p is
a prime in Z and f is a monic irreducible polynomial in X of positive degree.
3. Let R be a UFD, p be a prime ideal of R[X] with p R = 0. Show that p is principal.
4. Show that the polynomial f(X) = X
3
5X 1 is irreducible in Z[X].
5. Let n 2, m 1. Use induction on n to show that the polynomial f(X
1
, . . . , X
n
) =
_
n

i=1
X
m
i
_
1 is irreducible in [X
1
, . . . , X
n
].
6. Let I k[X, Y ] consist of all polynomials f(X, Y ) with coecients in a eld k such that
the sum of the coecients of f(X, Y ) is zero.
Show that I is an ideal of k[X, Y ].
Show that I is a maximal ideal.
7. Let R = Z[

5] and K = Quot(R). Show that f(X) = 3X


2
+ 4X + 3 is irreducible in
R[X] but is reducible in K[X].
8. Let k be a eld of characteristic p > 0 and let a k. Show that f(X) = X
p
X a is
irreducible in k[X] if f(X) has no root in k.
9. Show that f(X) = X
4
+ 3X
3
+ 3X
2
5 is irreducible in [X].
10. Let k be a eld and X, Y, t be indeterminates over k. Dene the homomorphism of rings
: k[X, Y ] k[t
2
, t
3
] k[t] by (X) = t
2
and (Y ) = t
3
. Show that Ker() = (X
3
Y
2
)
and that it is a prime ideal.
11. Let a
n
X
n
+ +a
0
be a zero divisor in R[X]. Show that there exists b R, b ,= 0, with
ba
i
= 0 for all i.
12. Let R be the set of all polynomials a
0
+a
1
X+ +a
n
X
n
such that the a
i
are all integers
and a
1
is even. Prove that R is an integral domain which is not a UFD.
13. Let k be a eld and let R = f(X) k[X] [ f
t
(0) = 0. Show that R is an integral domain
which is not a UFD.
CHAPTER 8
Fields and Field Extensions
Here k, K, L will denote elds.
1. Basic properties of eld extensions
If K is a subeld of a eld L, then L is said to be a eld extension of K. If K L is a eld
extension, then L is a K-vector space. Write [L: K] = dim
K
L and call [L: K] the degree of the
eld extension. We say that the eld extension is nite if [L: K] < (otherwise innite).
Proposition 8.1. Let k K L be eld extensions. Then [L: k] = [L: K][K: k].
Proof. Suppose rst that [L: k] is nite. Since K is a subspace of L (as a vector space over
k), [K: k] is also nite. Any nite set that spans L over k, for instance a basis, will also span L over
K; hence [L: K] is also nite. To complete the proof of the theorem we assume that [K: k] = m
and [L: K] = n and we prove that [L: k] is nite and equals mn. Let u
1
, . . . , u
n
be a basis of L
over K and v
1
, . . . , v
m
a basis of K over k. We claim that the mn elements u
i
v
j
with 1 i n
and 1 j m form a basis of L over k. We must show that: they span L and that they are
linearly independent over k. Let z be an element of L. We can write z =

b
i
u
i
with b
i
K.
Each b
i
can in turn be written b
i
=

c
ij
v
j
with c
ij
k. This yields z =

c
ij
u
i
v
j
, this last sum
being over both i and j. Suppose now that

c
ij
u
i
v
j
= 0 where each c
ij
is in k and the sum is
over both i and j. We must show c
ij
= 0. Write b
i
=

c
ij
v
j
. Then b
i
is in K and

b
i
u
i
= 0.
Since the us are linearly independent over K, each b
i
= 0, i.e.,

c
ij
v
j
= 0. Since the vs are
linearly independent over k, we conclude that c
ij
= 0.
Let K L be a eld extension and let Z be a subset of L. We write K[Z] to denote the subring
of L generated by Z over K: it is the smallest subring of L containing K and Z. In other words,
K[Z] consists of all polynomial expressions of elements in Z with coecients in K. We write
K(Z) to denote the subeld of L generated by Z over K: it is the smallest subeld of L containing
K and Z. In other words, K(Z) consists of all quotients f/g with f, g in K[Z] and g ,= 0. If
Z = u
1
, . . . , u
n
we write K[u
1
, . . . , u
n
] = K[Z] and K(u
1
, . . . , u
n
) = K(Z).
Remark 8.2. K(Z) = Quot(K[Z]).
Proof. See Proposition 6.60.
The eld extension K L is nitely generated if L = K(u
1
, . . . , u
n
). The eld extension K L
is simple if L = K(u). Let K
1
and K
2
be subelds of a eld L. We write K
1
K
2
to denote
the compositum of K
1
and K
2
in L: it is the smallest subeld of L containing K
1
and K
2
. In
other words, K
1
K
2
= K
1
(K
2
) = K
2
(K
1
). Suppose that we have the following eld extensions:
k K
1
L and k K
2
= k(Z) L. Then K
1
K
2
= K
1
(Z).
K
1
K
2
= K
1
(Z)
,
K
1
K
2
= k(Z)
,
k
Let K L be a eld extension.
(a) u L is called algebraic over K if there exists f(X) K[X], f(X) ,= 0, with f(u) = 0.
Otherwise u is called transcendental over K.
(b) The eld extension is called algebraic if every u L is algebraic over K. Otherwise it is
called transcendental.
63
64 8. FIELDS AND FIELD EXTENSIONS
Example 8.3.
(1) K is algebraic over K.
(2) i is algebraic over ; e and are transcendental over .
(3) K(X
1
, . . . , X
n
) the eld of rational functions is transcendental over K if n 1.
Let L
1
K L
2
be eld extensions. The homomorphism : L
1
L
2
is a K-homomorphism if
it is a homomorphism of rings with
[K
= id
K
.
Remark 8.4. Let K L be a eld extension and let u L an element which is transcendental
over K. Then there is a K-isomorphism K(X)

= K(u) mapping X to u.
Proof. There is an epimorphism of K-algebras
1
K[X] K[u] mapping X to u, which has to
be injective since u is transcendental. Thus K[X]

= K[u], which induces K(X) = Quot(K[X])

=
Quot(K[u]) = K(u) (see Remark 8.2).
Let K L be a eld extension and let u L be an element which is algebraic over K. Then there
exists a unique monic polynomial f(X) K[X] of minimal degree with f(u) = 0. This polynomial
is called the minimal polynomial of u over K.
Proposition 8.5. Let K L be a eld extension, u L an element which is algebraic over
K and p(X) the minimal polynomial of u over K.
(a) There is a K-isomorphism K[X]/(p(X))

= K[u] mapping X to u.
(b) K[u] = K(u).
(c) p(X) is irreducible.
(d) [K(u): K] = deg p(X) < (called the degree of u over K).
(e) 1, u, . . . , u
n1
form a K-basis of K(u), where n = deg p(X).
Proof. There exists an epimorphism of K-algebras : K[X] K[u] mapping X to u. By
the proof of Theorem 7.11, Ker() = (p(X)) since K[X] is an Euclidean domain and p(X) ,= 0 is of
minimal degree in Ker(). Now use the First Isomorphism Theorem 6.23.I. This proves (a). Being
a subring of a eld, K[u] is a domain. Thus by (a), (p(X)) is a prime ideal. Hence p(X) is a prime
element since p(X) ,= 0. Thus p(X) is irreducible and (c) is established. Since p(X) is irreducible
and K[X] is a PID, K[X]/(p(X)) is a eld. Hence by (a) K[u] is a eld, hence K[u] = K(u).
Thus (b) is proved. Since u
n

n1

i=0
Ku
i
, we have that K[u] =
n1

i=0
Ku
i
. Also, 1, u, . . . , u
n1
are
linearly independent over K by the minimality of n = deg p(X). This yields property (e). Finally,
(d) follows from (e).
Proposition 8.6. Let K L be a eld extension. Then L is nite over K if and only if L is
nitely generated and algebraic over K.
Proof. If L is nite over K then L is clearly nitely generated over K. To show that L
is algebraic over K, let u L. Since dim
K
L < , there exists n with 1, u, . . . , u
n
linearly
dependent over K. Thus
n

i=0
a
i
u
i
= 0 for some a
i
K, not all a
i
= 0. Thus 0 ,= f =
n

i=0
a
i
X
i
is
such that f(u) = 0. Conversely, suppose that L = K(u
1
, . . . , u
n
) and write L
i
= K(u
1
, . . . , u
i
) for
0 i n. Then for 1 i n, u
i
is algebraic over K, hence over L
i1
. Now, by Proposition 8.5
we have that [L
i
: L
i1
] < since L
i
= L
i1
(u
i
) and u
i
is algebraic over L
i1
. Now, we have
K = L
0
L
1
L
n
= L with each [L
i
: L
i1
] < . Thus [L: K] < by Proposition 8.1.
Remark 8.7. Let K L be a nite eld extension, and let u L. By Proposition 8.6, u
is algebraic over K, hence has a minimal polynomial p(X). On the other hand, K(u) is a nite
dimensional K-vector space and =
u
, multiplication by u, is a K-linear map. Now p(X) is also
the characteristic polynomial of .
1
Let R be a ring. An R-algebra is a ring S together with a homomorphism of rings : R S with (R) Z(S).
An R-subalgebra of an R-algebra S is a subring of S containing (R), and a homomorphism of R-algebras is an
R-linear homomorphism of rings.
2. RULER AND COMPASS CONSTRUCTIONS 65
Proof. Let p(X) = X
n
+a
n1
X
n1
+ +a
1
X +a
0
be the minimal polynomial of u. Then
a matrix representation of is given by the n n matrix
A =
_

_
0 a
0
1 0 a
1
1
.
.
.
a
2
.
.
.
0
.
.
.
1 a
n1
_

_
,
the companion matrix of q(X). It is easy to check that det(XI
n
A) = p(X).
Let ( be a class of eld extensions. We say that ( is distinguished if
2
(1) Let k K L be eld extensions. Then k L belongs to ( if and only if k K and
K L belong to (.
(2) Let k K and k L be contained in some eld. If k K belongs to ( then L KL
belongs to (.
(3) Let k K and k L be contained in some eld. If k K and k L belong to (, then
k KL belongs to (.
Remark 8.8. The class of nite extensions is distinguished.
Proof. Property (1) follows from Proposition 8.1, and (2) is clear.
Proposition 8.9. The class of algebraic extensions is distinguished.
Proof. Let k K L be eld extensions. Obviously, if k L is algebraic, then so is
k K and K L. Conversely, assume k K and K L are algebraic. Let u L. Since u
is algebraic over K, there exists 0 ,= f =
n

i=0
a
i
X
i
K[X] with f(u) = 0. Thus u is algebraic
over K
0
= k(a
1
, . . . , a
n
). Since K
0
K and k K is algebraic, k K
0
is algebraic. But K
0
is
nitely generated over k, hence nite over k by Proposition 8.6. Also, since u is algebraic over K
0
,
K
0
(u) is nite over K
0
by Proposition 8.5. Thus by Remark 8.8, K
0
(u) is nite over k, hence u is
algebraic over k by Proposition 8.6. This proves (1). As far as (2) is concerned, let u KL. Then
u L(u
1
, . . . , u
n
) with u
i
K.
Let K L be a eld extension. Then K
L
= u L[ u is algebraic over K is the algebraic closure
of K in L.
Proposition 8.10. K
L
is a subeld of L containing K.
Proof. It suces to show that if u
1
and u
2
are in K
L
then K(u
1
, u
2
) K
L
. Equivalently, it
suces to show that if u
1
L and u
2
L are algebraic over K, then K(u
1
, u
2
) is algebraic over K.
But if u
1
and u
2
are algebraic over K, then K(u
1
) and K(u
2
) are nite over K, see Proposition 8.5,
hence algebraic over K, see Proposition 8.6. Thus by Proposition 8.9, K(u
1
, u
2
) = K(u
1
)K(u
2
) is
algebraic over K.
Corollary 8.11. Let K L = K(Z) be a eld extension. Then L is algebraic over K if and
only if every element of Z is algebraic over K.
Corollary 8.12. K
L
L
= K
L
.
2. Ruler and compass constructions
We shall indicate briey how Propositions 8.1 and 8.5 suce to show the impossibility of the
classical ruler
3
and compass constructions. To do this, we must translate the geometric problem
into algebra. We take the point of view of analytic geometry, labelling the points in the Euclidean
plane with ordered pais of numbers. We take it as our starting point that all points with integral
2
Notice that (3) follows from (1) and (2)
3
To be picky one should call them straight-edge and compass constructions.
66 8. FIELDS AND FIELD EXTENSIONS
coordinates are in our possession. We are then allowed to perform ruler and compass constructions
to acquire new points. Any point obtainable this way we may call constructible. We call the real
number a constructible if the point (a, 0) is constructible. Evidently, (a, b) is a constructible point
if and only if a and b are constructible numbers. The rule and compass constructions that are
permitted may be set forth carefully as follows
4
(1) Given four distinct points A, B, C, D such that AB and CD are distinct non-parallel lines
we are allowed to acquire the point of intersection of AB and CD.
(2) Given distinct points A, B and distinct points C, D, such that the circle with center A
and radius AB meets CD, we are allowed to acquire the points of intersection of and
CD.
Now suppose the coordinates of the points A, B, C, D lie in a subeld of the eld of real numbers.
Then simple arguments from analytic geometry show that in case (1) the coordinates of the new
point lie in K, while in case (2) the coordinates of the new point lie either in K or in K(

a) where
a is a positive number in K. It follows that any constructible number u lies in a subeld K
n
of
the real numbers which is the end product of a series of adjunctions
Rationals = K
0
K
1
K
n
with each K
i
equal to K
i1
(

a
i
), a
i
a positive number in K
i1
. By iterated use of Proposition 8.1,
[K
n
: K
0
] is a power of 2; and then by another application of Proposition 8.1, [K(u): K
0
] is a power
of 2. We have proved
Theorem 8.13. Any constructible real number is algebraic over the rational numbers, and its
degree over the rational numbers is a power of 2.
It is now a simple matter to demolish the three classical problems on ruler and compass construc-
tions.
(1) Squaring the circle: This means constructing . Since is not even algebraic over the
rational numbers (this is a hard theorem!) the question of degrees does not even enter.
(2) Duplicating the cube: The number
3

2 (the real cube root of 20 is to be constructed.


Since X
3
2 is the irreducible (by Eisensteins Criterion) polynomial for
3

2, then
3

2
has degree 3 over the rationals and is not constructible.
(3) Trisection of angles: Some angles, e.g. 90

, can be trisected by ruler and compass. We ex-


hibit one angle, namely 60

, that cannot be trisected by ruler and compass. The question


is equivalent to the constructibility of cos 20

or u = 2 cos 20

. From the trigonometric


identity cos 3 = 4 cos
3
3 cos we deduce u
3
3u 1 = 0. Since the polynomial
X
3
3X 1 is irreducible over the rationals (check!), it follows that u has degree 3 and
is not constructible.
3. Algebraic closure
Let k K be a eld extension and L a eld. We say that : k L is an embedding if is
an injective homomorphism of rings, that is is a nonzero homomorphism of rings. We say that
: K L is an embedding over if is an embedding with
[k
= , that is extends . We
say that : K L is an embedding over k if k L and is an embedding over id
k
, that is
is a k-embedding. An embedding : k L extends to an embedding of polynomial rings
k[X] L[X] mapping X to X, which we still denote with . Images under are often denoted
by ()

. E.g., u

= (u), k

= (k), f

= (f) with u k, f k[X].


Remark 8.14. Let f(X) k[X] and u a root of f(X) in K. If : K L is an embedding
over , then u

is a root of f

(X). In particular, if : K L is an embedding over k, then u

is a root of f(X).
Proof. (f(u))

= f

(u

) = f

(u

).
Proposition 8.15. Let k be a eld, f(X) k[X] k. Then there exists a eld extension k K
so that f(X) has roots in K.
4
The case where two intersecting circles are drawn can be reduced to (1) and (2).
3. ALGEBRAIC CLOSURE 67
Proof. Let p(X) be an irreducible factor of F(X) (which exists since f(X) , k). We may
replace f(X) by p(X). Since p(X) is irreducible L = k[X]/(p(X)) is a eld. Let : k[X] L be
the natural projection and =
[k
: k L. Then is an embedding since ,= 0 and k is a eld.
Also p

(X) = (p(X)) = 0. Thus p

(X) has a root in L, namely X. Write K = k


.
(L (k)),
and extend : k L to a bijective map : K L, and use to dene a eld structure on K.
Then k K is a eld extension, and p(X) has a root in K, namely,
1
(X).
Corollary 8.16. Let k be a eld, f
1
(X), . . . , f
n
(X) k[X] k. Then there exists a eld
extension k K so that f
1
(X), . . . , f
n
(X) have roots in K.
Proof. Induct on n, the case n = 1 being clear by Proposition 8.15. So let n 2. By
induction hypothesis, there is a eld extension k L so that f
1
(X), . . . , f
n1
(X) all have roots
in L. Now f
n
(X) L[X] L. Thus there is a eld extension L K so that f
n
(X) has a root in
K.
A eld K is called algebraically closed if every f(X) K[X] K has a root in K. Equivalently, if
every f(X) K[X] K is a product of linear polynomials in K[X].
Theorem 8.17. Let k be a eld. There exists an algebraic extension k K so that K is
algebraically closed.
Proof. To every f(X) k[X] k, associate a letter X
f
, and let S = X
f
[ f(X) k[X] k,
R = k[S]. In R consider the R-ideal I generated by f(X
f
) [ f(X) k[X] k. We claim I ,= R.
Suppose I = R, then
n

i=0
g
i
f
i
(X
f
i
) = 1 for some g
i
R. Write X
i
= X
f
i
. Now
n

i=0
g
i
f
i
(X
i
) = 1.
By Corollary 8.16, there is a eld extension k K with f
i
(u
i
) = 0 for some u
i
K, 1 i n.
Now substituting u
i
for the variables X
i
and 0 for the other variables, the equation
n

i=0
g
i
f
i
(X
i
) = 1
would imply 0 = 1 in L, which is impossible since L is a eld. Thus I ,= R. Thus there exists a
maximal ideal m of R with I m. Now L = R/m is a eld, =
[k
: k L is an embedding,
and for all f(X) k[X] k we have f

(X
f
) = (f(X
f
)) = 0 since I m. By the set theoretic
argument of the proof of Proposition 8.15, we obtain an honest eld extension k L
1
so that
every f(X) k[X] k has a root in L
1
. Now inductively one obtains a sequence of eld extensions,
k = L
0
L
1
L
i
so that for i 0, all f(X) L
i
[X] L
i
has a root in L
i+1
. Now set
L =

_
i=0
L
i
. Then L is a eld and L is algebraically closed. Now dene K = k
L
. Then k K is an
algebraic eld extension by Proposition 8.10. Furthermore K
L
= K by Corollary 8.12. Thus K is
algebraically closed. For let f(X) K[X] K, then f(X) has a root u in L since L is algebraically
closed. But u L is algebraic over K, hence u K
L
= K. Thus f(X) has a root u in K.
Proposition 8.18. Let k K = k(u) be an algebraic eld extension, let p(X) be the minimal
polynomial of u over k, and let : k L be an embedding. Then for every extension : K L
of , u

is a root of p

(X). Conversely, for every root v of p

(X) in L, there exists a unique


extension : K L of so that u

= v. In particular
#extensions : K L of = #roots of p

(X) in L deg p(X) = [K: k].


Proof. Let v be a root be a root of p

(X) in L. Now : k[X] k

[X] induces an isomor-


phismk[X]/(p(X))

k

[X]/(p

(X)) that extends . By Proposition 8.5, there are isomorphisms


over k and k

K = k(u)

k[X]/(p(X)) mapping u to X and k

[X]/(p

(X))

k

(u) L
mapping X to u. The composition of these three embeddings gives an embedding : K L
that extends and maps u to v. Such is uniquely determined since K = k(u).
Theorem 8.19. Let k K be an algebraic extension, and let : k L be an embedding,
where L is an algebraically closed eld. Then there exists an embedding : K L that extends
. If K is algebraically closed and k

L is algebraic, then any such is an isomorphism.


68 8. FIELDS AND FIELD EXTENSIONS
Proof. Let = (F, ) [ k F K elds, : F L embedding extending . On
dene a partial order by (F, ) (F
t
,
t
) if and only if F F
t
and extends
t
. Now ,=
since (k, ) , and every totally ordered subset of has an upper bound in . Thus by Zorns
Lemma, has a maximal element, say (E, ). We claim that E = K. Suppose not, then since
K is algebraic over E, there exists a simple algebraic extension E _ E(u) K. Let p(X) be the
minimal polynomial of u over E. Then p

(X) has a root in L since p

(X) , L and L is algebraically


closed. But then by Proposition 8.18, there is an embedding E(u) L that extends . This is
a contradiction to the maximality of (E, ) in . Thus : K L is an embedding that extends
. Now suppose that K is algebraically closed and k

L is algebraic. Let : K L be an
embedding that extends . Then K

is algebraically closed, and K

L is algebraic since k

L
is algebraic. Thus K

= L, which shows that is an isomorphism.


Corollary 8.20. Let k be a eld and let k K and k K
t
be algebraic eld extensions with
K and K
t
algebraically closed. Then there is an isomorphism : K

K
t
over k.
Let k be a eld. By Theorem 8.17 and Corollary 8.20, there is an algebraic eld extension k K
with K algebraically closed, and such K is unique up to k-isomorphism. This K is denoted by k
and called the algebraic closure of k.
Remark 8.21. Let k K be an algebraic eld extension. Then by Theorem 8.19, there is an
embedding : K k over k.
Remark 8.22. Let k K be an algebraic eld extension. Then there is a k-isomorphism
k

= k
K
K.
Example 8.23. 1 = C. The proof will be given later. Also, 1 = C. Notice that is
countable but 1 is not, hence there exist uncountable many real numbers which are transcendental
over !
4. Splitting elds and normal extensions
Let k be a eld, f(X) k[X] k. A splitting eld of f(X) over k is a eld extension k K so
that f(X) (X
1
) (X
n
) in K[X] and K = k(
1
, . . . ,
n
).
Remark 8.24. Every f(X) k[X]k has a splitting eld. Indeed, f(X) (X
1
) (X
n
)
in k[X]. Now take K = k(
1
, . . . ,
n
) k.
Theorem 8.25. Let k be a eld, f(X) k[X] k, K and K
t
splitting elds of f(X) over k.
Then there exists an isomorphism : K

K
t
over k. Furthermore, if K
t
k, then for every
embedding : K k over k, K

= K
t
.
Proof. We rst prove the second part. Let f(X) = c(X
1
) (X
n
) in K[X] with c k,
and f(X) = c(X
1
) (X
n
) in K
t
[X]. Then f = f

= c(X

1
) (X

n
). Thus up to
a permutation,
1
, . . . ,
n
and

1
, . . . ,

n
coincide. Hence K

= k(

1
, . . . ,

n
) = k(
1
, . . . ,
n
) =
K
t
. To show the rst claim, choose a splitting eld K
tt
k and k-embeddings : K k,

t
: K
t
k which exists by Theorem 8.19. Now by the above, K

= K
tt
= K
t

. Thus K

K
t
are isomorphic over k.
Because of Remark 8.24 and Theorem 8.25, every f(X) k[X] k has a splitting eld and this
eld is unique up to k-isomorphism. It is called the splitting eld of f over k.
Let f
i

iI
be a family of polynomials in k[X] k. A splitting eld of f
i

iI
over k is a eld
extension k K so that f
i


(X
ij
) in K[X] and K = k(
ij
). As in the case of
one polynomial one can see that a splitting eld of f
i

iI
over k exists and is unique up to
k-isomorphism. We call it the splitting eld of f
i

iI
over k.
For a nite set f
1
, . . . , f
n
of polynomials in k[X] k, the splitting eld of f
1
, . . . , f
n
over k
equals the splitting eld of
n

i=1
f
i
over k.
An algebraic eld extension k K is normal if every irreducible polynomial p(X) k[X] that has
a root in K splits into linear factors in K[X]. This is equivalent to saying that for every K,
the minimal polynomial of over k splits into linear factors in K[X].
5. SEPARABLE EXTENSIONS 69
Theorem 8.26. Let k K be an algebraic eld extension. Choose k with K k. (Just take
k = K). The following are equivalent:
(a) k K is normal.
(b) K is the splitting eld of a family f
i

iI
k[X] k.
(c) For every embedding : K k over k, K

= K.
Proof. We rst prove that (a) implies (b). For every K let p

(X) k[X] k be the


minimal polynomial of over k. Since p

(X) splits over K into linear factors, we have that K


is the splitting eld of p

(X) over k. Lets now show that (b) implies (c). Let K
i
K be
the splitting eld of f
i
(X) over k. Then by Theorem 8.25, K

i
= K
i
. But K = k(
ij
) with
K
i
= k(
ij
[ i xed). Thus K

= K. Finally, we show that (c) implies (a). Let K and let


p(X) k[X] be the minimal polynomial of over k. Let be any root of p(X) in k. We need
to show that K. Since and are roots of the same irreducible polynomial p(X) k[X],
there exists an embedding : k() k over k mapping to by Proposition 8.18. Now can
be extended to an embedding : K k by Theorem 8.19. Thus is an embedding over k with

= . By assumption K

= K, hence =

= K. So K.
Theorem 8.27.
(a) Let k K L be eld extensions. If k L is normal, then so is K L.
(b) Let k K and k L be eld extensions contained in some eld. If k K is normal
then so is L KL.
(c) Let k K and k L be eld extensions contained in some eld. If k K and k L
are normal then so are k KL and k K L.
Proof. All assertions follow using the characterization of normality in Theorem 8.26(b), ex-
cept for the very last claim. For that we use the denition of normality. So assume k K and
k L are normal, and choose k with KL k. We want to show that k K L is normal. Let
K L and let p(X) be the minimal polynomial of over k. Then every root of p(X) in k is
in K and in L, hence in K L.
Example 8.28.
(1) Let k K be a eld extension with [K: k] = 2. Then k K is normal.
Proof. Let K and let p(X) be the minimal polynomial of over k. We may
assume that , k. Thus deg p(X) = 2 by Proposition 8.5, say p(X) = X
2
+ aX + b =
(X )(X ), where a, b in k, K. But + = a, hence K.
(2) Let
3

2 1. Then (
3

2) is not normal since the minimal polynomial X


3
2 has
also non real roots. In fact K = (
3

2,

3i) C is the splitting eld of X


3
2 over
, hence K is normal. Notice that (
3

2) K with (
3

2) not normal,
hence the class of normal extensions is not distinguished.
(3) Let
4

2 1. Now (

2) and (

2) (
4

2) are eld extensions of degree 2, hence


normal. But (
4

2) is not normal, since the minimal polynomial X


4
2 of
4

2 has
non real roots.
5. Separable extensions
Let k be a eld
(a) A polynomial f k[X] k is called separable if every irreducible factor of f has only
simple roots in the splitting eld of f over k. Otherwise f is called inseparable.
(b) Let k K be a eld extension. An element K that is algebraic over k is called
separable over k if the minimal polynomial of over k is separable. Otherwise it is called
inseparable over k.
(c) An algebraic eld extension k K is called separable if every K is separable over k.
Otherwise it is called inseparable.
Let k K be a eld extension, f(X) k[X], and K. Recall that is a multiple root of f(X)
i.e., (X )
2
[f(X) in K[X] if and only if f() = 0 = f
t
() (see Proposition 7.36).
70 8. FIELDS AND FIELD EXTENSIONS
Lemma 8.29. Let k be a eld, f(X) k[X] k, K the splitting eld of f(X) over k. Then
f(X) has no multiple roots in K if and only if f(X) and f
t
(X) are relatively prime in K[X].
Proof. Suppose that there exists g(X) k[X], deg g(X) 1, with g(X)[f(X) and g(X)[f
t
(X)
in k[X]. Then g(X) has a root K (since deg g(X) 1 and g(X)[f(X)), and then f() = 0 =
f
t
() (since g(X)[f(X) and g(X)[f(X) in K[X]), which forces to be a multiple root of f(X)
in K[X]. Conversely, suppose there exists a multiple root of f(X) in K. Then f() = 0f
t
().
Hence if q(X) denotes the minimal polynomial of over k, q(X)[f(X) and q(X)[f
t
(X) in K[X].
Proposition 8.30. Let k be a eld, q(X) k[X] an irreducible polynomial. Then q(X) is
separable if and only if q
t
(X) ,= 0.
Proof. By Lemma 8.29, q(X) is separable if and only if g.c.d.(q(X), q
t
(X)) 1. But since
q(X) is irreducible and deg q
t
(X) < deg q(X), g.c.d.(q(X), q
t
(X)) 1 if and only if q
t
(X) ,= 0.
Proposition 8.31. Let k be a eld, q(X) k[X] an irreducible polynomial.
(a) If char k = 0, then q
t
(X) ,= 0.
(b) If char k = p > 0, then q
t
(X) ,= 0 if and only if q(X) , k[X
p
].
Proof. We have already seen this in Remark 7.35.
Corollary 8.32. In characteristic zero, every algebraic eld extension is separable.
Proof. Use Proposition 8.30 and Proposition 8.31.
Example 8.33. Let k = Z
p
(Z) be the eld of rational functions over Z
p
. Then q(X) =
X
p
Z k[X] is irreducible (by Eisensteins Criterion, since Z is a prime element in the UFD
Z
p
[Z]), char k = p > 0, and q(X) k[X
p
]. Thus q(X) is not separable by Proposition 8.30, and
k K is an inseparable eld extension, where K denotes the splitting eld of q(X) over k.
Proposition 8.34. Let k K = k(
1
, . . . ,
n
) be a nite eld extension, let K L be a eld
extension where L is algebraically closed, let m
i
the number of distinct roots in L of the minimal
polynomial of
i
over k(
1
, . . . ,
i1
). Then every embedding : k L has exactly m
1
m
n
distinct extensions : K L.
Proof. We induct on n. Let n = 1 and let q(X) be the minimal polynomial of
1
over k.
Then m
1
= #distinct roots of q(X) in L = #distinct roots of q

(X) in L, and then Proposi-


tion 8.18 shows that has exactly m
1
distinct extensions : K = k(
1
) L. Now let n 2. By
induction hypothesis, has exactly m
1
m
n1
extensions to k(
1
, . . . ,
n1
) and each of these
has exactly m
n
extensions to K, which gives exactly m
1
m
n
distinct extensions of to K.
Remark 8.35. Let k K L be eld extensions and L algebraic over k. If is separable
over k, then it is separable over K.
Proof. Let p(X) and q(X) be the minimal polynomial of over k and K, respectively. Then
q(X)[p(X) in K[X]. Now the assertion follows by considering splitting elds of p(X) over k and
q(X) over K in some xed algebraic closure of K.
Theorem 8.36. Let k K be a nite eld extension, let K L be a eld extension where L
is algebraically closed. Let : k L be any embedding.
(a) There are at most [K: k] distinct extensions : K L of .
(b) k K is separable if and only if there are exactly [K: k] distinct extensions : K L
of .
Proof. Write K = k(
1
, . . . ,
n
), let q
i
(X) be the minimal polynomial of
i
over k(
1
, . . . ,
i1
),
and let m
i
be the number of distinct roots of q
i
(X) in L. Notice that m
i
deg q
i
(X), where equal-
ity holds if and only if q
i
(X) is separable (recall that L is algebraically closed), if and only if
i
is separable over k(
i
, . . . ,
i1
). But deg q
i
(X) = [k(
1
, . . . ,
i
): k(
1
, . . . ,
i1
)] by Proposi-
tion 8.5, and
n

i=1
[k(
1
, . . . ,
i
): k(
1
, . . . ,
i1
)] = [k(
1
, . . . ,
n
): k] = [K: k] by Proposition 8.1
5. SEPARABLE EXTENSIONS 71
Now
n

i=1
m
i
[K: k] and equality holds if and only if
i
is separable over k(
1
, . . . ,
i1
) for
1 i n. Now, part (a) follows from Proposition 8.34. As far as (b) is concerned, by Remark 8.35
we have that
i
is separable over k(
1
, . . . ,
i1
) for all 1 i n. Thus
n

i=1
m
i
= [K: k]. Now
use Proposition 8.34. Conversely, let K. We may take
1
to be . Now by Proposition 8.34
and the above,
1
= is separable over k.
Corollary 8.37. Let k K be a nite eld extension, let K L be a eld extension where
L is algebraically closed.
(a) There are at most [K: k] distinct embeddings : K L over k.
(b) The eld extension k K is separable if and only if there are exactly [K: k] distinct
embeddings : K L over k.
Theorem 8.38. Let k K = k(
i

iI
) be an algebraic eld extension. Then k K is
separable if and only if
i
is separable over k for all i I.
Proof. We only need to show the only if part. Every K is contained in k(
i
1
, . . . ,
i
n
)
for a nite subset
i
1
, . . . ,
i
n

i

iI
. Thus we may assume that K = k(
1
, . . . ,
n
) with

i
separable over k hence over k(
1
, . . . ,
i1
). Thus
n

i=1
m
i
= [K: k] (in the notation of Propo-
sition 8.34, see the proof of Theorem 8.36), hence there are exactly [K: k] distinct embeddings
: K K over k (see Proposition 8.34), hence k K is separable (see Corollary 8.37).
Theorem 8.39. Separable algebraic extensions form a distinguished class of eld extensions.
Proof. Let us prove property (1). Let k K L be eld extensions. We want to show that
k L is separable algebraic if and only if k K and K L are. For algebraic we know this by
Proposition 8.9, and for separable the if part follows from Remark 8.35. So assume that k K
and K L are separable algebraic, and show that the algebraic extension k L is separable. Let
L. Let q(X) =
n

i=0
a
i
X
i
be the minimal polynomial of over K (recall that is algebraic over
K), and set K
0
= k(a
0
, . . . , a
n
). Then q(X) is still the minimal polynomial of over K
0
. Thus is
still algebraic and separable over K
0
, thus K
0
K
0
() is nite and separable (see Theorem 8.38).
But k K
0
if nite (Proposition 8.6) and separable (since K
0
K). Now, replacing K by K
0
and
L by K
0
(), we may assume that the extensions k K L are all nite. Now by Corollary 8.37,
there are exactly [K: k] distinct embeddings : K L over k (since k K is separable and
nite), and by Theorem 8.36, each has exactly [L: K] distinct extensions : L L (since
K L is separable and nite). Thus there are exactly [L: K][K: k] = [L: k] distinct embeddings
L L over k. Hence k L is separable by Corollary 8.37.
Let us prove now property (2). Let k K and k L be eld extensions in some eld. We
want to show that if k K is separable algebraic then so is L KL. Again, we know this for
algebraic by Proposition 8.9. Notice that KL = L(
i
), where k(
i
) = K. By Theorem 8.38
it suces to show that
i
are separable over L. So let be in K. Since k L it follows that the
minimal polynomial of over L divides the minimal polynomial of over k (in L[X]), and hence
has to be separable as well.
Let k K be a eld extension: k
sep
= K[ algebraic and separable over k k
K
is called
the separable closure of k in K.
Theorem 8.40. Let k K be a eld extension.
(a) k
sep
is a subeld of K containing k.
(b) (k
sep
)
sep
= k
sep
.
Proof. Obviously k k
sep
. To show that k
sep
is a subeld of K, it suces to check that if
and are in k
sep
, then k(, ) k
sep
. But this follows from Theorem 8.38. This proves (a). Part
(b) follows instead from the transitivity of separability (see Theorem 8.39).
72 8. FIELDS AND FIELD EXTENSIONS
Let k K be an algebraic eld extension: [K: k]
s
= [k
sep
: k] is called the separable degree of K
over k.
A eld k is called perfect if every algebraic eld extension k K is separable.
Example 8.41.
(1) Every eld of characteristic zero is perfect (see Corollary 8.32).
(2) Every algebraically closed eld is perfect.
(3) Z
p
(Z) is not perfect (see Example 8.33).
If k is a eld of characteristic p > 0, recall that F : k k with x x
p
is an embedding (called
the Frobenius homomorphism). Write k
p
= x
p
[ x k = image(F), which is a subeld of k.
Theorem 8.42. A eld k is perfect if and only if
5
char k = 0, or char k = p > 0 and k = k
p
.
Proof. By Corollary 8.32 we may assume that char k = p > 0. Let us suppose that k is a
perfect eld. Let a k and let k be a root of the polynomial X
p
a k[X]. Then
p
= a and
X
p
a = X
p

p
= (X )
p
. Now k k() is separable (since k is perfect), and the minimal
polynomial q(X) of over k divides X
p
a = (X )
p
. Thus deg q(X) = 1, thus k. Hence
every a K has a p-th root k. Conversely, Let q(X) k[X] be irreducible in k[X]. We need
to show that q(X) , k[X
p
] (see Proposition 8.30 and Proposition 8.31). So suppose q(X) k[X
p
].
Then q(X) k[X
p
] = k
p
[X
p
] = F(k[X]). Thus q(X) = F(f(X)) = f(X)
p
for some f(X) k[X],
contradicting the irreducibility of q(X).
Corollary 8.43. Every nite eld is perfect.
Proof. If k is nite, then F is surjective since it is injective. Now use Theorem 8.42
Let k K = k() be a simple eld extension. Then is called a primitive element of the eld
extension.
Theorem 8.44 (Primitive element theorem). Let k K be a nite eld extension.
(a) The eld extension k K is simple if and only if there exist only nitely many elds L
with k L K.
(b) If k K is separable, then it is simple.
Proof. If k is nite then K is nite, K

is cyclic (see Example 7.34(a)), hence k K is


simple. Also, there are only nitely many elds L with k L K. Thus we may from now on
assume that k is innite.
Let us prove the equivalence in (a). Since K = k(
1
, . . . ,
n
), by induction on n, it suces
to show that for , in K there exists in K with k(, ) = k(). For each c k consider the
elds L
c
= k( +c). Since there are only nitely many such elds by assumption and since k is
innite, there exist c
1
,= c
2
in k with L
c
1
= L
c
2
. Thus k( + c
1
) = k( + c
2
). We claim that
k(, ) = k(+c
1
). Since k(+c
1
) = k(+c
2
), we have k(+c
1
) (+c
1
) (+c
2
) =
(c
1
c
2
). But since c
1
,= c
2
we have that (c
1
c
2
)
1
k and hence k( + c
1
). Thus
= ( + c
1
) c
1
k( + c
1
). This shows that k(, ) = k( + c
1
). Conversely, Let
be a primitive element of k K, and let q(X) be the minimal polynomial of over k. For
any eld L with k L K, let q
L
(X) be the minimal polynomial of over L. Then q
L
(X)
is a monic polynomial dividing q(X) in K[X]. There are only nitely many such polynomials
q
L
(X). Thus we are done once we have shown that q
L
(X) uniquely determines L. So, write
q
L
(X) =
n

i=0
a
i
X
i
and L
0
= k(a
0
, . . . , a
n
). Now k L
0
L and q
L
(X) L
0
[X]. Thus q
L
(X)
is also the minimal polynomial of over L
0
. Thus [L
0
(): L
0
] = deg q
L
(X) = [L(): L]. But
K = k() L
0
() L() K, hence L
0
() = L() = K, which gives [K: L
0
] = [K: L]. Thus
since L
0
L, we have L = L
0
and hence L = k(a
0
, . . . , a
n
) is uniquely determined by q
L
(X).
Let us prove (b). It suces to consider the case K = k(, ). Write n = [K: k]. Since
k K is separable by Corollary 8.37 there exist n distinct embeddings
i
: K K over k,
5
This is equivalent to F being an automorphism, which in turn is equivalent to the fact that every element of
K has a p-th root in k.
6. PURELY INSEPARABLE EXTENSIONS 73
1 i n. We will nd c k, so that for = +c, one has
i
() ,=
j
() whenever i ,= j. Then

i[k()
: k() K are n distinct embeddings over k, hence [k(): k] n by Theorem 8.36(a).
Thus K = k(). We now nd c. Since k is innite there exists c in k which is not in the nite
(possibly empty) set
_

i
()
j
()

i
()
j
()
[
i
() ,=
j
()
_
. Thus
i
() ,=
j
() whenever
i
() ,=
j
().
On the other hand, if i ,= j and
i
() =
j
(), then
i
() ,=
j
() (since K = k(, ) and
i
,=
j
)
and hence
i
() =
i
() + c
i
() ,=
j
() + c
i
() =
j
() + c
j
() =
j
(). Thus in either case

i
() ,=
j
() for i ,= j.
Corollary 8.45. In characteristic zero, every nite eld extension is simple.
Example 8.46. (i,

5) = (i +

5).
6. Purely inseparable extensions
In this section k K is a extension of elds of characteristic p.
Proposition 8.47. Let K be algebraic over k. Then for some e 0,
p
e
is separable over
k.
Proof. Let q(X) be the minimal polynomial of over k. Since q(X) is not constant there
exists e 0 with q(X) k[X
p
e
] but q(X) , k[X
p
e+1
]. Thus q(X) = h(X
p
e
) with h(Y ) k[Y ] but
h(Y ) , k[Y
p
]. Now h(Y ) is irreducible (since otherwise h(X
p
e
) = q(X) cannot be irreducible) and
h(
p
e
) = q() = 0. Thus h(Y ) is the minimal polynomial of
p
e
over k. Since h(Y ) , k[Y
p
], it
follows that
p
e
is separable over k (see Proposition 8.30 and Proposition 8.31).
An element K is called purely inseparable over k if
p
e
k for some e 0. The extension
k K is called purely inseparable if every K is purely inseparable over k.
Notice that every k is purely inseparable over k.
Proposition 8.48. Let K be purely inseparable over k, let e 0 be minimal with
p
e
k
and write a =
p
e
. Then X
p
e
a is the minimal polynomial of over k.
Proof. Since X
p
e
a = (X )
p
e
, we know that the minimal polynomial of over k is of
the form (X )
n
k[X], n p
e
. Suppose n < p
e
; then n = p
f
l, with f < e and p [ l. Now
k[X] (X )
n
= (X
p
f

p
f
)
l
= X
p
f
l
l
p
f
X
p
f
(l1)
+ lower terms. Thus l
p
f
k. But l 1 is
a unit in k since p [ l, thus
p
f
k, contradicting the minimality of e.
Corollary 8.49. Let k K be nite and purely inseparable. Then [K: k] = p
n
for some
n 0.
Proof. Write K = k(
1
, . . . ,
s
), and let e
i
0 be minimal with
p
e
i
i
k(
1
, . . . ,
i1
).
Then by Proposition 8.48, [K: k] = p

e
i
.
Corollary 8.50. Let K be separable and purely inseparable over k. Then k.
Proof. By Proposition 8.48, the minimal polynomial of over k is of the form q(X) =
X
p
e
a = (X )
p
e
for some e 0. Now since is separable over k, one has p
e
= 1.
Theorem 8.51. Let k K be an algebraic extension. Then k
sep
K is purely inseparable.
Proof. Let K. By Proposition 8.47, there exists e 0 so that
p
e
is separable over k,
which means
p
e
k
sep
. Thus is purely inseparable over k
sep
.
Notice that by the above theorem, for every algebraic eld extension k K, there exists an
intermediate eld L = k
sep
so that k L is separable and L K is purely inseparable.
Corollary 8.52. The eld extension k K is purely inseparable if and only if k K is
algebraic and the elements of k are the only elements of K separable over k.
Proof. The if part follows from Corollary 8.50. As far as the only if part, by assumption we
have that k = k
sep
and by Theorem 8.51 k
sep
K is purely inseparable.
74 8. FIELDS AND FIELD EXTENSIONS
Let k K be an algebraic eld extension. Then [K: k]
i
= [K: k
sep
] is called the inseparable degree
of K over k.
Lemma 8.53. Let K L be a eld extension where L is algebraically closed, and let : k L
be an embedding. If k K is purely inseparable, then there exists exactly one extension : K L
of .
Proof. The existence follows from Theorem 8.19. As to the uniqueness, let K. Then

p
e
k for some e. Thus F
e
(()) = (())
p
e
= (
p
e
) = (
p
e
) is determined. Now use the fact
that F is injective.
Theorem 8.54. Let k K be a nite eld extension, let K L be a eld extension where L is
algebraically closed, and let : k L be an embedding. Then there are exactly [K: k]
s
distinct
embeddings : K L over .
Proof. Recall that [K: k]
s
= [k
sep
: k] with k k
sep
nite and separable. Hence by Theo-
rem 8.36(b), there are exactly [K: k]
s
= [k
sep
: k] distinct embeddings : k
sep
L over , each of
which has exactly one extension : K L by Lemma 8.53 since k
sep
K is purely inseparable
by Theorem 8.51.
Corollary 8.55. Let k K L be nite eld extensions. Then
[L: k]
s
= [L: K]
s
[K: k]
s
and [L: k]
i
= [L: K]
i
[K: k]
i
.
Proof. Consider id: k L and use Theorem 8.54.
Proposition 8.56.
(a) Let k K = k(
i

iI
) with
i
purely inseparable over k. Then k K is purely
inseparable.
(b) Purely inseparable extensions form a distinguished class of eld extensions.
Proof. This follows immediately from the denition.
Theorem 8.57. Let k K be algebraic.
(a) If k K is separable, then K = kK
p
n
for every n 1.
(b) Assume that k K is nite. If K = kK
p
, then k K is separable.
Proof. The extension kK
p
n
K is separable by Theorem 8.39 and purely inseparable, hence
kK
p
n
= K by Corollary 8.50. This proves (a). As far as (b) is concerned, write K = k(
1
, . . . ,
r
).
By Theorem 8.51,
p
e
i
i
k
sep
for some e
i
. Thus, taking e = maxe
i
[ 1 i r we have
p
e
i
k
sep
for all i. Thus k
sep
k(
p
e
1
, . . . ,
p
e
r
) = kK
p
e
= K.
Proposition 8.58. Let K be algebraic over k, let q(X) be the minimal polynomial of
over k, and let
1
, . . . ,
s
be the distinct roots of q(X) in K. Then
(a) q(X) =
s

j=1
(X
j
)
[k(): k]
i
and s = [k(): k]
s
.
(b)
[k(): k]
i
is separable over k.
Proof. Let e be maximal so that q(X) k[X
p
e
]. Then q(X) = h(X
p
e
) for some h(Y ) k[Y ]
where h(Y ) is separable and irreducible (see the proof of Proposition 8.47). Let d = deg h(Y ) so
that deg q(X) = dp
e
. Then h(Y ) has d distinct roots a
1
, . . . , a
d
in K, and these are separable over
k. Let
j
be the p
e
-th root of a
j
in K. Then
1
, . . . ,
d
are distinct, and
p
e
j
are separable over k.
Now q(X) = h(X
p
e
) =
d

j=1
(X
p
e
a
j
) =
d

j=1
(X
j
)
p
e
. This shows
1
, . . . ,
d
=
1
, . . . ,
s
,
d = s, and
p
e
is separable. It remains to show that d = [k(): k]
s
and p
e
= [k(): k]
i
. However,
k(
p
e
) k
sep
, and k(
p
e
) k() is purely inseparable. Thus k(
p
e
) = k
sep
by Corollary 8.50.
Thus [k(): k]
s
= [k
sep
: k] = [k(
p
e
): k] = deg h(Y ) = d. Now [k(): k]
i
= [k(): k]/[k(): k]
s
=
deg q(X)/d = p
e
.
7. FINITE FIELDS 75
7. Finite elds
Recall that if k is a nite eld, then char k = p > 0 is prime, Z
p
k, and [k[ = p
n
for some n 1.
Theorem 8.59. Let p be a prime and n 1. Then k is a eld with [k[ = p
n
if and only if k
is isomorphic to a splitting eld of X
p
n
X over Z
p
.
Proof. Notice that f(X) = X
p
n
X Z
p
[X] has no multiple roots in any eld extension of
Z
p
since f
t
(X) = 1 is a non-zero constant polynomial (see Proposition 7.36). Suppose [k[ = p
n
.
Since k

is a multiplicative group of order p


n
1, we have
p
n
1
= 1 for all k

. Hence
p
n
=
for all k

, and hence also for all k. Thus every element of k is a root of X


p
n
X. Since
[k[ = p
n
= deg(X
p
n
X), it then follows that k is a splitting eld of X
p
n
X over Z
p
. Conversely,
let k be a splitting eld of X
p
n
X over Z
p
, and let S = roots of X
p
n
X k. Then [S[ = p
n
since f(X) = X
p
n
X has no multiple roots in k. It is easy to see that the set S is a subeld of
k. Thus S = k. Hence [k[ = [S[ = p
n
.
Notice that if k is as in Theorem 8.59, then the proof shows that every element of k is a root of
X
p
n
X.
Corollary 8.60. Let p be a prime and n 1. Then there exists a eld k with [k[ = p
n
. Such
k is unique up to isomorphism (and denoted by F
q
where q = p
n
).
Proof. By Theorem 8.59, it suces to show that a splitting eld of X
p
n
X over Z
p
exists
and is unique up to isomorphism. But this follows from Remark 8.24 and Theorem 8.25.
Corollary 8.61. Let k be a nite eld, and let N 1 be any integer. Then there exists a
eld extension k K with [K: k] = N. Any such K is unique up to k-isomorphisms.
Proof. Write [k[ = p
n
. To show the existence, let K be the splitting eld of X
p
nN
X over
k. However, for all k,
p
n
= , hence
p
nN
= ((
p
n
)
p
n
)
p
n
= , which shows that every
element of k is a root of X
p
nN
X. Thus K is also the splitting eld of X
p
nN
X over F
p
, where
F
p
k. Thus [K[ = p
nN
by Theorem 8.59, and hence [K: k] = N. To show the uniqueness, notice
that for k K with [K: k] = N, one has [K[ = p
nN
. Thus K is a splitting eld of X
p
nN
X
over F
p
(see Theorem 8.59). So K is a splitting eld of X
p
nN
X over k. Hence K is unique up
to k-isomorphism by Theorem 8.25.
76 8. FIELDS AND FIELD EXTENSIONS
Homework set # 9.
Choose ve of the following problems. The due date is on March 3 (Monday), 2002.
1. Let K = (

2,

3) 1. Determine [K: ] and nd u K with K = (u).


2. Let k K, k L be eld extensions contained in some eld. Show that
[KL: k] [K: k][L: k].
[KL: L] [K: k].
3. Let k K and k L be nite eld extensions contained in some eld. Show that if
[KL: k] = [K: k][L: k] then K L = k. Is the converse true?
4. Let k K be an algebraic eld extension. Show that every embedding : K K over
k is an isomorphism.
5. Show that arccos 11/16 can be trisected with ruler and compass.
6. Prove that the regular pentagon can be constructed with ruler and compass.
Prove that the regular 9-gon cannot be constructed with ruler and compass.
7. Let K = k() be an algebraic extension of odd degree. Show that K = k(
2
).
8. Let k be a eld with char k ,= 2 and let a, b k. Let
2
= a and
2
= b. Assume that
, have degree 2 over k, and both lie in some extension of k. Prove that k() = k() if
and only if there exists c k such that a = c
2
b.
9. Let k be a eld, k(X) the eld of rational functions, u k(X) k. Write u = f/g with f
and g relatively prime in k[X]. Show that [k(X): k(u)] = maxdeg f, deg g.
10. Let k be a eld, let f k[X] be a polynomial of degree n 1, and let K be the splitting
eld of f over k. Show that [K: k] divides n!
11. Determine the splitting elds of the following polynomials over :
X
3
2; X
8
5; X
6
+X
3
+ 1.
Homework set # 10.
Choose ve of the following problems. The due date is on April 1 (Monday), 2002.
1. Let k be a eld and K = k(X) be the eld of rational functions. Show that (X) =
aX +b
cX +d
with a, b, c, d in k and ad bc ,= 0 determines an automorphism of K over k.
Conversely, show that every automorphism of K over k is uniquely determined by such
a rational function.
2. Let k K be a nite eld extension. Show that k is perfect if and only if K is perfect.
3. Let k be a eld of characteristic p > 0, k(X, Y ) the eld of rational functions in two
variables.
Show that [k(X, Y ): k(X
p
, Y
p
)] = p
2
.
Find innitely many distinct elds L with k(X
p
, Y
p
) L k(X, Y ).
4. Let K be the splitting eld of X
p
X 1 over k = Z
p
. Show that k K is normal,
separable, of degree p.
5. Let k K be an extension of elds of characteristic p > 0. Show that K is purely
inseparable over k if and only if the minimal polynomial of over k has only one root in
K.
6. Let k K be a nite extension of elds of characteristic p > 0. Show that if p [ [K: k],
then k K is separable. Is the converse true?
7. Let k K be an algebraic extension of elds of characteristic p > 0, let L be an al-
gebraically closed eld containing K, and let : k L be an embedding. Show that
k K is purely inseparable if and only if there exists exactly one embedding : K L
over .
8. Let k K = k(, ) be an algebraic extension of elds of characteristic p > 0, where
is separable over k and is purely inseparable over k. Show that K = k( +).
9. Let f(X) F
q
[X] be irreducible. Show that f(X) divides X
q
n
X if and only if deg f(X)
divides n.
10. Show that in a nite eld, every element can be written as a sum of two perfect squares.
CHAPTER 9
Galois Theory
1. Galois extensions
Let k K be a eld extension. Then Aut
k
(K) = automorphisms of the eld K over k is a
subgroup of Aut(K) = automorphisms of the eld K. Notice that
(1) Aut() = id, Aut(F
p
) = id.
(2) If k is the prime eld of K then Aut
k
(K) = Aut(K).
(3) Aut
1
(C)

= Z
2
.
Let Aut
1
(C). Then (a + ib) = a + (i)b for any a, b 1. On the other hand
i
2
= 1, so that ((i))
2
= (i
2
) = (1) = 1. Hence either (i) = i, in which
case is the identity automorphism, or (i) = i, in which case is the conjugation
automorphism. Thus there are only two distinct automorphisms of C over 1.
(4) Aut

(1) = Aut(1) = id.


By (2), it is enough to show the second equality. Let Aut(1). If a 1
+
then
a = b
2
for some b 1. Thus (a) = (b
2
) = ((b))
2
1
+
and hence a > b implies
(a) > (b). In other words, any such preserves the total order of 1. On the other
hand (p/q) = p/q for and p/q . Moreover, it is well known that is dense in 1, so
that any a R is trapped in between rational numbers, say p/q < a < p
t
/q
t
. But then
p/q = (p/q) < (a) < (p
t
/q
t
) = p
t
/q
t
, that is (a) is trapped between the same rational
numbers as a. Thus (a) = a for any a 1.
(5) Let k be a eld and K = k(X) be the eld of rational functions. Then, see HW 10.1, we
have that Aut
k
(K) =
_
(X) =
aX +b
cX +d
[ a, b, c, d k, ad bc ,= 0
_

= PGL
2
(k).
Proposition 9.1. Let k K be an extension of nite elds. Write [K: k] = N, [k: F
p
] = n.
Then Aut
k
(K) = F
n
is cyclic of order N.
Proof. By Theorem 8.36(a), [Aut
k
(K)[ [embeddings : K K over k[ [K: k]; and
by Corollary 8.43, F Aut(K) and then F
n
Aut(K). Also, for all k,
p
n
= , thus
F
n
() = , which gives F
n
Aut
k
(K). So it suces to show that [F
n
[ N. Suppose (F
n
)
s
= id,
then every element of K is a root of X
p
ns
X, hence [K[ p
ns
, hence s N.
Corollary 9.2. Let q = p
N
. Then Aut(F
q
) = F is cyclic of order N.
An algebraic extension k K is called Galois if it is separable and normal.
Proposition 9.3. Let k K be a nite eld extension. Then k K is Galois if and only if
[Aut
k
(K)[ = [K: k].
Proof. One has [Aut
k
(K)[ [embeddings : K K over k[, where equality holds if and
only if k K is normal Theorem 8.26. Furthermore embeddings : K K over k[ [K: k],
where equality holds if and only if k K is separable Corollary8.37(b).
If k K is Galois then G(K/k) = Aut
k
(K) is called the Galois group of K over k. Notice that if
k K is nite and Galois, then [G(K/k)[ = [K: k].
Example 9.4.
(1) k K = k is Galois with G(K/k) = id
k
.
(2) In characteristic zero, every normal extension is Galois.
77
78 9. GALOIS THEORY
(3) Let k K be an extension of nite elds with [k[ = p
n
. Then k K is Galois with
G(K/k) = F
n
(use Proposition 9.1 and Proposition 9.3).
(4) Let k _ K be a purely inseparable extension. Then k K is normal, but not Galois,
Aut
k
(K) = id
k
(use Proposition 8.48, Corollary 8.50, Lemma 8.53).
(5) (
3

2) is separable, but not Galois, Aut

((
3

2)) = id

.
Let K be a eld, G a subgroup of Aut(K). Then K
G
= K[

= for all G is called


the xed eld of G. Notice that
(1) K
G
is a subeld of K;
(2) If G
1
< G
2
< Aut(K), then K
G
1
K
G
2
.
Theorem 9.5 (Fundamental Theorem of Galois Theory). Let k K be a nite Galois exten-
sion with G = G(K/k).
(a) There is an order reversing bijection (Galois correspondence)
L[ k L K, L subeld H[ H subgroup of G
given by L G(K/L) and K
H
. H.
(b) Under this correspondence, k L is a normal eld extension if and only if H = G(K/L)
is a normal subgroup of G, in which case G/H

= G(L/k) via the map induced by
[L
,
G.
Remark 9.6.
(1) We had already seen that the correspondence in Theorem 9.5(a) is order reversing.
(2) If k K is Galois and k L K, then L K is Galois (Theorem 8.27(a) and Theo-
rem 8.39). Hence it makes sense to consider G(K/L). Furthermore by Proposition 9.3,
[K: L] = [G(K/L)[. Thus [L: k] = [G: G(K/L)].
(3) Pictorially
K G = G(K/k)
[ [
L
2
H
1
= G(K/L
1
)
[ [
L
1
H
2
= G(K/L
2
)
[ [
k 1 = G(K/K)
Theorem 9.7. Let k K be a Galois extension with G = G(K/k).
(a) K
G
= k;
(b) L[ k L K, L subeld H[ H subgroup of G with L G(K/L) is an injective
map with left inverse K
H
. H.
Proof. In order to prove (a), let K k. We will nd G(K/k) with

,= . This
will show that , K
G
. However, since is separable over k, there exists [k(): k] > 1 distinct
embeddings : k() K over k (Corollary 8.37(b)). Thus

,= for one such . Now can


be extended to an embedding : K K (Theorem 8.19). Since k K is normal and is an
embedding over k, (K) = K (Theorem 8.26). Thus G(K/k). But

,= . We now
prove part (b). For every L with k L K, we have that L K is Galois (Theorem 8.27(a) and
Theorem 8.39). Thus by part (a), we have have that L = K
G(K/L)
, which shows that the map
H[ H subgroups of G L[ k L K, L subeld given by H K
H
is a left inverse of
the map in (b).
Corollary 9.8. With the assumptions of Theorem 9.7, let L
i
be subelds of L containing k,
and let H
i
= G(K/L
i
) be the corresponding subgroups of G.
(a) G(K/L
1
L
2
) = H
1
H
2
and L
1
L
2
= K
H
1
H
2
;
(b) L
1
L
2
if and only if H
1
H
2
.
Proof. As far as (a) is concerned, the rst assertion is clear while the second one follows from
the rst via Theorem 9.7(b). Now, if L
1
L
2
it is clear that H
1
H
2
. The converse follows via
Theorem 9.7(b). This completes the proof of (b).
1. GALOIS EXTENSIONS 79
Corollary 9.9. With the assumptions of Theorem 9.7, every subeld L of K containing k is
the xed eld of some subgroup of G(K/k).
Proof. This is immediate from Theorem 9.7(a).
Corollary 9.10. Let k K be a nite separable extension. Then there are only nitely many
subelds L of K containing k (see also Theorem 8.44).
Proof. Let K = k(
1
, . . . ,
t
), q
i
(X) the minimal polynomial of
i
over k, and K
t
a splitting
eld of q
1
(X), . . . , q
t
(X) over K. Then K
t
is also a splitting eld of q
1
(X), . . . , q
t
(X) over
k, and thus k K
t
is normal (Theorem 8.26). Now k K
t
is still nite and separable. Thus
k K K
t
, where k K
t
is nite and Galois. Now G(K
t
/k) is a nite group (Theorem 8.36(a)),
hence has only nitely many subgroups. Thus by Theorem 9.7(b), k K
t
has only nitely many
intermediate elds. Hence the same holds for k K.
Lemma 9.11. Let k K be an algebraic separable extension. Assume that for some positive
integer n, every K has degree at most n. Then [K: k] n.
Proof. Let K be an element of maximal degree over k, say m (where m n). We claim
that K = k(). Suppose not, then there exists K k(). Now [k(, ): k] > [k(): k] = m.
But since k k(, ) is nite and separable, by the Primitive Element Theorem (Theorem 8.44(b)),
k(, ) = k() for some . Now has degree > m, contradicting the maximality of m.
Lemma 9.12. Let K be a eld, K, G a nite subgroup of Aut(K), and let

1
, . . . ,

r
=

[ G. Then f(X) =
r

i=1
(X

i
) is a separable polynomial of degree [G[ in K
G
[X] with
f() = 0.
Proof. We only need to prove that f(X) K
G
[X], or equivalently, f

(X) = f(X) for all


G. But f

(X) =
r

i=1
(X

) =
r

i=1
(X

i
) = f(X), because

, . . . ,

1
, . . . ,

since G

= G.
Theorem 9.13. Let K be a eld, G a nite subgroup of Aut(K) with [G[ = n, and k = K
G
.
Then k K is a nite Galois extension with [K: k] = n, and G(K/k) = G.
Proof. By Lemma 9.12, every element K is algebraic, separable, and of degree n over
k. Thus by Lemma 9.11, [K: k] n. By denition, Aut
k
(K) G. Thus [K: k] n = [G[
[Aut
k
(K)[ [K: k], where the latter inequality follows from Theorem 8.36(a). Hence [K: k] = n
and Aut
k
(K) = G. Now [Aut
k
(K)[ = [K: k], hence k K is Galois by Proposition 9.3 (this also
follows from Lemma 9.12).
Corollary 9.14. Let k K be a nite Galois extension with G = G(K/k). Then the map
L[ k L K, L subeld H[ H subgroup of G with L G(K/L) is a surjective map
with right inverse K
H
. H.
Proof. First notice that the map K
H
. H is well dened since K
H
K
G
k. We need to
check that G(K/K
H
) = H. This follows from Theorem 9.13 since H G is nite.
Notice that Theorem 9.7(b) and Corollary 9.14 complete the proof of Theorem 9.5(a).
Corollary 9.15. Let k K be a nite normal extension with G = Aut
k
(K). Then k K
G
is purely inseparable with [K
G
: k] = [K: k]
i
and K
G
K is separable with [K: K
G
] = [K: k]
s
.
Furthermore, K = k
sep
K
G
and k
sep
K
G
= k
K = k
sep
K
G
p.insep sep
k
sep
K
G
sep p.insep
k = k
sep
K
G
80 9. GALOIS THEORY
Proof. By Theorem 9.13, K
G
K is separable with [K: K
G
] = [Aut
k
(K)[. But since k K
is normal, by Proposition 8.30, [Aut
k
(K)[ = [embeddings : K K over k[, where the latter is
[K: k]
s
by Theorem 8.54. Thus [K: K
G
] = [K: k]
s
. Hence [K
G
: k] = [K: k]
i
. By Corollary 8.55
we have that [K: k]
i
= [K: K
G
]
i
[K
G
: k]
i
which is [K
G
: k]
i
since K
G
K is separable. Now
[K
G
: k] = [K
G
: k]
i
, which shows that k K
G
is purely inseparable. Since k
sep
K is purely
inseparable (Theorem 8.51), k
sep
K
G
K is purely inseparable (Proposition 8.56(b)). Since K
G

K is separable, k
sep
K
G
K is separable (Theorem 8.39). So k
sep
K
G
K is separable and purely
inseparable, thus k
sep
K
G
= K (Corollary 8.50). Likewise k
sep
K
G
= k.
Lemma 9.16. Let k L K be elds, G = Aut
k
(K), G. Then Aut
(L)
(K) =
Aut
L
(K)
1
(i.e., Aut
(L)
(K) and Aut
L
(K) are conjugate in G).
Proof. The inclusion Aut
L
(K)
1
Aut
(L)
(K) is immediate from the denition. Ap-
plying this to the situation k (L) K and
1
G, one obtains
1
Aut
(L)
(K) (
1
)
1

Aut

1
((L))
(K) = Aut
L
(K). Thus Aut
(L)
(K) Aut
L
(K)
1
.
Theorem 9.17. Let k K be a Galois extension with G = G(K/k), and let k L K, L
a subeld of K, with H = G(K/L). Then k L is a normal eld extension if and only if H is a
normal subgroup of G. If this is the case, then the restriction map
[L
gives an epimorphism
G G(L/k), whose kernel is H; in particular, G(L/k)

= G(K/k)/G(K/L).
Proof. Assume that k L is normal. Let : G G(L/k) be given by () =
[L
. This
map is well dened since k L is normal and therefore (L) = L (Theorem 8.26). Also, is
clearly a homomorphism of groups. To see that is surjective, let G(L/k). By Theorem 8.19,
there exists an embedding : K K over . As is an embedding over k and k K is normal,
we have (K) = K (Theorem 8.26), hence G. But
[L
= , which means that () = . Thus
is an epimorphism of groups. It follows from the denition that Ker() = G(K/L) = H. Thus H
is a normal subgroup of G and G/H

= Im() = G(L/k). It remains to show that k L is normal
if H is a normal subgroup of G. By Theorem 8.26, we need to check that (L) = L for every
embedding : L K over k. Again, by Theorem 8.19 and Theorem 8.26, there exists G
with
[L
= . Since HG, we have that H
1
= H. But by Lemma 9.16, H
1
= G(K/(L)).
Thus G(K/(L)) = G(K/L). Hence by Theorem 9.7(b), (L) = L (notice that k (L)). Hence
(L) = L.
Theorem 9.17 completes the proof of Theorem 9.5(b)
Let k K be a Galois extension: We say that k K is Abelian or cyclic if G(K/k) is an Abelian
or cyclic group.
Corollary 9.18. Let k K be a Galois extension that is Abelian (or cyclic). Then for every
eld L, k L K, the extension k L is normal, and k L, L K are Abelian (or cyclic).
Proof. This follows from Theorem 9.17.
Theorem 9.19. Let k K be a nite Galois extension, k L any eld extension with K and
L contained in a common eld. Then L KL and K L K are nite Galois extensions, and
there is an isomorphism G(KL/L)

= G(K/K L) induced by
[K
.
Proof. That L KL and KL K are nite Galois follows from Remark 8.8, Theorem 8.27,
Theorem 8.39. Now let G(KL/L); then
[K
: K KL is an embedding over K L. Thus
(K) = K because KL K is normal (Theorem 8.26). Thus : G(KL/L) G(K/KL) is
well-dened. Obviously is a homomorphism of groups with Ker() = id
KL
. It remains to show
that is surjective. Write H = Im(). Then K
H
= K[
[K
() = for all G(KL/L) =
K[ () = for all G(KL/L) K (KL)
G(KL/L)
= K L, where the last equality
follows from Theorem 9.7(b). Now K
H
= KL implies by Theorem 9.13 that H = G(K/KL),
proving the surjectivity of .
Corollary 9.20. With the assumptions of Theorem 9.19, [KL: L] divides [K: k].
2. THE GALOIS GROUP OF A POLYNOMIAL 81
Proof. By Proposition 9.3 and Theorem 9.19 we have that [KL: L] = [G(KL/L)[ = [G(K/K
L)[ = [K: K L] divides [K: k].
Example 9.21. If k K is not normal, then Corollary 9.20 may fail. Let k = , K = (
3

2)
1, L = (
3

2) C with = e
2/3i
, which is a root of X
2
+X+1. Then [K: k] = 3 = [L: k], since
X
3
2 is the minimal polynomial of
3

2 and
3

2 over . But KL = (
3

2,
3

2) = (
3

2, ),
which has degree 6 over . Thus [KL: L] = 2 does not divide 3 = [K: k].
Theorem 9.22 (Fundamental Theorem of Algebra). C is algebraically closed.
Proof. Let C K be a nite eld extension. We need to show that K = C. Since 1 K
is nite and separable (Corollary 8.32), K = 1() (by Theorem 8.44). Let L be the splitting
eld of the minimal polynomial of over 1. Then C K L, and we may replace K by
L to assume that 1 K is nite and Galois. Write G = G(K/1). Then G is a nite group
and hence has a 2-Sylow subgroup H. Let L = K
H
. Then 1 L K and [L: 1] = [G: H]
(Proposition 9.3), which is odd since H is a 2-Sylow subgroup. Let be a primitive element
of L over 1 (which exists by Theorem 8.44), and let q(X) be the minimal polynomial of over
1. Then deg q(X) = [L: 1] is odd. But every polynomial of odd degree over 1 has a root
in 1, by the Intermediate Value Theorem. Thus deg q(X) = 1 since q(X) is irreducible over
1. Thus 1 = deg q(X) = [L: 1] = [G: H], which gives G = H. Thus [K: 1] is a power of 2
(Proposition 9.3), and hence [K: C] is a power of 2, say [K: C] = 2
n
.
Now let G = G(K/C) (notice that C K is still Galois). Then [G[ = 2
n
(Proposition 9.3).
Suppose n > 0. Then G has a subgroup H with [H[ = 2
n1
(by Sylows Theorem, Part I). Let
L = K
H
. Then C L K and [L: C] = [G: H] = 2 (Proposition 9.3). Thus there would exist
an irreducible quadratic polynomial over C. But this is impossible by the quadratic formula since
every z C has a square root in C: Let z = a + ib, a, b 1; then the square root of z in C is
given by c + id where c, d 1 with c
2
= 1/2(a +

a
2
+b
2
) and d
2
= 1/2(a +

a
2
+b
2
) (notice
that c, d exist) and cd > 0 if and only if b > 0.
Corollary 9.23. Let 1 K be an algebraic eld extension. Then K = 1 or K is isomorphic
over 1 to C.
Proof. Use Remark 8.21, Theorem 9.22 and the fact that [C: 1] = 2.
2. The Galois group of a polynomial
Let k be a eld and f(X) k[X] k be a separable polynomial. The Galois group of f(X) is the
group G(K/k), where K is a splitting eld of f(X) over k. Notice that k K is a nite Galois
extension (Theorem 8.26), and K is unique up to k-isomorphism (Theorem 8.25).
Proposition 9.24. Let k be a eld, f(X) k[X] a separable polynomial of degree n 1 with
Galois group G.
(a) G is isomorphic to a subgroup of S
n
.
(b) If f(X) is irreducible, then G is isomorphic to a transitive
1
subgroup of S
n
.
Proof. Let K be a splitting eld of f(X) over k, and let
1
, . . . ,
m
be the distinct roots
of f(X) in K. Then K = k(
1
, . . . ,
m
), and m n with m = n in (b). Now let G.
Then (
1
, . . . ,
m
)
1
, . . . ,
m
, thus (
1
, . . . ,
m
) =
1
, . . . ,
m
by Remark 8.14.
Thus F : G Sym(
1
, . . .
m
) with F() =
[
1
,...,
m

is a well dened map. Clearly F is


a homomorphism of groups and injective. Now (a) follows since Sym(
1
, . . . ,
m
)

= S
m
S
n
(since m n). In (b), m = n, and we only need to show that F(G) is transitive. For i, j there
exists a k-isomorphism : k(
i
)

k(
j
) over k with (
i
) =
j
since
i
and
j
have the same
minimal polynomial over k (Proposition 8.18). Now can be extended to G(K/k) since k K
is normal (Theorem 8.19 and Theorem 8.26).
1
Let G < S
n
. We say that G is transitive if for all 1 i ,= j n there exists with (i) = j. This is equivalent
to saying that G acts transitively on |1, . . . , n.
82 9. GALOIS THEORY
Corollary 9.25. Let f(X) [X] be an irreducible polynomial with deg f(X) = p prime
and assume that f(X) has exactly two non real roots in C. Then the Galois group of f(X) is
isomorphic to S
p
.
Proof. Let K C be the splitting eld of f(X) over (notice that C is algebraically closed
by Theorem 9.22) and write G for G(K/). Let : C C denote complex conjugation. Then
Aut

(C) and then =


[K
G since K is normal (Theorem 8.26). Now interchanges
the two non real roots of f(X) and leaves the other roots xed. Thus is a transposition,
when identifying G with a transitive subgroup of S
p
via Proposition 9.24(b). But every transitive
subgroup of S
p
containing a transposition has to be S
p
.
Example 9.26. The Galois group of 2X
5
10X + 5 over is isomorphic to S
5
.
Remark 9.27. Let f(X) k[X] be an irreducible separable polynomial with Galois group G.
(a) If deg f(X) = 2, then G

= Z
2
.
(b) If deg f(X) = 3, then G

= Z
3
or G

= S
3
.
Proof. If deg f(X) = n, then [G[ = [K: k] n by Proposition 9.3 and G S
n
by Proposi-
tion 9.24.
Let f(X) k[X] be a polynomial of degree n 1 with (not necessarily distinct) roots
i
in a
splitting eld K, and write
=

i<j
(
i

j
).
Then D = D
f
=
2
is called the discriminant of f.
Proposition 9.28. In addition to the above notation, let f(X) be separable with Galois group
G S
n
.
(a) D
f
k.
(b) Let G. If A
n
then () = , and if , A
n
then () = .
Proof. Part (b) has been proved in Chapter 5. As far as (a) is concerned, by (b) we have
that (D
f
) = (())
2
= ()
2
= D
f
for all G. Thus D
f
K
G
. But f(X) is separable,
hence k K is nite Galois, hence K
G
= k by Theorem 9.5(a).
Corollary 9.29. In addition to the notation of Proposition 9.28, assume that char(k) ,= 2
and that
1
, . . . ,
n
are distinct (if and only if D
f
,= 0). Then k() = K
GA
n
and G(K/k()) =
G A
n
. In particular, G A
n
if and only if k (if and only if D
f
is a perfect square in k).
Proof. Notice that k K is nite Galois. Thus by Theorem 9.5, it suces to show that
G(K/k()) = G A
n
. Let G. Then G(K/k()) if and only if () = if and only if
A
n
. The last equivalence follows from Proposition 9.28(b) because ,= , since ,= 0 and
char(k) ,= 2.
2.1. Degree three.
Corollary 9.30. Let char(k) ,= 2, and let f(X) k[X] be an irreducible separable polynomial
of degree 3 with Galois group G. Then G

= Z
3
if and only if D
f
is a perfect square in k. Otherwise
G

= S
3
.
Proof. With G S
3
one has G = A
3

= Z
3
or G = S
3
. Now use Corollary 9.29.
If char(k) ,= 3 and f(X) = X
3
+a
2
X
2
+a
1
X +a
0
then h(X) = f(Xa
2
/3) = X
3
+aX +b. This
is called completing the cube, or Tschirnhausen transformation. Notice that

h
=

i<j
((
i
+a
2
/3) (
j
+a
2
/3)) =

i<j
(
i

j
) =
f
,
and D
h
= D
f
.
Proposition 9.31. Let h(X) = X
3
+aX +b. Then D
h
= 4a
3
27b
2
.
2. THE GALOIS GROUP OF A POLYNOMIAL 83
Proof. Write h(X) = X
3
+aX +b = (X
1
)(X
2
)(X
3
) and observe that

1
+
2
+
3
= 0
1

2
+
1

3
+
2

3
= a
1

3
= b.
Moreover, we also have that
2
1
+
2
2
+
2
3
= (
1
+
2
+
3
)
2
2(
1

2
+
1

3
+
2

3
) = 2a and
similarly we have that
2
1

2
2
+
2
1

2
3
+
2
2

2
3
= (
1

2
+
1

3
+
2

3
)
2
2
1

3
(
1
+
2
+
3
) = a
2
.
On the other hand, we have that
h
t
(X) = (X
2
)(X
3
) + (X
1
)(X
3
) + (X
1
)(X
2
)
so that D
h
= h
t
(
1
)h
t
(
2
)h
t
(
3
). Since h
t
(X) = 3X
2
+a, a quick calculation shows that
D
h
= h
t
(
1
)h
t
(
2
)h
t
(
3
)
= [3
2
1
+a][3
2
2
+a][3
2
3
+a]
= [27
2
1

2
2

2
3
+ 9a(
2
1

2
2
+
2
1

2
3
+
2
2

2
3
) + 3a
2
(
2
1
+
2
2
+
2
3
) +a
3
]
= [27(b)
2
+ 9aa
2
+ 3a
2
(2a) +a
3
]
= 27b
2
4a
3
as desired.
Example 9.32.
(1) Let f(X) = X
3
X+1 [X]. Now 1 is not a root of f(X). If f(X) would be reducible
in [X] then f(X) would be reducible in Z[X], hence would have a root in Z, which would
be 1. So f(X) is irreducible in [X]. By Proposition 9.31, D
f
= 4 27 = 23, which
is not a square. Thus G

= S
3
by Corollary 9.30.
(2) Let f(X) = X
3
+3X
2
1 [X]. Now f(X) is irreducible by the same reasoning as above.
We have h(X) = f(X1) = X
3
3X+1. By Proposition 9.31, D
f
= D
h
= 42727 = 81,
which is a perfect square. Thus G

= Z
3
by Corollary 9.30.
2.2. Degree four. Recall that S
4
H = id, (1 2)(3 4), (1 3)(2 4), (1 4)(2 3)

= Z
2
Z
2
. Hence
for G < S
4
one has GH G.
Lemma 9.33. Let f(X) k[X] be a polynomial of degree 4 with distinct roots
1
, . . . ,
4
in
a splitting eld K. Let G = G(K/k) S
4
and write u =
1

2
+
3

4
, v =
1

3
+
2

4
and
w =
1

4
+
2

3
. Then k(u, v, w) = K
GH
and G(K/k(u, v, w)) = G H.
Proof. Since k K is nite Galois, by Theorem 9.5, it suces to show that G(K/k(u, v, w)) =
G H. It is clear that G(K/k(u, v, w)) G H. To show the other inclusion we need to
show that every 2-cycle, 3-cycle, 4-cycle does not x u, v and w. By symmetry it suces to
consider (1 2), (1 2 3), (1 3 2 4). But (1 3 2 4) = (1 2)(1 3)(2 4) (1 2)H. So we only need to
check (1 2) and (1 2 3). Suppose that (1 2) leaves v xed. Then
1

3
+
2

4
=
2

3
+
1

4
,
which gives (
1

2
)(
3

4
) = 0, which is impossible. Suppose (1 2 3) leaves u xed. Then

2
+
3

4
=
2

3
+
1

4
, which gives (
1

3
)(
2

4
) = 0, which is impossible.
Lemma 9.34. With the notation of Lemma 9.33, (Xu)(Xv)(Xw) k[X]. In particular,
k(u, v, w) is the splitting eld of this polynomial (called the resolvant cubic of f(X)) over k.
Proof. It suces to show that every transposition of S
4
leaves the polynomial xed, and by
symmetry we only need to consider (1 2). But (1 2) leaves u xed and exchanges v and w.
Lemma 9.35. The resolvant cubic of X
4
+aX
2
+bX +c is X
3
aX
2
4cX + 4ac b
2
.
Proof. Express a, b, c and u, v, w in terms of
1
, . . . ,
4
.
Let f(X) be a polynomial of degree 4 with
1
, . . . ,
4
distinct.
K
[
K
HG
= k(u, v, w) = splitting eld of the resolvant cubic
[
(char(k) ,= 2) K
A
4
G
= k() = splitting eld of X
2
D
f
[
k
84 9. GALOIS THEORY
Theorem 9.36. Let f(X) k[X] be an irreducible separable polynomial of degree 4 with
Galois group G S
4
. Let k(u, v, w) be the splitting eld of the resolvant cubic of f(X) and let
m = [k(u, v, w): k]. Then m[6 and
(i ) m = 1 if and only if G = H.
(ii ) m = 2 if and only if G

= D
8
or G

= Z
4
, with G

= Z
4
if and only if f(X) ir reducible
over k(u, v, w).
(iii ) m = 3 if and only if G = A
4
.
(iv) m = 6 if and only if G = S
4
.
Proof. Since k(u, v, w) is the splitting eld of a polynomial of degree 3, one has that m[3! = 6
(see HW 9.10). On the other hand, by Proposition 9.24, G is a transitive subgroup of S
4
with
4 [ [G[. Thus [G[ is 24, 12, 8, or 4. If [G[ = 24, then G = S
4
. If [G[ = 12, then G = A
4
.
Considering D
8
as group of symmetries of a square (with vertices labelled 1, 2, 3, 4), we see that
D
8

= (1 2 3 4), (2 4) S
4
. Thus S
4
has a subgroup of order 8 which is isomorphic to D
8
. But
every subgroup of order 8 in S
4
is a 2-Sylow subgroup, and hence these groups are all isomorphic.
Thus every subgroup of S
4
of order 8 is isomorphic to D
8
. Finally, the subgroups of order 4 of
S
4
are cyclic, or H, or (i j), (l k) with i j l k = . But the latter ones are not transitive,
and hence ruled out. Thus G = S
4
, G = A
4
, G

= D
8
, G

= Z
4
, or G = H. Thus in the above
statements it suces to prove the suciency. Notice that by Lemma 9.33 (and Proposition 9.3),
m = [k(u, v, w): k] = [G: G(K/k(u, v, w))] = [G: GH]. If G = S
4
, then m = [G: GH] = 6. If
G = A
4
, then m = [G: GH] = [A
4
: H] = 3. If G

= D
8
, then G is a 2-Sylow subgroup of S
4
and
hence contains H since H is a normal 2-subgroup of S
4
. Thus m = [G: G H] = [G: H] = 2. If
G

= Z
4
, then G is generated by a 4-cycle, and may assume G = (1 2 3 4). Now (1 2 3 4) , H and
id ,= (1 2 3 4)
2
= (1 3)(2 4) H. Thus [GH[ = 2 and hence m = [G: GH] = 2. If G = H, then
m = [G: G H] = 1. It remains to show that if G

= D
8
then f(X) is irreducible over k(u, v, w),
and if G

= Z
4
then f(X) is reducible over k(u, v, w). Assume that G

= D
8
. Then we had seen
that G H = H. But by Lemma 9.33, G(K/k(u, v, w)) = G H. Thus G(K/k(u, v, w)) = H is a
transitive subgroup of S
4
. Now, if
1
, . . . ,
4
denote the roots of f(X) in K, then for all i ,= j there
exists G(K/k(u, v, w)) with (
i
) =
j
. Thus f(X) has to be irreducible over k(u, v, w) by
Remark 8.14. Next assume that G

= Z
4
. Then [K: k(u, v, w)] = [K: k]/2 = 4/2 = 2 < deg f(X).
Thus f(X) cannot be irreducible over k(u, v, w).
Example 9.37.
(1) Let f(X) = X
4
+ 4X
2
+ 2 [X], which is irreducible by Eisensteins Criterion. By
Lemma 9.34, the cubic resolvant is X
3
4X
2
8X + 32 = (X 4)(X
2
8). Thus
(u, v, w) = (

8) = (

2), and hence m = 2. By the quadratic formula, f(X) =


(X
2
(2+

2))(X
2
(2

2)) is reducible over (

2. Thus G

= Z
4
by Theorem 9.36.
(2) Let f(X) = X
4
10X
2
+ 4 [X], which is irreducible since one can check that it has
no linear and no quadratic monic factors in Z[X]. By Lemma 9.34, the cubic resolvant is
X
3
+ 10X
2
16X 160 = (X + 10)(X
2
16) = (X + 10)(X 4)(X + 4). Thus m = 1.
Hence G = H

= Z
2
Z
2
by Theorem 9.36.
(3) Let f(X) = X
4
2 [X]. Then f(X) is irreducible, and the splitting eld K of f(X)
is (
4

2, i), where
4

2 1. Since i , (
4

2) 1 and [(
4

2)(i): (
4

2)] 2, we have
[(
4

2, i): (
4

2)] = 2. Thus [K: ] = 8. Hence by Theorem 9.36, G



= D
8
. The eld
L = (

2, i) is the unique subeld of K that is normal and has degree 4 over , because
D
8
has only one normal subgroup of order 2 (use Theorem 9.5).
3. Symmetric functions
In this section R is a domain with k = Quot(R). Moreover, S = k[X
1
, . . . , X
n
] is a polynomial
ring with K = Quot(S) = k(X
1
, . . . , X
n
) its eld of rational functions. For S
n
there exists
a unique k-automorphism of K with X
i
X
(i)
, which we still denote by . With this iden-
tication we have that S
n
< Aut
k
(K). We say that f K is a symmetric rational function if
f(X
(1)
, . . . , X
(n)
) = f(X
1
, . . . , X
n
) for all S
n
. If in addition f S then we say that f
is a symmetric polynomial. Note that symmetric rational functions = K
S
n
is a subeld of K
containing k. Also, symmetric polynomials = K
S
n
S is a subring of S containing R.
3. SYMMETRIC FUNCTIONS 85
Our goal is to determine K
S
n
. We do this using the method of Lemma 9.12. The polynomial
(Y X
1
) (Y X
n
) =

S
n
(Y (X
1
))
has coecients in K
S
n
S (Lemma 9.12. So we may write (Y X
1
) (Y X
n
) = Y
n
s
1
Y
n1
+
+ (1)
n
s
n
with s
i
symmetric polynomials. It turns out that
s
i
=

l
1
<<l
i
X
l
1
X
l
i
,
which is a homogeneous polynomial of degree i. The polynomials s
1
, . . . , s
n
are called elementary
symmetric polynomials (functions). Notice that if
n

i=0
a
i
Y
i
= (Y
1
) (Y
n
) is any monic
polynomial of degree n, then a
i
= (1)
ni
s
ni
(
1
, . . . ,
n
), i.e., the coecients of a polynomial
can be expressed as elementary symmetric functions in the roots.
Theorem 9.38.
(a) K
S
n
= k(s
1
, . . . , s
n
) (i.e., every symmetric rational function is a rational expression
in s
1
, . . . , s
n
), K is the splitting eld of (Y X
1
) (Y X
n
) over k(s
1
, . . . , s
n
), and
[K: k(s
1
, . . . , s
n
)] = n!.
(b) K
S
n
S = R[s
1
, . . . , s
n
] (i.e., every symmetric polynomial is a polynomial expression in
s
1
, . . . , s
n
).
(c) X

1
1
X

n
n
[ 0
i
n i form a basis of K as a k(s
1
, . . . , s
n
)-vector space, and of S
as an R[s
1
, . . . , s
n
]-module.
Proof. Let us prove (a). Write f(Y ) = (Y X
1
) (Y X
n
). Since f(Y ) is a polynomial
over k(s
1
, . . . , s
n
), it follows that K = k(X
1
, . . . , X
n
) = k(s
1
, . . . , s
n
)(X
1
, . . . , X
n
) is a splitting
eld of f(Y ) over k(s
1
, . . . , s
n
). Thus [K: k(s
1
, . . . , s
n
)] (deg f(Y ))! = n! On the other hand,
[K: K
S
n
] = [S
n
[ = n! (Theorem 9.13), and k(s
1
, . . . , s
n
) K
S
n
. Thus K
S
n
= k(s
1
, . . . , s
n
) and
[K: k(s
1
, . . . , s
n
)] = n! Let us now prove (c). We rst show that W = X

1
1
X

n
n
[ 0
i
ni
is a spanning set of S = R[X
1
, . . . , X
n
] as a module over R[s
1
, . . . , s
n
]. But X
1
is the root of the
monic polynomial f(Y ) of degree n with coecients in R[s
1
, . . . , s
n
]. Thus 1, X
1
, . . . , X
n1
1
form
a spanning set of R[s
1
, . . . , s
n
][X
1
]( S) as a module over R[s
1
, . . . , s
n
]. Now since X
1
is a root
of f(Y ) in R[s
1
, . . . , s
n
][X
1
], we know that Y X
1
divides f(Y ) over the latter ring. Thus (Y
X
2
) (Y X
n
) is a polynomial with coecients in R[s
1
, . . . , s
n
][X
1
]. This polynomial is monic of
degree n1 and X
2
S is a root. Thus 1, X
2
, . . . , X
n2
2
is a spanning set of R[s
1
, . . . , s
n
][X
1
][X
2
]
as a module over R[s
1
, . . . , s
n
][X
1
]. Hence X

1
1
X

2
2
[ 0
1
n 1, 0
2
n 2 form a
spanning set of R[s
1
, . . . , s
n
][X
1
, X
2
] over R[s
1
, . . . , s
n
]. Continuing this was one sees that W is a
spanning set of S = R[s
1
, . . . , s
n
][X
1
, . . . , X
n
] over R[s
1
, . . . , s
n
]. Thus W is also a spanning set
of k(s
1
, . . . , s
n
)[X
1
, . . . , X
n
] as a vector space over k(s
1
, . . . , s
n
). But k(s
1
, . . . , s
n
)[X
1
, . . . , X
n
] =
k(s
1
, . . . , s
n
)(X
1
, . . . , X
n
) = K since X
i
are algebraic over k(s
1
, . . . , s
n
) (Proposition 8.5). Thus
W is a spanning set of K over k(s
1
, . . . , s
n
). But [W[ = n! = [K: k(s
1
, . . . , s
n
)] by (a). Thus
W is a basis of K over k(s
1
, . . . , s
n
). Since W is linearly independent over k(s
1
, . . . , s
n
), it is in
particular linearly independent over R[s
1
, . . . , s
n
]. Thus W is also a basis of S as a module over
R[s
1
, . . . , s
n
]. Finally, let us prove (b). We need to show that k(s
1
, . . . , s
n
) S = R[s
1
, . . . , s
n
].
Write W = 1

W
t
, which is a basis of S over R[s
1
, . . . , s
n
] and of K over k(s
1
, . . . , s
n
). Thus
S = R[s
1
, . . . , s
n
]

wW

R[s
1
, . . . , s
n
]w k(s
1
, . . . , s
n
)

wW

k(s
1
, . . . , s
N
)w = K. Hence, if an
element of S is in k(s
1
, . . . , s
n
) then all coecients of w W
t
vanish, hence the element is in
R[s
1
, . . . , s
n
].
Let k be a eld, k[u
1
, . . . , u
n
] is a polynomial ring, k(u
1
, . . . , u
n
) a eld of rational functions. The
polynomial g(Y ) = Y
n
+u
1
Y
n1
+. . . +u
n1
Y +u
n
k[u
1
, . . . , u
n
][Y ] k(u
1
, . . . , u
n
)[Y ] is called
the general polynomial of degree n over k. Notice that every monic polynomial of degree n in
k[Y ] is obtained from g(Y ) by specializing the coecients. Let
1
, . . . ,
n
be the roots of g(Y ) in
some splitting eld L og g(Y ) over k(u
1
, . . . , u
n
). Then u
i
= (1)
i
s
i
(
1
, . . . ,
n
) k(
1
, . . . ,
n
)
86 9. GALOIS THEORY
and hence L = k(u
1
, . . . , u
n
)(
1
, . . . ,
n
) = k(
1
, . . . ,
n
). Dene the k-algebra homomorphism
: k[X
1
, . . . , X
n
] L = k(
1
, . . . ,
n
) given by (X
i
) =
i
. Then (s
i
) = s
i
(
1
, . . . ,
n
) =
(1)
i
u
i
. Since u
1
, . . . , u
n
are variables over k it follows that
[k[s
1
,...,s
n
]
is surjective. Now let
z Ker(). Then

S
n
(z) Ker() since Ker() is an ideal. But

S
n
(z) k[s
1
, . . . , s
n
]
by Theorem 9.38(b). Thus

S
n
(z) = 0, hence z = 0. Thus is injective. Now passing to
the quotient eld, induces a k-isomorphism, which we still denote by : k(X
1
, . . . , X
n
)

L.
Notice that
[k(s
1
,...,s
n
)
: k(s
1
, . . . , s
n
)

k(u
1
, . . . , u
n
).
x
i

i
k(X
1
, . . . , X
n
)

L = k(
1
, . . . ,
n
)
[ [
k(s
1
, . . . , s
n
)

k(u
1
, . . . , u
n
)
s
i
u
i
Corollary 9.39. There is an isomorphism over k between k(s
1
, . . . , s
n
) and the rational
function eld k(u
1
, . . . , u
n
) given by s
i
u
i
. In particular, every symmetric polynomial can be
uniquely written as a polynomial expression in elementary symmetric polynomials.
Corollary 9.40. The general polynomial of degree n is separable over k(u
1
, . . . , u
n
) with
Galois group S
n
.
Proof. Use Theorem 9.38(a) and Theorem 9.13.
Corollary 9.41. Let G be a nite group. Then there exists a eld k and a nite Galois
extension k K with G(K/k)

= G.
Proof. We have that G S
n
. Now use Corollary 9.40 and Theorem 9.5(a).

4. Cyclotomic elds
Here k is a eld and n 1. A splitting eld K of x
n
1 over k is called a cyclotomic extension of
order n of k; the roots of X
n
1 are called n-th roots of unity, and the set of n-th roots of unity in
K is denoted by
n
. Recall that
n
is a nite subgroup of K

and hence cyclic (Theorem 7.33) and


that the generators of
n
are called primitive n-th roots of unity (Example 7.34(b)). In particular,
K = k(), with any primitive n-th root of unity.
If char k = p > 0, write n = mp
r
with r 0 and p [ m. Then X
n
1 = X
n
1
p
r
= (X
m
1)
p
r
.
Thus the cyclotomic extension of order n is equal to the cyclotomic extension of order m. Hence,
replacing n by m we may assume that char k [ n. But then X
n
1 and (X
n
1)
t
= nX
n1
have no
common roots in K (notice that n 1 ,= 0 in K), and hence X
n
1 has no multiple roots in K (see
Proposition 7.36). Thus [
n
[ = n and k K is Galois. Let G(K/k), then
[
n
:
n

n
is an automorphism of multiplicative groups (see Remark 8.14), so
[
n
Aut(
n
). But since
n
is cyclic of order n, Aut(
n
)

= (Z
n
)

= primitive n-th root of unity. Now
[
n
gives a
monomorphism of groups G(K/k) Aut(
n
)

= (Z
n
)

.
Proposition 9.42. Let K be a cyclotomic extension of order n over k with char k [ n. Then
K = k() for any primitive n-th root of unity, k K is nite Galois and Abelian, G(K/k)
(Z
n
)

, and [K: k][(n).


Proof. Recall that [K: k] = [G(K/k)[ (see Proposition 9.3) and that [(Z
n
)

[ = (n).
Let p [ char k and
i
the primitive n-th roots of unity. Then
n
(X) = (X
1
) (X
(n)
)
is called the n-th cyclotomic polynomial over k.
Lemma 9.43.
(a) X
n
1 =

d[n

d
.
5. CHARACTERS 87
(b)
1
= X 1 and for n > 1 we have
n
=
X
n
1

d[n
d<n

d
.
Proof. We prove (a). The polynomials X
n
1 and
d
have no multiple roots. Thus it suces
to show that roots of X
n
1 =

d[n
roots of
d
. But roots of X
n
1 = K

[
n
=
1 = K

[ [[ [n =

d[n
K

[ [[ = d =

d[n
roots of
d
.
Proposition 9.44.
n
(X) has coecients in the prime ring of k.
Proof. We induct on n 1, the assertion being clear for n = 1. So let n > 1. Then
by Lemma 9.43(b) and our induction hypothesis, X
n
1 =
n
f(X), where f(X) is a monic
polynomial with coecients in the prime ring. Since X
n
1 has all its coecients in the prime ring
it follows that
n
has the same property (write
n
=

a
i
X
i
and show by decreasing induction
on i that a
i
is in the prime ring).
Example 9.45.
(1) The n-th cyclotomic polynomial over is a polynomial in Z[X] of degree (n).
(2) If n = p is prime, then
p
(X) =
X
p
1
X 1
=
p1

i=0
X
i
.
(3)
4
(X) =
X
4
1
(X 1)(X + 1)
= X
2
+ 1.
Theorem 9.46. The n-th cyclotomic polynomial is irreducible over . In particular with
denoting a primitive n-th root of unity and K = (), one has that
n
is the minimal polynomial
of over , [K: ] = (n) and G(K/k)

= (Z
n
)

.
Proof. Let f(X) be the minimal polynomial of over . We will show that
p
is again a
root of f(X) for all prime p [ n. Since
m
[ g.c.d.(m, n) = 1 is exactly the set of all primitive
n-th roots of unity it then will follow that every primitive n-th root of unity is a root of f(X).
Thus deg f(X) (n). On the other hand,
n
[X] is monic of degree (n) with
n
() = 0.
Thus f(X) =
n
(X). The rest follows from Proposition 9.42. We have f(X)[X
n
1 in [X],
and hence X
n
1 = f(X)g(X) with g(X) [X]. Since X
n
1 is monic and in Z[X] it follows
that f(X) and g(X) are in Z[X]. Let p be a prime with p [ n and suppose that
p
is not a root of
f(X). Since
p
is a root of X
n
1 = f(X)g(X), it follows that
p
is a root of g(X). Thus is a
root of g(X
p
), thus f(X)[g(X
p
), thus
g(X
p
) = f(X)h(X),
where h(X) is even in Z[X], since g(X
p
) is monic and in Z[X]. Let indicate images in Z
p
[X].
Then g(X
p
) = f(X) g(X). But g(X
p
) = g(X)
p
(since Frobenius is a homomorphism of rings on
Z
p
[X] inducing the identity on Z
p
). Thus
g(X)
p
= f(X) h(X),
which shows that g(X) and f(X) have a common irreducible factor (recall that deg f(X) 1).
But then X
n
1 = f(X) g(X) would have a multiple root in the algebraic closure of Z
p
, which is
impossible since char Z
p
= p [ n.
5. Characters
Here G is a monoid and K a eld. A character of G in K is a map : G K

with (xy) =
(x)(y) for all x, y G. Maps f
i
: G K are called linearly independent over K if

nite
a
i
f
i
= 0,
with a
i
K, implies a
i
= 0 for all i.
Example 9.47. An embedding : k K of elds gives a character =
[k
: k

of k

in K.
88 9. GALOIS THEORY
Theorem 9.48 (Artin). Distinct characters
1
, . . . ,
n
of G in K are linearly independent over
K.
Proof. The case n = 1 is clear. Let n 2 and suppose
1
, . . . ,
n
are not linearly independent.
Then a
1

1
+ + a
n

n
= 0 with a
i
K not all zero. Taking a shortest such linear combination
we may assume a
,
= 0 for all 1 i n. Notice n 2. Since
1
,=
2
, there exists z G with

1
(z) ,=
2
(z). Now (a
1

1
+ + a
n

n
)(zx) = a
1

1
(z)
1
(x) + + a
n

n
(z)
n
(x) = 0 for all
x G. Thus a
1

1
(z)
1
+ a
n

n
(z)
n
= 0. On the other hand a
1

1
(z)
1
+ +a
n

1
(z)
n
= 0,
as can be seen by multiplying a
1

1
+ + a
n

n
= 0 by
1
(z). Subtracting these equations we
obtain
a
2
(
2
(z)
1
(z))
2
+ +a
n
(
n
(z)
1
(z))
n
= 0,
with a
2
(
2
(z)
1
(z)) ,= 0. Thus contradicting the minimality of n.
Corollary 9.49. Distinct automorphisms
1
, . . . ,
n
of a eld K are linearly independent
over K.
Corollary 9.50. Let
1
, . . . ,
r
be distinct nonzero elements of a eld K, and let a
i
K
with a
1

1
+ +a
n

n
= 0 for all 1. Then a
i
= 0 for all i.
Proof. Let
i
: N K

be given by
i
() =

i
, and apply Theorem 9.48.
6. Norm and trace
Let k K be a nite eld extension. Let r = [K: k]
s
and let
j
: K K, 1 j r, be the
distinct embeddings over k of K into K (see Proposition 8.54), and let K.
(a) N
K
k
() =
_
_
r

j=1

j
()
_
_
[K: k]
i
is called the norm of from K to .
(b) Tr
K
k
() = [K: k]
i
r

j=1

j
() is called the trace of from K to k.
Notice that
(a) If k K is not separable then Tr
K
k
() = 0 (since then char k = p > 0 and [K: k]
i
= p
n
for n > 0).
(b) If k K is separable then N
K
k
() =
r

j=1

j
() and Tr
K
k
() =

r
j=1

j
().
Theorem 9.51. Let k K be a nite eld extension and K.
(a) N
K
k
: K

is a homomorphism of (multiplicative) groups; Tr


K
k
: K K is a
homomorphism of (additive) groups (even of k-vector spaces).
(b) Let L be an intermediate eld, k L K. Then N
L
k
N
K
k
= N
K
k
and Tr
L
k
Tr
K
k
= Tr
K
k
.
(c) If k then N
K
k
() =
[K: k]
and Tr
K
k
() = [K: k].
(d) Let f(X) = X
n
+ a
n1
X
n1
+ + a
0
be the minimal polynomial of over k. Then
N
K
k
() = ((1)
n
a
0
)
[K: k()]
and Tr
K
k
() = [K: k()](a
n1
).
Proof. Part (c) follows from the denitions because [K: k] = [K: k]
s
[K: k]
i
. As far as (d)
is concerned, since every emebdding : k() K over k has exactly [K: k()]
s
extensions
: K K (Theorem 8.54) and since [K: k]
i
= [K: k()]
i
[k(): k]
i
(Corollary 8.55), it follows ???????????
that N
K
k
() = (N
k()
k
())
[K: k()]
and Tr
K
k
() = [K: k()]Tr
k()
k
(). Thus we may assume that
K = k(). But then by Proposition 8.58, f(X) = (
r

j=1
(X
j
))
[K: k]
i
where r = [K: k]
s
and

1
, . . . ,
r
are the distinct roots of f(X) in K. But since f(X) is irreducible over k, for every j
there exists an embedding
j
: K K over k with
j
() =
j
. Since
1
, . . . ,
r
are distinct,

1
, . . . ,
r
are distinct. Thus
1
, . . . ,
r
=
1
, . . . ,
r
(recall that r = [K: k]
s
). Hence f(X) =
7. CYCLIC EXTENSIONS 89
r

j=1
(X
j
())
[K: k]
i
where
1
, . . . ,
r
are the distinct embeddings over k of K into K. Thus
N
K
k
() = (1)
[K: k]
a
0
= (1)a
0
and Tr
K
k
() = a
n1
. To prove (a), let K. Then (d)
implies that N
K
k
() k and Tr
K
k
() k. The rest is clear. Finally, let us prove (b). Notice that
the compositions N
L
k
N
K
L
and Tr
L
k
Tr
K
L
make sense by (a). Let s = [L: k]
s
and t = [K: L]
s
.
Then st = [K: k]
s
by 8.55. Let
i
: L K, 1 i s, be the distinct embeddings over k
( 8.54), and choose extensions which we still denote by
i
: K K ( 8.19). Let
j
: K K,
1 j t, be the distinct embeddings over L. Then
i

j
: K K are st distinct embeddings
over k. As st = [K: k]
s
= r, these are all the distinct embeddings
1
, . . . ,
r
over k of K into
K. Also recall [K: k]
i
= [K: L]
i
[L: k]
i
. Now let K. Then N
K
k
() = (

i,j

i
(
j
()))
[K: k]
i
=
(

i
((

j
())
[K: L]
i
))
[L: k]
i
= N
L
k
(N
K
L
()) (notice that N
K
L
() and hence the
i
can again be
regarded as embeddings of L). Likewise one shows the claim for the trace.
Proposition 9.52. Let k K be a nite eld extension. Then k K is separable if and only
if Tr
K
k
,= 0.
Proof. We have already noticed the only if part. For the converse, since k K is separable
we have that Tr
K
k
=
1
+ . . . +
r
with
1
, . . . ,
r
distinct embeddings. But
1
+ . . . +
r
,= 0 by
Theorem 9.48.
Example 9.53.
(1) Let denote complex conjugation, = a + ib C with a, b 1. Then N
C
1
() = =
a
2
+b
2
.
(2) Let = a +b

5 (

5), with a, b . Then N


(

5)

() = = a
2
+ 5b
2
.
(3) Let = a+b

2 (

2), with a, b . Then N


(

2)

() = (a+b

2)(ab

2) = a
2
2b
2
.
7. Cyclic extensions
Theorem 9.54 (Hilberts Theorem 90). Let k K be a nite Galois cyclic extension with
Galois group G = , and let K. Then N
K
k
() = 1 if and only if = /

for some 0 ,=
K (i.e., thinking of N: K

as a homomorphism of groups, Ker(N) = /

[ K

).
Proof. Let us prove the only if part. Since G = G

we have that N
K
k
() = N
K
k
(

), thus
by the multiplicativity of the norm, N
K
k
() = 1. As far as the if part is concerned: Write n = [[.
Then id, ,
2
, . . . ,
n1
are distinct automorphisms of K. Hence by Corollary 9.49, these are
linearly independent over K. Thus
= id + +()
2
+. . . +()
n2
()
n1
is a non zero map on K. Let K with () ,= 0, and set = (). Then ,= 0, and
= +() +()
2
() +. . . +()
n2
()
n1
().
Thus () = () + ()
2
() + . . . + ()
n2
()
n1
() + ()
n1
()
n
(),
where the last summand is N
K
k
() = = 1 = . Thus () = , ,= 0.
Theorem 9.55. Let k be a eld, n > 0 with char(k) [ p, and assume that k contains a primitive
n-th root of unity.
(a) Let k K be a (Galois) cyclic extension of degree n. Then there exists K such that
K = k() and is a root of X
n
a, a k.
(b) Let a k and a root of X
n
a. Then k k() is (Galois) cyclic of degree d[n and

d
k.
Proof. Let us prove (a). Let k be a primitive n-th root of unity. Then N
K
k
(
1
) =
(
1
)
n
= 1 by Theorem 9.51(c) and thus since k K is cyclic,
1
= /() for some 0 ,= K
by Theorem 9.54, where denotes a generator of G = G(K/k). Hence () = . As k,
90 9. GALOIS THEORY

i
() =
i
. In particular,
i
() are distinct for 1 i n since char(k) [ n. Since
i
() are roots
of the minimal polynomial f(X) of over k, it follows that deg f(X) n. Thus K = k(). On
the other hand, (
n
) = (())
n
= ()
n
=
n
. Thus
n
K
G
= k (Theorem 9.5(a)). Write
a =
n
. Then a k and is a root of X
n
a. As far as (b) is concerned, we may assume that
,= 0. Let be a primitive n-th root of unity in k. Then
i
, 1 i n, are n distinct roots of
X
n
a in k() (since char(k) [ n and k). Thus all roots of X
n
a are of the form
i
. Also
k K = k() is Galois. Write G = G(K/k). For all G we have that () is a root of X
n
a,
hence of the form
i
for unique
i

n
. Thus
i
gives a map which is easily seen to be a
monomorphism of groups G
n
(recall that k). Since
n
is cyclic of order n, it follows that
G is cyclic with d = [K: k] = [G[ [ n. Finally, N
K
k
() =
l

d
for some integer l, thus
d
k since

l
k and N
K
k
() k by Theorem 9.51(a).
Corollary 9.56. Let k K be a eld extension, n > 0 with char(k) [ n, and assume that k
contains a primitive n-th root of unity. The following are equivalent:
(a) k K is cyclic of degree d with d[n.
(b) K is the splitting eld of X
n
a k[X] (in which case K = k() for any root of
X
n
a).
(c) K is the splitting eld of an irreducible polynomial X
d
b k[X] with d[n (in which case
K = k() for any root of X
d
b).
Theorem 9.57 (Hilberts Theorem 90, additive form). Let k K be a nite cyclic extension
with Galois group G = , and let K. Then Tr
K
k
() = 0 if and only if = () for some
K (i.e., Ker(Tr
K
k
) = () [ K).
Proof. The only if part is clear. As far as the converse is concerned, since k K is separable
by Proposition 9.52 there exists K so that Tr
K
k
() ,= 0. Now set =
1
Tr
K
k
()
(() + ( +
())
2
() +. . . + ( +() +. . . +
m
())
m
()). Then +() = .
Theorem 9.58 (Artin-Schreier). Let k be a eld of characteristic p > 0.
(a) Let k K be a cyclic extension of degree p. Then there exists K such that K = k()
and is a root of X
p
X a, a k.
(b) Let a k. Then f(X) = X
p
X a has either all its roots in k or f(X) is irreducible
over k. In the latter case, let be a root of f(X). Then k k() is (Galois) cyclic of
degree p.
Proof. Let us prove (a). Let G = G(K/k) = . Since Tr
K
k
(1) = [K: k](1) = p(1) = 0
(Theorem 9.51(c)), Theorem 8.26 implies that 1 = () for some K. Thus () = +1.
Hence
i
() = + i. Thus
i
() are distinct for 1 i p, which implies [k(): k] p. Hence
K = k(). Set a =
p
. Then (a) = (+1)
p
(+1) =
p
= a. Hence a K
)
= K
G
= k.
Now is a root of X
p
X a k[X]. Now, the rst assertion in (b) was shown in HW 8.8. So
assume that f(X) = X
p
X a is irreducible and is a root. Then + i, 0 i p 1, are
p distinct roots of f(X). Thus f(X) has p distinct roots in k(). Thus k k() is normal and
separable, hence Galois. Since +1 is also a root of the irreducible polynomial f(X), there exists
G = G(k()/k) with () = + 1 (Proposition 8.18). Since
i
() = + i are distinct for
0 i p 1, we have [[ p. On the other hand [G[ = [k(): k] = deg f(X) = p. Thus G =
and [k(): k] = p.
8. Solvable and radical extensions
Let k be a eld. An extension k K is called solvable if k K is Galois with solvable Galois
group. An extension k K is called radical if K = k(
1
, . . . ,
s
) with
n
i
i
k(
1
, . . . ,
i1
).
An extension k K is called solvable by radicals if K L for some radical extension k L. A
polynomial f(X) k[X] k is called solvable by radicals if the splitting eld of f(X) is solvable
by radicals.
Lemma 9.59. Let k K be an extension which is solvable by radicals. Then K L for some
normal radical extension k L.
8. SOLVABLE AND RADICAL EXTENSIONS 91
Proof. Let K E with k E radical and let
i
: E E, 1 i r, be the distinct
embeddings over k. Let L be the compositum of
1
(E), . . . ,
r
(E) in E. Let : L E be an
embedding over k. Then
i
: E E is an embedding over k, thus
i
=
j
for some j,
which implies (
i
(E)) =
j
(E) L for all i. Thus (L) L, so (L) = L, hence k L is normal
by Theorem 8.26. Clearly E L. (L is called the normal closure of k E). It is clear that
k L is still radical: Write E = k(
1
, . . . ,
r
) with
n
i
i
k(
1
, . . . ,
i1
) and use the fact that
L = k(
1
(
1
), . . . ,
1
(
s
),
2
(
1
), . . . ,
2
(
r
), . . . ,
r
(
1
), . . . ,
r
(
s
)).
Theorem 9.60. Let k K be a nite Galois extension of degree n.
(a) If k K is solvable by radicals then k K is solvable.
(b) Assume char(k) [ n. If k K is solvable, then k K is solvable by radicals.
Proof. Let us prove (a). By Lemma 9.59, there is a normal radical extension k L with K
L. Since both k L and k K are normal, there is an epimorphism of groups Aut
k
(L) G(K/k)
given by
[L
(Theorem 8.26). Thus it suces to show that Aut
k
(L) is a solvable group. Write
G = Aut
k
(L) and k
0
= L
G
. Then k k
0
L, hence k
0
L is still radical, and k
0
L is
Galois with Galois group G = Aut
k
(L) (Theorem 9.13). Thus, replacing k K by k
0
L , we
may assume that k K is radical and Galois and we need to show that G = G(K/k) is solvable.
Since k K is radical, K = k(
1
, . . . ,
s
) with
n
i
i
k(
1
, . . . ,
i1
), and we may assume that all
n
i
are prime numbers. Suppose that some n
i
= p = char(k). Then
i
is purely inseparable over
k(
1
, . . . ,
i1
). Since
i
is also separable over k(
1
, . . . ,
i1
) (Theorem 8.39), it follows that
i
is
in k(
1
, . . . ,
i1
) (Corollary 8.50) and hence can be deleted. Thus we may assume char(k) [ n
i
for
all i. Write m = n
1
n
s
. Then char(k) [ m. By assumption, K is the splitting eld of a separable
f(X) k[X]. Let L be the splitting eld of f(X)(X
m
1) k[X], and let be a primitive m-th
root of unity. Then L = K(), and since f(X)(X
m
1) is a separable polynomial, k L is Galois.
Now by HW 28.5(b) it suces to show that k L is solvable. Consider k k() L = k()K.
Here k() L is Galois and radical with the same n
1
, . . . , n
s
. Now k k() is Galois and Abelian,
hence solvable. Thus by HW 28.5(b) it suces to show that k() L is solvable. Thus replacing
k K by k() L, we may assume that k K = k(
1
, . . . ,
s
) is Galois,
n
i
i
k(
1
, . . . ,
i1
)
with char(k) [ n
i
, and k contains a primitive m = n
1
n
s
-th root of unity, thus a primitive n
i
-th
root of unity. Now by Theorem 9.55, k(
1
, . . . ,
i1
) k(
1
, . . . ,
i
) is Galois with cyclic Galois
group for all 1 i s. Since k K = k(
1
, . . . ,
s
) is Galois, Theorem 9.5 then implies that
G(K/k) is solvable. Let us prove (b). Let K be a primitive n-th root of unity and consider
k k() L = K(). It suces to show that k() L is radical. But by HW 28.5.c, k() L is
still Galois and solvable, and by Corollary 9.20, [L: k()] divides [K: k]. By the latter condition,
char(k) [ [L: k()] and k() contains a primitive [L: k()]-th root of unity. Now replacing k K
by k() L we may assume that k K is solvable of degree n with char(k) [ n and k containing
a primitive n-th root of unity. Since G = G(K/k) is nite and solvable, G has a normal series
G = G
0
G
1
G
s
= id with cyclic factors. Write K
i
= K
G
i
. Then by Theorem 9.5,
k = K
0
K
1
K
s
= K where K
i1
K
i
are Galois with Galois group G
i1
/G
i
. Thus
K
i1
K
i
are cyclic of degree n
i
say, where n
i
[n. Since char(k) [ n
i
and K
i1
contains a primitive
n
i
-th root of unity, Theorem 9.55 now implies that K
i
= K
i1
(
i
) with
n
i
i
K
i1
.
Corollary 9.61. Let f(X) k[X] be a separable polynomial of degree n 1 with Galois
group G.
(a) If f(X) is solvable by radicals then G is solvable.
(b) Assume char(k) = 0 or char(k) > n. If G is solvable, then f(X) is solvable by radicals.
Proof. Use Theorem 9.60 and the fact that the degree of the splitting eld divides n! (see
HW 9.10).
Corollary 9.62. Let k be a eld of char(k) ,= 2, 3, and let f(X) k[X] be a polynomial of
degree 4. Then f(X) is solvable by radicals.
Proof. Notice that f(X) is separable. Let G be the Galois group of f(X). Then G S
4
(Proposition 9.24), S
4
is solvable, thus G is solvable. Now use Corollary 9.61(b).
92 9. GALOIS THEORY
Recall that for a eld k, g(Y ) = Y
n
+u
1
Y
n1
+ +u
n1
Y +u
n
k(u
1
, . . . , u
n
)[Y ] is called the
general polynomial of degree n over k.
Corollary 9.63. For n 5 the general polynomial of degree n is not solvable by radicals.
Proof. By Corollary 9.40 the general polynomial of degree n has Galois group S
n
. But S
n
is
not solvable for n 5. Now use Corollary 9.61(a).
Example 9.64. The polynomial 2X
5
10X + 5 [X] is not solvable by radicals since its
Galois group is S
5
.
8. SOLVABLE AND RADICAL EXTENSIONS 93
Homework set # 11.
Choose ve of the following problems. The due date is on April ?? (Monday), 2002.
1. Let k be an innite eld, K = k(X) where X is an indeterminate. Prove that k K is
a Galois extension.
2. Let k K L be eld extensions. Suppose that k K and K L are Galois extensions.
Assume that every automorphism of K/k can be extended to an automorphism of L/k.
Prove that k L is a Galois extension.
3. Let K = (r) where r
3
+ r
2
2r 1 = 0. Verify that s = r
2
2 is also a root of the
equation X
3
+ X
2
2X 1 = 0. Show that K is a Galois extension and compute
G(K/).
4. Show that the extension K = (

2,

3, u), where u
2
= (9 5

3)(2

2), is a
Galois extension. Compute G(K/).
5. Let X
3
+ aX + b be irreducible over a eld k of characteristic 2. Prove that the Galois
group is A
3
or S
3
according as Y
2
+bY +a
3
+b
2
has or has not a root in k.
6. Let X
3
+ pX + q be irreducible over a nite eld k. Prove that 4p
3
27q
2
is a square
in k.
7. Let X
4
+dX +e be irreducible over a nite eld k of characteristic 2. Prove that d is a
cube in k.
8. Over a eld k of characteristic ,= 2, let f(X) be a cubic whose discriminant is a square
in k. Prove that f(X) is either irreducible or factors completely in k.
9. Over any base eld k prove that X
3
3X + 1 is either irreducible or factors completely
in k.
10. Let X
1
, . . . , X
n
be independent indeterminates over a eld k. The coecients of the
polynomial f(X) = (XX
1
)(XX
2
) . . . (XX
n
) are the elementary symmetric functions
on the indeterminates X
1
, . . . , X
n
f(X) = X
n
p
1
X
n1
+p
2
X
n2
. . . + (1)
n
p
n
.
Let s
k
= X
k
1
+X
k
2
+. . . +X
k
n
. Prove that the s
k
satisfy Newtons identities:
s
k
p
1
s
k1
+p
2
s
k2
. . . + (1)
k1
p
k1
s
1
+ (1)
k
kp
k
= 0,
for k 1.
11. Let E be a eld of characteristic zero and let F be its algebraic closure. Let be an
automorphism of F/E and let K be the xed eld of . Prove that every nite extension
of K is cyclic.
12. What is the explicit formula for the solutions of a quartic (characteristic ,= 2, 3)?
13. Describe the splitting eld of X
5
7 over the rationals. What is its degree? Show that
the Galois group is generated by two elements , satisfying the relations

5
= 1,
4
= 1,
1
=
2
.
14. Find a cyclotomic polynomial whose coecients are not all 0 or 1.
15. Let k be a eld of characteristic ,= 2. Prove that if 1 is a sum of squares then any
element of k is a sum of squares.
16. Let p be a prime unequal to the characteristic of the eld k. Show that, if a k then
X
p
a is either irreducible in k[X] or it has a root in k.
17. Assume that X
p
a, a , is irreducible in [X]. Show that the Galois group of X
p
a
over is isomorphic to the group of transformations of Z
p
of the form y ky +l where
k, l Z
p
and k ,= 0.
18. Let k K be a nite extension. Prove that if every element of K belongs to an interme-
diate extension that is Galois over k then K is Galois over k.
19. Let k be a eld of characteristic ,= 2, 3. Show that the following statements are equivalent:
Any sum of squares in k is a square.
Whenever a cubic polynomial f(X) factors completely in k, so does its derivative.
94 9. GALOIS THEORY
20. Let K = Z
p
(t), where t is a transcendental over Z
p
. Let G be the group of automorphisms
of K generated by the automorphism dened by (t) = t + 1. Determine k = K
G
and
[K: k].
21. Same as the Exercise above with G replaced by the group of automorphisms such that
t (at +b)(ct +d)
1
, where a, b, c, d Z
p
and ad bc = 1.
* * *
22. Let k K = k() be a simple eld extension, let G =
1
, . . . ,
n
be a nite subgroup of
Aut
k
(K), and write
n

i=1
(X

i
) =
n

i=0
a
i
X
i
. Show that f(X) is the minimal polynomial
of over K
G
and that K
G
= k(a
0
, . . . , a
n1
).
23. Let k be a eld, K = k(X) the rational function eld, Aut
k
(K) given by (X) =
1/(1 X), and G = . Show that [G[ = 3 and determine K
G
.
24. Use the notation of the previous problem, but with (X) = X+1. Assume that char(k) =
0. Show that G is innite and determine K
G
.
25. Let k K be a nite Galois extension with G = G(K/k), let L be a subeld of K
containing k with H = G(K/L), and let L
t
be the compositum in K of the elds (L),
H. Show that L
t
is the unique smallest subeld of K that contains L and is Galois
over k. Also, G(K/L
t
) =

G
H
1
.
26. Show that every algebraic extension of a nite eld is Galois and Abelian.
27. Compute
Aut

((
3

2)).
The Galois group of X
4
8X
2
+ 15 over .
The Galois group of 2X
5
10X + 5 over .
28. Let k be a eld of characteristic ,= 2 and f(X) k[X] a cubic whose discriminant is a
square. Show that f is either irreducible or a product of linear polynomials in k[X].
29. Let k be a eld of characteristic ,= 2, and let f(X) = X
4
+aX
2
+b k[X] be irreducible
with Galois group G. Show:
If b is a square in k, then G = H.
If b is not a square in k, but b(a
2
4b) is, then G

= Z
4
.
If neither b nor b(a
2
4b) is a square in k, then G

= D
8
.
30. Determine the Galois group of
X
4
5 over , over (

5), over (

5).
X
3
10 over , over (

2).
X
4
4X
2
+ 5 over .
X
4
+ 3X
3
+ 3X 2 over .
X
4
+ 2X
2
+X + 3 over .
31. Let K be the splitting eld of X
4
X
2
1 over . Determine all intermediate elds L,
L K. Which of these are Galois over ?
32. Let k be a subeld of 1, f(X) k[X] an irreducible quartic having exactly 2 non real
roots in C. Show that the Galois group of f(X) is isomorphic to D
8
or S
4
.
33. Show that the general polynomial g(Y ) = Y
n
+u
1
Y
n1
+. . . +u
n1
Y +u
n
is irreducible
in k(u
1
, . . . , u
n
)[Y ].
34. Let k be a eld.
Compute the discriminant of Y
3
Y k[Y ] and of Y
3
1 k[Y ] without using
Proposition 9.31.
Show that the discriminant of the polynomial (Y X
1
)(Y X
2
)(Y X
3
) over
k(X
1
, X
2
, X
3
) is of the form

1
s
6
1
+
2
s
4
1
s
2
+
3
s
3
1
s
3
+
4
s
2
1
s
2
2
+
5
s
1
s
2
s
3
+
6
s
3
2
+
7
s
2
3
with
i
k.
8. SOLVABLE AND RADICAL EXTENSIONS 95
Using the above results, conclude that the discriminant of Y
3
+ aY + b k[Y ] is
4a
3
27b
2
.
35. Let
n
(X) be the n-th cyclotomic polynomial over .
Let n = p
r
1
1
p
r
s
s
with p
1
, . . . , p
s
distinct prime numbers and r
i
> 0. Show that

n
(X) =
p
1
p
n
(X
p
r
1
1
1
p
r
s
1
s
).
For a prime p with p [ n show that
p
n(X) =

n
(X
p
)

n
(X)
.
36. Let n 3 and a primitive n-th root of unity over . Show that [(+
1
): ] = (n)/2.
37. Let be a primitive n-th root of unity over . Determine all n so that () is cyclic.
38. Let k K be an extension of nite elds. Show that N
K
k
and Tr
K
k
are surjective maps
from K to k.
39. Let f(X) k[X] be a separable polynomial of degree n 3 whose Galois group is
isomorphic to S
n
, and let k be a root of f(X).
Show that f(X) is irreducible.
Show that Aut
k
(k()) = id.
Show that
n
, k if n 4.
40. Let k K be a Galois extension.
For k L K show that G(K/L) is solvable if G(K/k) is solvable.
For k L K with k L normal show that G(L/k) and G(K/L) are solvable if
and only if G(K/k) is solvable.
For k L with K and L in a common eld show that G(KL/L) is solvable if G(K/k)
is solvable.
CHAPTER 10
Solution to Homework Problems
7.1 Let R = f : [0, 1] 1[ f continuous. Then R is a commutative ring with the usual
addition and multiplication of functions as operations.
Show that R is not Noetherian.
Determine m-Spec(R).
Answer: For n N let f
n
R be given by f
n
(x) = x 1/n for 1/n x 1 and
0 otherwise. Let I = (f
n
[ n N) and suppose that I is generated by nitely many
elements h
1
, . . . , h
s
. Each of the nitely many h
i
s is a linear combination of nitely
many f
i
s (with coecients in R). Thus I = (f
1
, . . . , f
m
) for some m. Hence f
m+1
=
a
1
f
1
+ +a
m
f
m
, a
i
R. But now evaluation at 1/m yields a contradiction. This shows
that R is not Noetherian. For x
0
[0, 1] let m
x
0
= f R[ f(x
0
) = 0. We claim that
m-Spec(R) = m
x
0
[ x
0
[0, 1]. Let
x
0
: R 1 be given by
x
0
(f) = f(x
0
). Then
x
0
is an epimorphism of rings with m
x
0
= Ker(
x
0
). Since 1 is a eld, then m
x
0
is a maximal
ideal of R. Conversely, let m m-Spec(R). For f R, write Z
f
= x [0, 1] [ f(x) = 0,
which is a closed subset of [0, 1]. Suppose that

fm
Z
f
= . Since [0, 1] is compact, there
exist nitely many f
1
, . . . , f
n
in m such that
n

i=1
Z
f
i
= . Thus h = f
2
1
+ +f
2
n
is nowhere
zero on [0, 1], and hence is a unit in R. But this is impossible since h m ,= R. Therefore
pick x
0

fm
Z
f
,= . Then m m
x
0
, hence m = m
x
0
by the maximality of m.
7.2 Let R be a PID. Show that every non-zero prime ideal of R is a maximal ideal.
Answer: Let (p) be a non-zero prime ideal and suppose that (p) (a). Thus p = ax for
some x R. Since p is a prime element we have that p[a or p[x. If p[a then a = pt for
some t R. Thus p = ax = ptx and the fact that R is a domain imply that x is a unit
so that (p) = (a). If p[x we conclude, with a similar argument, that a is a unit, hence
(a) = R. Thus (p) is a maximal ideal of R.
7.3 Let R be a commutative ring with 1 ,= 0. A homomorphism D: R R of additive
groups satisfying D(ab) = aD(b) +bD(a) for every a, b R is called a derivation on R.
Show that Ker(D) is a subring of R containing 1.
Show that 0 is the only derivation on Z[i].
Answer: Since Ker(D) is an additive subgroup of R, it suces to show that 1 Ker(D)
and that Ker(D) is closed under multiplication. But D(1) = D(1 1) = 1D(1) +1D(1) =
D(1) + D(1). Thus D(1) = 0 and 1 Ker(D). Now let a, b Ker(D). Then D(ab) =
aD(b) + bD(a) = a0 + b0 = 0. Thus ab Ker(D). Suppose now that R = Z[i]. We
just observed that Ker(D) is a subring, thus it contains Z. So it is enough to show that
i Ker(D). But i
2
= 1, and hence 0 = D(1) = D(i
2
) = 2iD(i). Thus D(i) = 0 since
Z[i] is a domain.
7.4 Let S , 0 be a multiplicative subset of R. Show that if R is a UFD, then S
1
R is a UFD.
Answer: We want to prove that every non invertible and nonzero element r/s in S
1
R is
product of prime elements. Write r = p
1
p
n
as product of prime elements in R. Then
we have the factorization r/s = 1/s p
1
/1 p
n
/1. This leads to a prime factorization: In
fact, if some of the p
i
s are in S then they become invertible elements in S
1
R. Otherwise,
if (p) is prime ideal in R disjoint from S then S
1
(p) is a prime ideal in S
1
R, see
Theorem 6.62(b). Thus p/1 is a prime element in S
1
R.
7.5 Factor 3 + 4i into irreducible elements in Z[i].
97
98 10. SOLUTION TO HOMEWORK PROBLEMS
Find a generator for the ideal (85, 1 + 13i) in Z[i]. In other words, nd the greatest
common divisor for 85 and 1 + 13i.
Answer: Let u = 3 + 4i and observe that N(u) = 25. Thus we can write u = (2 + i)
2
.
We need to show that v = 2 + i is irreducible. Write v = 2 + i = xy and notice that
N(v) = 5 = N(x)N(y).Thus either N(x) or N(y) must be 1. Hence either x or y
belongs to Z[i]

= 1, i. Lets now consider the elements 85 and 1 + 13i. Observe


that 85 = (1 + 4i)(1 4i)(2 + i)(2 i) and 1 + 13i = (2 + i)(1 i)(1 + 4i). Thus
g.c.d.(85, 1 + 13i) (1 4i)(2 +i) = 6 7i.
7.6 Let R be a domain. If an l.c.m. of a and b exists then so does a g.c.d. (Notice that the
converse does not hold in general.)
Answer: First of all, if m is the l.c.m. of a and b we have that (a) (b) = (m). Thus
there exists d such that ab = md. From m (a) we get that b (d) and similarly a (d),
so that (a, b) (d). Now, if c is a common divisor of a and b then a = ct and b = cs,
so that cst is a common multiple of a and b, and hence is divisible by m. Then from
md = ab = c cst we get that d is divisible by c. Therefore d is a g.c.d. of a and b.
7.7 If R is a UFD, show that the intersection of an arbitrary collection of principal ideals is
again principal.
Answer: If

iI
(a
i
) ,= 0, then factorize each a
i
as a product of primes: a
i
= u
i

p
r(i,)

,
with u
i
units, p

prime elements such that (p

) ,= (p

) for ,= . Then

iI
(a
i
) = (d),
where d =

p
maxr(i,) [ iI

.
7.8 If R is a UFD, show that every non-zero minimal prime ideal is principal.
Answer: Let p be a non-zero minimal prime of R. Take any non-zero a p, and express
a as a product of prime elements, say a = p
1
p
n
. Then at least one the p
i
s belongs to
p. If p
i
p then (p
i
) p. But (p
i
) is a non-zero prime ideal and by the minimality of p
we conclude that p = (p
i
).
7.9 An integral domain R is a UFD if and only if the ascending chain condition holds for
principal ideals, and any two elements of R has an l.c.m.
Answer: The only if is already known, and we prove the if. From the rst condition
it follows that every element which is neither 0 nor a unit can be written as a product of
irreducible elements, so that we need only prove that an irreducible element is prime. Let
a be an irreducible element, and let xy (a) and x , (a). By assumption we can write
(a) (x) = (m); now 1 is a g.c.d. of a and x, so that one sees from the proof of HW 7.6
that (m) = (ax), and then xy (a) (x) = (ax) implies that y (a). Therefore (a) is
prime.
8.1 Is the polynomial x
4
5 irreducible in Z
3
[X]? If not, nd a factorization.
Answer: Notice that X
4
5 X
4
+ 1 in Z
3
[X]. Moreover, it doesnt have any root in
Z
3
so that if it factors it must factor as a product of quadratic polynomials. Check that
X
4
+ 1 = (X
2
+ 2X + 2)(X
2
+X + 2).
8.2 Let m be a maximal ideal of Z[X] such that mZ ,= 0. Show that m = (p, f), where p is
a prime in Z and f is a monic irreducible polynomial in X of positive degree.
Answer: Observe that m Z is a nonzero prime ideal in Z, hence m Z = pZ for
some prime number p in Z. Now m/pZ[X] is a maximal ideal in Z[X]/pZ[X]. But
Z[X]/pZ[X]

= (Z/pZ)[X] is a polynomial ring over a eld, thus a PID. Now every nonzero
ideal (in particular every maximal ideal) in the latter ring is generated by a monic poly-
nomial. Thus m/pZ[X] = (p, f)Z[X]/pZ[X] for some monic irreducible polynomial f in
Z[X], and hence m = (p, f).
8.3 Let R be a UFD, p be a prime ideal of R[X] with p R = 0. Show that p is principal.
Answer: We may assume that p ,= 0. Let f(X) be an element of smallest degree in p.
Write f(X) = c(f)f
1
(X) with f
1
(X) primitive. Since p R = 0 and c(f) R, we have
c(f) , p. But then f
1
(X) p since p is prime. Thus we may assume that f(X) = f
1
(X)
is primitive. But then f(X) is irreducible in R[X], since otherwise by the primitivity,
f(X) = g(X)h(X) with deg g(X) < d and deg h(X) < d, and since by the primeness of
10. SOLUTION TO HOMEWORK PROBLEMS 99
p, g(X) p or h(X) p, contradicting the minimality of f(X). Thus f(X) p R is
irreducible in R[X]. Thus f(X) is irreducible in K[X], where K = Quot(R). But K[X]
is a PID, thus (f(X))K[X] is a maximal ideal in K[X]. Since (f(X))K[X] pK[X] it
follows that either: (1) pK[X] = K[X] or (2) pK[X] = (f(X))K[X]. Suppose that (1)
holds; then
n

i=0
g
i
(X)h
i
(X) = 1 for some g
i
(X) p and h
i
(X) K[X]. Let 0 ,= a R
with ah
i
(X) R[X] for all 0 i n. Then
n

i=0
g
i
(X)(ah
i
(X)) = a p R. This is
impossible. Thus (2) holds. Thus for all g(X) p, f(X) divides g(X) in K[X], hence
f(X) divides g(X) in R[X] since f(X) is primitive. Hence p = (f(X))R[X] is principal.
8.4 Show that the polynomial f(X) = X
3
5X 1 is irreducible in Z[X].
Answer: The polynomial f(X) is primitive so that it is irreducible in Z[X] if and only
if it is irreducible in [X]. Since it has degree 3, any factorization must contain a factor
of degree 1, thus f(X) must have a root in . However, the only possible roots can be
1 since the polynomial is monic and the constant term is 1 but f(1) = 5 and
f(1) = 3. Thus f(X) is irreducible.
8.5 Let n 2, m 1. Use induction on n to show that the polynomial f(X
1
, . . . , X
n
) =
_
n

i=1
X
m
i
_
1 is irreducible in [X
1
, . . . , X
n
].
Answer: Write f
n
=
_
n

i=1
X
m
i
_
1. If n = 2 then we can write f
2
= X
m
2
f
1

[X
1
][X
2
], where f
1
[X
1
]. Now f
2
is irreducible by Eisensteins Criterion since f
1
=
(X 1)
m1

i=0
X
i
1
, where X
1
1 is prime and (X
1
1) [
m1

i=0
X
i
1
(since 1 is not a root
of
m1

i=0
X
i
1
). If n > 2, write f
n
= X
m
n
f
n1
[X
1
, . . . , X
n1
][X
n
]. Now f
n1
is a
prime element of [X
1
, . . . , X
n1
] by induction hypothesis. Hence f
n
is irreducible by
Eisensteins Criterion.
8.6 Let I k[X, Y ] consist of all polynomials f(X, Y ) with coecients in a eld k such that
the sum of the coecients of f(X, Y ) is zero.
Show that I is an ideal of k[X, Y ].
Show that I is a maximal ideal.
Answer: Let : k[X, Y ] k be given by (

i
1
,i
2
a
i
1
i
2
X
i
1
Y
i
2
) =

i
1
,i
2
a
i
1
i
2
. Then is an
epimorphism of rings (check!) with Ker() = I. Since k is a eld, then I is a maximal
ideal of R.
8.7 Let R = Z[

5] and K = Quot(R). Show that f(X) = 3X


2
+ 4X + 3 is irreducible in
R[X] but is reducible in K[X].
Answer:
8.8 Let k be a eld of characteristic p > 0 and let a k. Show that f(X) = X
p
X a is
irreducible in k[X] if f(X) has no root in k.
Answer: Let be a root of f in an algebraic closure of k. Then +i, for i = 0, . . . , p1
are roots as well
( +i)
p
( +i) a =
p
+i
p
i a =
p
a = 0,
as i
p
= i. Thusf(X) =
p1

i=0
(X i). Now suppose that f(X) = g(X)h(X), where
g(X) is monic in k[X] with 1 d = deg g(X) p 1. Then g(X) is a product of d
linear polynomials of the form X i. Thus the coecient of X
d1
in g is d +
100 10. SOLUTION TO HOMEWORK PROBLEMS
with d k

and k. As this coecient is in k, it follows that k, which yields a


contradiction.
8.9 Show that f(X) = X
4
+ 3X
3
+ 3X
2
5 is irreducible in [X].
Answer:
8.10 Let k be a eld and X, Y, t be indeterminates over k. Dene the homomorphism of rings
: k[X, Y ] k[t
2
, t
3
] k[t] by (X) = t
2
and (Y ) = t
3
. Show that Ker() = (X
3
Y
2
)
and that it is a prime ideal.
Answer: Geometrically, R = k[t
2
, t
3
] = p(t
2
, t
3
) [ p(X, Y ) k[X, Y ] is the coordinate
ring that corresponds to the cusp. It is convenient though to see R as the quotient
ring of a polynomial ring modulo an ideal. This can be achieved as follows. Dene the
homomorphism of rings : k[X, Y ] k[t
2
, t
3
] by (X) = t
2
and (Y ) = t
3
. By the First
Homomorphism Theorem for rings we have that k[t
2
, t
3
]

= k[X, Y ]/ker(). Note that
ker() is nitely generated as k[X, Y ] is a Noetherian ring: actually ker() is a principal
ideal as it is a height one prime ideal in a UFD (see HW 7.8). Clearly, X
3
Y
2
belongs
to Ker() as (X
3
Y
2
) = (t
2
)
3
(t
3
)
2
= 0. We claim that ker() = (X
3
Y
2
).
Indeed gives a surjective map between the vector spaces V = k[X, Y ]/(X
3
Y
2
) and
W = k[t
2
, t
3
]. Observe that X
i
, X
i
Y

i=0
forms a generating set of the k-vector space
V and that t
2
, t
3
, t
4
, t
5
, . . . is a basis of the k-vector space W. Moreover, sends
the spanning set for V onto the basis of W: (X
i
) = t
2i
and (X
i
Y ) = t
2i+3
. Hence
X
i
, X
i
Y

i=0
is a basis for V as well (check!). Thus k[t
2
, t
3
]

= k[X, Y ]/(X
3
Y
2
).
8.11 Let a
n
X
n
+ +a
0
be a zero divisor in R[X]. Show that there exists b R, b ,= 0, with
ba
i
= 0 for all i.
Answer: Let f(X) = a
n
X
n
+ + a
0
and let g(X) R[X] with g(X) ,= 0 and
f(X)g(X) = 0. We induct on m = deg g(X) 0. If m = 0, we are done. So let m > 0 and
assume that the assertion holds for smaller values than m. Write g(X) = b
m
X
m
+ +b
0
,
with b
m
,= 0. If f(X)b
m
= 0, we are done. Otherwise f(X)b
m
,= 0, hence a
j
b
m
,= 0
for some j, hence a
j
g(X) ,= 0 for some j. Let i = maxj [ a
j
g(X) ,= 0. Then
a
i
g(X) ,= 0 and f(X)[a
i
g(X)] = 0. Thus we are done by induction once we have seen
that deg(a
i
g(X)) < m, or equivalently a
i
b
m
= 0. However, by the maximality of i,
0 = f(X)g(X) =
_
_
i

j=0
a
j
X
j
_
_
g(X). Thus a
i
b
m
= 0.
8.12 Let R be the set of all polynomials a
0
+a
1
X+ +a
n
X
n
such that the a
i
are all integers
and a
1
is even. Prove that R is an integral domain which is not a UFD.
Answer: R is clearly an integral domain as it is contained in Z[X], which is an integral
domain. Observe that 2X is irreducible in R. On the other hand, 2X is not a prime
element. Indeed 2X[2 X
3
but 2X [ 2 and 2X [ X
3
. Hence R is not a UFD.
8.13 Let k be a eld and let R = f(X) k[X] [ f
t
(0) = 0. Show that R is an integral domain
which is not a UFD.
Answer: Observe that R consists of all polynomials in k[X] with a
1
= 0. Clearly, R is
an integral domain as it is contained in k[X]. However it is not a UFD. Indeed, X
3
is an
irreducible element in R but it is not a prime element, as X
3
[X
2
X
2
= X
4
but X
3
[ X
2
.
9.1 Let K = (

2,

3) 1. Determine [K: ] and nd u K with K = (u).


Answer: Notice that [(

2): ] = 2 as X
2
2, by Eisensteins Criterion, is the minimal
polynomial of

2 over . On the other hand

3 , (

2): In fact,

3 = a + b

2 for
some a, b would imply that

2 = (3 a
2
2b
2
)/(2ab) . Thus X
2
3 is an
irreducible polynomial also over (

2) so that [(

2,

3): (

2)] = 2. Hence by the


degree formula we have that [(

2,

3): ] = 4. Let now u =

2+

3. We clearly have
that (u) (

2,

3). On the other hand, f(X) = X


4
10X
2
+ 1 is an irreducible
polynomial with f(u) = 0. Thus [(

2,

3): (u)] = 1. Thus (u) = (

2,

3).
9.2 Let k K, k L be eld extensions contained in some eld. Show that
[KL: k] [K: k][L: k].
[KL: L] [K: k].
10. SOLUTION TO HOMEWORK PROBLEMS 101
Answer: Write [KL: k] = t, [K: k] = n and [L: k] = m. If t is nite so are m and n,
since L and K are subelds of KL. We assume henceforth that m and n are nite. We
shall prove t is nite and at most mn by induction on n. The case n = 1 (that is K = k)
being trivial, we assume n > 1. Let u be an element in K but not in k. Write r for the
degree of u over k and s for the degree of u over L. We have s r, for the irreducible
polynomial for u over k is a multiple of the irreducible polynomial of u over L. We know
that [L(u): k] = ms and hence [L(u): k(u)] = ms/r. We also know that [K: k(u)] = n/r.
We apply our inductive assumption to the elds K and L(u) over k(u), and deduce that
[KL(u): k(u)] ms/r n/r = mns/r
2
mn/r.
Evidently we have that KL(u) = KL. Finally, we have that t = [KL: k] = [KL: k(u)]
[k(u): k] mn/r r = mn. This proves our rst assertion. The second one follows from
[KL: L][L: k] = [KL: k] [K: k][L: k] and cancellation.
9.3 Let k K and k L be nite eld extensions contained in some eld. Show that if
[KL: k] = [K: k][L: k] then K L = k. Is the converse true?
Answer:
The converse is false. For example, let u
1
be a real root of X
3
2 and u
2
a complex root
of X
3
2. Then [(u
i
): ] = 3 for i = 1, 2. However (u
1
)(u
2
) = (
3

2, i

3) is the
splitting eld of X
3
2 over : this one has degree 6 over . However (u
1
)(u
2
) = .
9.4 Let k K be an algebraic eld extension. Show that every embedding : K K over
k is an isomorphism.
Answer: Since K is a eld, Ker() is either the zero ideal or the whole eld. However
,= 0, so that is injective. We need to show that is also surjective. Let u K and let
f(X) K[X] be its minimal polynomial. Observe that
i
(u) K are also roots of f(X):
indeed, 0 =
i
(f(u)) = f(
i
(u)) as
[k
= id
k
. But f(X) has at most deg f(X) distinct
roots, so there exists i > j such that
i
(u) =
j
(u) or, equivalently,
j
(
ij
(u) u) = 0.
But
j
is injective as is. So we have that (w) = u, where w =
ij1
(u).
9.5 Show that arccos 11/16 can be trisected with ruler and compass.
Answer: Let 3 = arccos 11/16 or equivalently cos(3) = 11/16. It is enough to show
that cos can be constructed with ruler and compass. We know that cos(3) = 4 cos
3

3 cos . Thus, if we let = cos we get
4
3
3 = 11/16 or 64
3
4811 = 0.
It is easy to check that = 1/4 is a root. Thus, using Runis method
64 0 48 11
1/4 16 4 11
64 16 44 0
we get that 64
3
4811 = (+1/4)(64
2
1644) = 4(+1/4)(16
2
411) = 0.
Finally, the quadratic formula gives us that the two roots of the quadratic term are

1,2
=
1 3

5
8
.
Notice that (1 + 3

5)/8 is the only positive root and it is constructible with ruler and
compass: this is the value for = cos that what we were looking for.
9.6 Prove that the regular pentagon can be constructed with ruler and compass.
Prove that the regular 9-gon cannot be constructed with ruler and compass.
Answer:
Suppose we were able to construct with ruler and compass a regular 9-gon. This is
actually equivalent to being able to subdivide 2 into 9 equal parts, i.e., to being able to
construct 2/9 = 40

. But if we were able to construct 2/9, then we would also be able


to construct the angle 1/2(2/9) = /9 = 20

. This is impossible, though.


9.7 Let K = k() be an algebraic extension of odd degree. Show that K = k(
2
).
102 10. SOLUTION TO HOMEWORK PROBLEMS
Answer: Since
2
K = k() we clearly have that k k(
2
) k(). By the degree
formula we have that
[k(): k(
2
)][k(
2
): k] = [k(): k] = n (n odd).
Thus [k(): k(
2
)] divides n, which is an odd number. On the other hand, we have that
[k(): k(
2
)] 2, since is a root of the polynomial X
2

2
k(
2
)[X]. Hence, the
only possibility is that [k(): k(
2
)] = 1 so that k() = k(
2
).
9.8 Let k be a eld with char k ,= 2 and let a, b k. Let
2
= a and
2
= b. Assume that
, have degree 2 over k, and both lie in some extension of k. Prove that k() = k() if
and only if there exists c k such that a = c
2
b.
Answer: Suppose that a = c
2
b for some 0 ,= c k. Then = c with c K. Hence
k() and then k() k(). On the other hand, = /c so we also conclude
that k() and then k() k(). Conversely, suppose that K = k() = k(). Since
the minimal k-polynomials of and are X
2
a = 0 and X
2
b = 0 respectively, let
Aut
k
(K) be such that () = and () = . As K = k() = k(), we
can write = d + c for some c, d k. But then we have that d c = = () =
(d + c) = (d) + (c)() = d c. Hence, since char k ,= 2, we conclude that d = 0
and = c. Thus a = c
2
b with c k.
9.9 Let k be a eld, k(X) the eld of rational functions, u k(X) k. Write u = f(X)/g(X)
with f(X) and g(X) relatively prime in k[X]. Show that [k(X): k(u)] = maxdeg f(X),
deg g(X).
Answer: Let (Y ) be the polynomial in k(u)[Y ] given by
(Y ) = ug(Y ) f(Y ) =
f(X)
g(X)
g(Y ) f(Y ) k(u)[Y ].
The degree of (Y ) is exactly maxdeg f(Y ), deg g(Y ). Moreover, by construction,
we have that (X) = 0. Thus X is algebraic over k(u) and therefore [k(X): k(u)]
maxdeg f, deg g. Our claim follows once we show that (Y ) is an irreducible polyno-
mial over k(u)[Y ]. In order to do that, it is easy to observe that u is transcendental
over k: If not, we would also conclude that X is algebraic over k. In particular, k[u]
is a UFD with eld of fractions k(u). Since (Y ) is a primitive polynomial we know
that (Y ) is irreducible in k(u)[Y ] if and only if it is irreducible in k[u][Y ]. Assume that
(Y ) = h
1
(u, Y )h
2
(u, Y ) in k[u][Y ] where h
i
(u, Y ) =
s
i

j=0
h
ij
(u)Y
j
with h
ij
(u) k[u],
0 < s
i
< deg (Y ), and i = 1, 2. Since (Y ) is linear in u, we may also assume that
h
1j
(u) = a
1j
+b
1j
u and h
2j
(u) = c
2j
with a
1j
, b
1j
, c
2j
k. Therefore we have that
(Y ) = ug(Y ) f(Y ) =
s
1

j=0
a
1j
y
j

s
2

j=0
c
2j
y
j
+u
s
1

j=0
b
1j
y
j

s
2

j=0
c
2j
y
j
.
However u is transcendental over k, thus we conclude that
f(Y ) =
s
1

j=0
a
1j
y
j

s
2

j=0
c
2j
y
j
g(Y ) =
s
1

j=0
b
1j
y
j

s
2

j=0
c
2j
y
j
which contradicts, since s
2
> 0, the fact that f and g are relatively prime.
9.10 Let k be a eld, let f k[X] be a polynomial of degree n 1, and let K be the splitting
eld of f over k. Show that [K: k] divides n!
Answer:
9.11 Determine the splitting elds of the following polynomials over :
X
3
2; X
8
5; X
6
+X
3
+ 1.
Answer: The splitting eld of X
3
2 is (
3

2, i

3) and the degree of the extension


(
3

2, i

3) is 6 = 3!
Notice that f(X) = X
6
+X
3
+1 is an irreducible polynomial over and Z. This is because
f(X + 1) = X
6
+ 6X
5
+ 15X
4
+ 21X
3
+ 18X
2
+ 9X + 3 is irreducible by Eisensteins
10. SOLUTION TO HOMEWORK PROBLEMS 103
Criterion with p = 3. Let now
a =
1
2
+i

3
2
= e
i
2
3
and =
3

a = e
i
2
9
.
Using the quadratic formula it is easy to check that X
6
+ X
3
+ 1 = (X
3
a)(X
3
a),
where a denotes the complex conjugate of a. It is also easy to check that a = a
2
. Finally,
given that = a
3
we have that
f(X) = (X
3

3
)(X
3

6
) = (X )(X
2
+X +
2
)(X
2
)(X
2

2
X +
4
)
= (X )(X a)(X a)(X
2
)(X a
2
)(X a
2
)
= (X )(X
4
)(X
7
)(X
2
)(X
5
)(X
8
).
Thus f(X) factors completely in (). Hence the degree of the extension () is
6. Notice that f(X) is the product of the primitive 9-roots of unity. We usually denote
f(X) with
9
(X) and we emphasize that [(): ] = (9) = 6.
10.1 Let k be a eld and K = k(X) be the eld of rational functions. Show that (X) =
aX +b
cX +d
with a, b, c, d in k and ad bc ,= 0 determines an automorphism of K over k.
Conversely, show that every automorphism of K over k is uniquely determined by such
a rational function.
Answer: Observe that an automorphism of K = k(X) over k is determined by its value
on X. Next, notice that
(X) =
aX +b
cX +d
=
bc ad
c
2

1
X +
d
c
+
a
c
.
Thus (X) is obtained by composing maps of the form: X X + , X 1/X and
X X for some , k and ,= 0. These maps are clearly automorphisms of K
over k. Thus is an automorphism of K over k. Conversely, suppose that is an
automorphism of K over k. As we pointed out, is determined once we know (X).
Suppose (X) = u =
f(X)
g(X)
with g(X) ,= 0 and g.c.d.(f(X), g(X)) = 1. Now (k(X)) =
k(u). However, since is an automorphism of K we must have k(u) = k(X) = K. Thus,
from 9.9 we conclude that 1 = [k(X): k(u)] = maxdeg f(X), deg g(X).
10.2 Let k K be a nite eld extension. Show that k is perfect if and only if K is perfect.
Answer: Suppose that k is a perfect eld and let K L be an algebraic extension.
Since k K is a nite eld extension it is also an algebraic extension. Thus k L is
algebraic as well. Since k is perfect k L is also a separable extension. But then K L
is separable as well. Thus K is a perfect eld. Conversely, suppose that K is perfect and
suppose that there exists a k such that a ,= b
p
for some b k, that is a
1/p
, k. But the
we have k
p
k k
1/p
k
1/p
2
K. In this way we see that the perfect closure of
k, which is the union of k
1/p
n
is innite dimensional over k. A contradiction since k K
is nite dimensional.
10.3 Let k be a eld of characteristic p > 0, k(X, Y ) the eld of rational functions in two
variables.
Show that [k(X, Y ): k(X
p
, Y
p
)] = p
2
.
Find innitely many distinct elds L with k(X
p
, Y
p
) L k(X, Y ).
Answer: Consider the eld extensions k(X
p
, Y
p
) k(X
p
, Y ) k(X, Y ). Note that
[k(X, Y ): k(X
p
, Y )] = p, as k(X, Y ) = k(X
p
, Y )(X) and X is a root of the irreducible
polynomial Z
p
X
p
k(X
p
, Y )[Z]. The irreducibility follows from Eisensteins criterion:
Indeed X
p
is a prime element of k[X
p
, Y ], since the quotient ring is the domain k[Y ].
Similarly, one has that [k(X
p
, Y ): k(X
p
, Y
p
)] = p. Thus [k(X, Y ): k(X
p
, Y
p
)] = p
2
.
However L
c
= k(X
p
, Y
p
, X + cY ), with c k(X
p
, Y
p
), provide an innite family of
intermediate elds. In fact, if L
c
= L
c
with c ,= c
t
we conclude that Y and then X
belong to L
c
. But this impossible, since [L
c
: k(X
p
, Y
p
)] = p and not p
2
.
104 10. SOLUTION TO HOMEWORK PROBLEMS
10.4 Let K be the splitting eld of X
p
X 1 over k = Z
p
. Show that k K is normal,
separable, of degree p.
Answer: We already showed in 8.8 that f(X) = X
p
X 1 is irreducible over k[X]
if it has no root in k. Every element a of Z
p
satises a
p
= a (Fermats little Theorem),
so that a
p
a 1 = 1. Thus, f(X) is irreducible over Z
p
. However the splitting eld
K of f(X) is of the form K = Z
p
(), where is a root of f(X). Indeed, the roots
of f(X) are + i with 0 i p 1. Thus f(X) is separable. Hence the extension
Z
p
K = Z
p
() is separable, as is. It is also a normal extension as it is the splitting
eld of a polynomial. Finally [K: k] = p has degree p, since K = k() with a root of
an irreducible polynomial of degree p.
10.5 Let k K be an extension of elds of characteristic p > 0. Show that K is purely
inseparable over k if and only if the minimal polynomial of over k has only one root in
K.
Answer: Suppose that a =
p
e
k with e minimal. Then we have shown that f(X) =
X
p
e
a k[X] is the minimal polynomial of . Clearly, f(X) has only one root, since
f(X) = (X )
p
e
in K[X]. Conversely, suppose that the minimal polynomial f(X) of
over k factors as f(X) = (X )
n
in K[X] (or actually, in K[X]). Write n = p
f
l
with p [ l. Thus we conclude that k[X] f(X) = ((X )
p
f
)
l
= (X
p
f

p
f
)
l
=
X
p
f
l
l
p
f
X
p
f
(l1)
+ . Hence l
p
f
k and thus
p
f
k, since p [ l which implies
that l 1 k

. Thus is purely inseparable.


10.6 Let k K be a nite extension of elds of characteristic p > 0. Show that if p [ [K: k],
then k K is separable. Is the converse true?
Answer: If the extension k K is not separable then there exists an element K k
which is inseparable over k. Thus its minimal polynomial h(X) is such that h(X) k[X
p
].
Thus [k(): k] = np = deg
X
(h(X)) for some n. Hence p [ [K: k] = [K: k()][k(): k]...a
contradiction. The converse is false, as shown in example 10.4. That is, we can have
separable extensions of degree p.
10.7 Let k K be an algebraic extension of elds of characteristic p > 0, let L be an al-
gebraically closed eld containing K, and let : k L be an embedding. Show that
k K is purely inseparable if and only if there exists exactly one embedding : K L
over .
Answer: We have seen that if k K is a purely inseparable algebraic extension then
there exists a unique embedding over . Conversely, we know that [k
sep
: k] = [K: k]
s
=
1 since there is exactly one extension of . Thus k = k
sep
. We also showed that
k = k
sep
K is purely inseparable.
10.8 Let k K = k(, ) be an algebraic extension of elds of characteristic p > 0, where
is separable over k and is purely inseparable over k. Show that K = k( +).
Answer: Let K L be a splitting eld of the minimal polynomial f(X) of over k
and let
1
= , ...,
n
be the distinct roots of f(X). As customary, let g(X) = X
p
e
b =
(X )
p
e
k[X] be the minimal polynomial of . Dene = + , and consider the
polynomials f(X) and h(X) = g( X) both in k()[X]. Observe that f() = h() = 0,
while h(
i
) ,= 0 for i 2 since = + ,=
i
+ for i 2. Thus g.c.d(f(X), h(X)) =
(X )
m
. Since is separable we conclude that m = 1. So that k(). But then
k() as well, so that K = k(, ) = k().
10.9 Let f(X) F
q
[X] be irreducible. Show that f(X) divides X
q
n
X if and only if deg f(X)
divides n.
Answer:
10.10 Show that in a nite eld, every element can be written as a sum of two perfect squares.
Answer: Since k is a nite eld we have that [k[ = p
n
for some n. Let a k and consider
the two sets
1
= a b
2
[ b k and
2
= c
2
[ c k. Observe that, with the exception
of 0, an element and its opposite give raise to the same element in
i
for i = 1, 2. Thus
[
i
[ = 1/2(p
n
1) + 1 for i = 1, 2. Thus [
1
[ + [
2
[ = p
n
+ 1 > [k[. Thus we can nd
d
1

2
, so that a b
2
= d = c
2
for some b, c k, and hence a = b
2
+c
2
.
CHAPTER 11
Exam Material
Old Prelims Questions in Group Theory.
1. Show that a group of order 296 is not a simple group.
2. Let G be a group of order 168. Prove that if G has a subgroup of order 28 then G is not
simple. (Hint: consider the action of G on some set.)
3. Prove that an innite simple group cannot have a proper subgroup of nite index.
4. Let G be a group of order 140 and let H be a subgroup of order 28. prove that G is not
simple.
5. Let G be a group of order 126 and let H be a subgroup of order 21. prove that G is not
simple.
6. Let G be a nite simple group having a subgroup H of index n > 2.
Show that there is an isomorphism : G S
n
.
Show that (G) A
n
.
7. Let p be a prime, G a nite p-group and n a positive integer not divisible by p. If S has
n elements and if G acts on S, prove that there is some s S such that g s = s for every
g G.
8. Prove that a nite group G is a p-group if and only if the order of G is p
n
for some n > 0.
9. Argue that a group of order 143 is necessarily cyclic.
10. Let G be a group with 48 elements and that it contains a normal subgroup of order 3.
Argue that it contains a normal subgroup of order 24.
11. Let G be a nite group and let a G. Let S be the conjugacy class of a. Prove that the
number of elements in S divides the order of the group G.
12. Let G be a group of order pq, where p and q are primes such that p < q and such that
(p, q 1) = 1. Prove that G is cyclic.
13. Argue that there is no simple group of order 132.
14. If N G and N, G/N are both p-groups then G is a p-group.
105
106 11. EXAM MATERIAL
First Midterm - October 29, 2001.
PART I
Give full and detailed explanations for each of the next questions
Each question is worth 5 points
1. State the Fundamental Theorem of Arithmetic.
2. Diagram the lattice of subgroups of the cyclic group Z
45
.
3. Suppose G
1
and G
2
are groups and N
i
is a normal subgroup of G
i
, i = 1, 2.
If N
1

= N
2
and G
1
/N
1

= G
2
/N
2
, prove or disprove that G
1
is isomorphic to G
2
.
4. Let H be a subgroup of a group G. Dene the centralizer C
G
(H) of H in G.
Dene the normalizer N
G
(H) of H in G. Dene the center Z(G) of G.
5. Let G be the cyclic group Z
12
. What is the order of Aut(Z
12
) ? Is the group Aut(Z
12
)
cyclic? What is Aut(Z
12
) isomorphic to?
6. What is the commutator (derived) subgroup G
t
of a group G?
Explain why G
t
G and G/G
t
is abelian. What can you conclude if G
t
= 1?
7. State the Second Isomorphism Theorem.
8. What does it mean that a group G is solvable? Explain why D
8
is solvable.
9. What does it mean that a group G acts on a nonempty set A?
What does it mean that a group G acts on itself by left multiplication? by conjugation?
10. Let =
_
1 2 3 4 5 6 7 8 9
3 5 4 2 9 8 6 7 1
_
be a permutation in S
9
.
Write the cycle decomposition of and of
1
. Also, compute [[, sign().
PART II
Give full and detailed proofs for each of the next problems
Each problem is worth 10 points
I. Let G be a group and assume that g
2
= 1 for every g G. Show that G is abelian.
II. Let H and K be subgroups of a group G. If H K is a subgroup of G, prove that either
H K or K H.
III. Let G be a group such that G/Z(G) is abelian. If 1 ,= H G then prove that
H Z(G) ,= 1.
IV. Let G be a nite group and let H be a proper subgroup of G. Show that
_
gG
gHg
1
,= G.
(Hint: Count the number of elements of the left hand side.)
V. Let H G and N G be subgroups of a nite group G.
Show that if ([H[, [G: N]) = 1 then H N. (Hint: Whats the order of HN/N ?)
11. EXAM MATERIAL 107
bbllx= 0pt,bblly= 65pt,bburx= -70pt,
UNIVERSITY
OF KENTUCKY
Algebra Preliminary Examination
Department of Mathematics
May 30, 2001 3 hours time
Provide all the required proofs and clearly formulate your statements. Problems 1. and 4. are
considered essential.
1. (a) State Sylows Theorem for the nite group G.
(b) Let G be a group. Dene what is meant for G to be solvable.
(c) Let G be a nite group of order p
2
q, where p and q are distinct primes. Use (a) to
prove that G is solvable.
2. Let G be a group of order 312. Use 1.(a) to show that G has a normal Sylow subgroup.
3. Let f(x) be an irreducible polynomial in F[x], where F is a eld.
(a) Construct a eld E in which f(x) has a root.
(b) If deg f(x) = n, construct an F-basis for E.
4. Let K be a nite extension of the eld F.
(a) Dene what it means for K/F to be a Galois extension.
(b) Give two characterizations of K/F to be a Galois extension.
(c) Let K/F be a Galois extension with Galois group G. State the Fundamental Theorem
of Galois Theory.
(d) Let f(x) = x
5
3 [x]. Construct a splitting eld K for f(x) and determine the
Galois group G = Gal(K/).
5. (a) Dene what it means to say that a ring R is an Euclidean domain.
(b) Show that Z[i] = a +ib [ a, b Z form an Euclidean domain.
6. (a) State Gauss Lemma.
(b) State Eisensteins Criterion.
(c) Show that f(x) = 2x
5
6x
3
+ 9x
2
15 is irreducible in both [x] and Z[x].
7. Let R be a commutative ring with identity and let I and J be two ideals such that
I + J = R. First, dene what the product IJ is. Then, show that: IJ = I J and
R/IJ

= R/I R/J.
8. Let I be an ideal in a commutative ring R and let: Rad(I) = r R[ r
n
I for some n.
(Rad(I) is referred to as the radical of I.)
(a) Show that I Rad(I).
(b) Show that Rad(I) is an ideal of R.
(c) Show that Rad(Rad(I)) = Rad(I).
9. (a) Dene the notion of linear transformation T between two vector spaces V and W.
How can you represent T if both V and W are nite dimensional?
(b) Let V be the vector space of all n n matrices dened over 1, and let B be a xed
n n matrix. For any A in V dene the map T by letting: T(A) = AB BA.
i. Verify that T is a linear transformation from V to V .
ii. What is the nullspace of T?
iii. What is T
2
(A) = T(T(A))?
108 11. EXAM MATERIAL
bbllx= 0pt,bblly= 65pt,bburx= -70pt,
UNIVERSITY
OF KENTUCKY
Algebra Preliminary Examination
Department of Mathematics
January 7, 2002 3 hours time
Provide all the required proofs and clearly formulate your statements.
1. Let H be a subgroup of the nite group G. State and prove Lagranges Theorem with
respect to H.
2. Argue that there is no simple group of order 432.
3. (a) State Sylows Theorem for the nite group G.
(b) Let T be a normal subgroup of G and let P be a p-Sylow subgroup of T, p a prime.
Prove that G = TN
G
(P).
4. Let m be a maximal ideal in Z[x] with m Z ,= 0. Show that m = (p, f), where p is a
prime number in Z and f is a monic polynomial.
5. Let R be a commutative ring with 1 ,= 0. An ideal Q is called a primary ideal if xy Q
and x , Q implies y
n
Q for some n. Show that

Q is a prime ideal of R.
6. Let R be a commutative ring with 1. A derivation on R is a homomorphism D: R R
of additive groups satisfying D(ab) = aD(b) +bD(a) for every a, b R. Show that
(a) Ker(D) is a subring of R containing 1.
(b) 0 is the only derivation on Z[i].
7. Determine the Galois group of X
4
5 [X].
8. Show that the polynomial X
5
4X + 2 [X] is not solvable by radicals.
9. Determine the Galois group of the polynomial X
4
2X + 2 over .
10.
11.
Bibliography
[1] Dummit and Foote, .
109

You might also like